Download as pdf or txt
Download as pdf or txt
You are on page 1of 99

ATOMIC

CHAPTER
2
STRUCTURE
Recap of Early Classes
In earlier classes we have studied basic mole concept. We have also learnt that an atom consists of
certain fundamental particles – protons, neutrons and electrons. The existence of atoms has been proposed
since the time of early Indian and Greek philosophers (400 B.C.) who were of the view that atoms are the
fundamental building blocks of matter. According to them, the continued subdivisions of matter would ultimately
yield atoms which subdivisions of matter would not be further divisile. This chapter aims of understand varies
theories experiments which late to the development of structure of atom.

1.0 INTRODUCTION TO ATOM 11.4 Schrodinger Equation in Polar


2.0 DALTONS ATOMIC THEORY Coordinates
3.0 THOMSON'S MODEL OF ATOM [1904] 11.5 Quantum Number
11.6 Radial probability distribution function
4.0 RUTHERFORD'S SCATTERING
(rpdf), 4p r2y 2(r)
EXPERIMENT
11.7 Angular Probability distrubtion function
4.1 Applications of Rutherford Model
12.0 ENERGY LEVEL DIAGRAM
5.0 WAVE THEORY
12.1 Energy level diagram of Hydrogen atom
5.1 Wave characteristics
12.2 Energy level diagram of multi-electron
5.2 Electromagnetic waves (EM waves) atoms
or Radiant Energy/Electromagnetic
12.3 Bohr Bury (n + l) rule (for multi-electronic
radiation atoms)
6.0 PLANCK'S QUANTUM THEORY 13.0 RULES FOR FILLING OF ORBITALS IN AN
6.1 Photoelectric effect ATOM
7.0 BOHR'S ATOMIC MODEL 13.1 Aufbau Principle
7.1 Radii of various orbits of hydrogen atom 13.2 Pauli's Exclusion principle
7.2 Calculation of energy of an electron 13.3 Hund's Maximum Multiplicity Rule
7.3 Calculation of velocity 13.4 Electronic configuration of atoms of various
7.4 Terms associated with Bohr's model as elements
per energy considerations 13.5 Exception of Aufbau principle
8.0 SPECTRUM 13.6 Electronic configuration of ions
8.1 Emissions spectrum 14.0 MAGNETIC PROPERTIES
8.2 Absorption spectrum EXERCISE-1
8.3 Hydrogen line spectrum or Hydrogen EXERCISE-2
spectrum
EXERCISE-3
9.0 THE DUAL NATURE OF MATTER (THE
EXERCISE-4(A)
WAVE NATURE OF ELECTRON)
EXERCISE-4(B)
10.0 HEISENBERG UNCERTAINTY
EXERCISE-5
PRINCIPLE
11.0 WAVE MECHANICAL MODEL OF ATOM
11.1 Concept of Orbitals
11.2 The wave function
11.3 Schrodinger Equation in Cartesien
Coordinates
SDFD
Atomic Structure
1.0 INTRODUCTION TO ATOM
The ideas of classical mechanics developed by Galileo, Kepler and Newton, when applied to atomic and
molecular systems were found to be inadequate. Need was felt for a theory to describe, correlate and predict
the behaviour of the sub-atomic particles. The quantum theory, proposed by Max Planck and applied by Einstein
and Bohr to explain different aspects of behaviour of matter, is an important milestone in the formulation of
the modern concept of atom.
In this chapter, we will study how black body radiation, photoelectric effect and atomic spectra of hydrogen
can be explained on the basis of theories proposed by Max Planck, Einstein and Bohr. They based their theories
on the postulate that all interactions between matter and radiation occur in terms of definite packets of energy,
known as quanta. Their idea, when extended further, led to the evolution of wave mechanics, which shows the
dual nature of matter and energy.

2.0 DALTONS ATOMIC THEORY


SL
This theory is based on law of mass conservation and law of definite proportions.
The salient feature's of this theory are :
(i) Each element is composed of extremely small particles called atoms.
(ii) Atoms of a particular element are like but differ from atom's of other element.
(iii) Atom of each element is an ultimate particle and it has a characteristic mass but is structureless.
(iv) Atom's are indestructible i.e. they can neither be created nor be destroyed.
(v) Atom of element's take part in chemical reaction to form molecule.

3.0 THOMSON'S MODEL OF ATOM [1904]


SL

(i) Thomson was the first to propose a detailed model of the


atom.
(ii) Thomson proposed that an atom consists of a uniform
sphere of positive charge in which the electrons are
present at some places.
(iii) This model of atom is known as 'Plum-Pudding model'.

l Drawbacks
(i) An important drawback of this model is that the mass of the atoms is considered to be evenly spread over
that atom.
(ii) It is a static model. It does not reflect the movement of electron.

4.0 RUTHERFORD'S a–SCATTERING EXPERIMENT


SL AL

a-scattering experiment
JPR\COMP.251\D\Allen(IIT-JEE Wing)\2020–21\Nurture\Che\Unit-01\Atomic Structure

41
JEE-Chemistr y
l Ruther ford observed that
(i) Most of the a-particles (nearly 99.9%) went
straight without suffering any deflection.
(ii) A few of them got deflected through small
angles.
(iii) A very few a-particles (about one in 20,000)
did not pass through the foil at all but suffered
large deflections (more than 90°) or even
come back in the direction from which they
have come i.e. a deflection of 180°.

l Following conclusions were drawn from the above observations


(i) Since most of the a-particle went straight through the metal foil undeflected, it means that there must be
very large empty space within the atom.
(ii) Since few of the a-particles were deflected from their original path through moderate angles; it was
concluded that whole of the +ve charge is concentrated and the space occupied by this positive charge
is very small in the atom.Whenever a-particles come closer to this point, they suffer a force of repulsion and
deviate from their paths.The positively charged heavy mass which occupies only a small volume in an
atom is called nucleus.It is supposed to be present at the centre of the atom.

(iii) A very few of the a-particles suffered strong deflections on even returned on their path indicating that the nucleus
is rigid and a-particles recoil due to direct collision with the heavy positively charged mass.

4.1 Applications of Rutherford Model


SL AL

On the basis of scattering experiments, Rutherford proposed the model of an atom, which is known as nuclear
atomic model. According to this model -
(i) An atom consists of a heavy positively charged nucleus where all the protons are present.
(ii) The volume of the nucleus is very small and is only a minute fraction of the total volume of the atom.Nucleus
has a radius of the order of 10-13 cm and the atom has a radius of the order of 10-8 cm

rA radius of the atom 10-8


= = -13 = 105, rA = 105 rN
rN radius of the nucleus 10

Thus radius (size) of the atom is 105 times the radius of the nucleus.
The radius of a nucleus is proportional to the cube root of the no. of nucleons within it.
R µ A1/3 Þ R = R0A1/3 cm
Where R0 = 1.33 ´ 10-13(a constant) and, A = mass number (p + n)
R = radius of the nucleus.
R = 1.33 × 10–13 A1/3 cm
(iii) There is an empty space around the nucleus called extra nuclear part. In this part electrons are present.The
JPR\COMP.251\D\Allen(IIT-JEE Wing)\2020–21\Nurture\Che\Unit-01\Atomic Structure

no. of electrons in an atom is always equal to no. of protons present in the nucleus. As the nuclear part
of atom is responsible for the mass of the atom, the extra nuclear part is responsible for its volume. The
volume of the atom is about 1015 times the volume of the nucleus.
æ4 3ö
( )
3
vol.of the atom ç prA ÷ 10 -8
è 3 ø
= = = 1015
vol.of the nucleus æ 4 3 ö
ç prN ÷
è3 ø
10 (
-13 3
)
(iv) Electrons revolve round the nucleus in closed orbits with high speeds.
This model was similar to the solar system, the nucleus representing the sun and revolving electrons as
planets.

42
Atomic Structure
Distance of closest approach and estimating nuclear dimension from a-particle scattering experiment:
An a-particle traveling directly towards the centre of the nucleus slows down as it approaches the nucleus. At a
certain distance say r0 from the nucleus, the a-particle comes to rest and its initial kinetic energy is completely
converted into electrostatic potential energy. The distance r0 called the distance of closest approach.
Nucleus
(+Ze)

A u B
+
a-particle r0
(+2e)

At point B, K.E. of a-particle is completely converted to electrostatic potential energy (P.E.) of the system.
1 (2e)(Ze) 1
P.E. of system at B = ´ K.E. of a-particle = ´ m a ´ u2
4 pe 0 r0 2

2 Z e2 2Ze 2
Equating both we get , r0 = =
1
4 pe0 ´ ´ m a u 2 4 pe0 ´ K.E.
2

Illustration 1. An a-particle having K.E. = 7.7 MeV is scattered by gold (Z = 79) nucleus through 180°. Find
distance of closest approach.
Solution. K.E. = 7.7 M eV
= 7.7 × 106 × 1.6 × 10–19 J = 1.23 × 10–12 J
1
= 9 ´ 10 9 Nm 2 C -2
4 pe 0

9 ´ 10 9 ´ 2 ´ 79 ´ (1.6 ´ 10 -19 ) 2
Using we get :
1.23 ´ 10 -12
r0 = 3 × 10–14 m
From the above example it is clear that nuclear dimension cannot be greater than 3 × 10–14 m.

l Drawbacks of Rutherford model

(i) This theory could not explain the stability of an atom. According
to Maxwell electron loses it's energy continuously in the form of
electromagnetic radiations. As a result of this, the e- should
loss energy at every turn and move closer and closer to the
nucleus following a spiral path. The ultimate result will be that
it will fall into the nucleus, thereby making the atom unstable.
JPR\COMP.251\D\Allen(IIT-JEE Wing)\2020–21\Nurture\Che\Unit-01\Atomic Structure

(ii) If the electrons loss energy continuously, the observed spectrum should be continuous but the actual
observed spectrum consists of well defined lines of definite frequencies (discontinuous). Hence, the loss of
energy by electron is not continuous in an atom.

5.0 WAVE THEORY


SL AL

A wave is difined as a periodic disturbance in space or in a medium that involves elastic displacement of
material particles or a periodic change in some physical quantities such as T, P, V etc. Thus, wave motion
represents propagation of a periodic disturbance carying energy. The wave travels at right angles to the vibratory
motion of the object.

43
JEE-Chemistr y

5.1 Wave characteristics


SL AL

A wave is characterized by following six characterstics.


The upper most point of the wave is called crest and the
lower most point is called trough. Some of the terms
employed in dealing with the waves are described below.
l Wavelength l (Lambda)
(i) It is defined as the distance between two nearest crest or nearest trough.
(ii) It is measured in terms of a A° (Angstrom), pm (Picometre), nm (nanometer), cm(centimetre), m (metre)
1Å = 10–10 m, 1 pm = 10–12 m, 1nm = 10–9 m, 1cm = 10–2m

l Frequency (n) (nu)®


(i) Frequency of a wave is defined as the number of waves which pass through a point in 1 sec.
(ii) It is measured in terms of Hertz (Hz ), sec–1 , or cycle per second (cps)
1 Hertz = 1 sec–1 = 1 cps.

l Time period (T)


Time taken by a wave to pass through one point.
1
T= sec.
n
l Velocity ® (c)
Velocity of a wave is defined as distance covered by a wave in 1 sec.
C = l /T = ln Þ n = C/l
Since C is constants n µ 1/l
i.e. frequency is inversely proportional to l

l Wave number ® ( n ) ( nu bar) ®


It is the reciprocal of the wave length that is number of waves present in 1cm
1 m = 100 cm
1 1 100
n= =
l cm m
(1cm = 100 m-1)
–1

It is measured in terms of cm–1, m–1 etc,

l Amplitude ® (a)
The amplitude of a wave is defined as the height of crust or depth of trough.
C æ 1ö
n= = Cn ç n = ÷
l è lø

5.2 Electromagnetic waves (EM waves) or Radiant Energy/Electromagnetic radiation


JPR\COMP.251\D\Allen(IIT-JEE Wing)\2020–21\Nurture\Che\Unit-01\Atomic Structure

SL AL

• It is the energy transmitted from one body to another in the form of waves and these waves travel in the
space with the same speed as light ( 3 × 108 m/s) and these waves are known as Electromagnetic waves
or radiant energy.
• The radiant Energy do not need any medium for propogation.
Ex : Radio waves, micro waves, Infra red rays, visible rays, ultraviolet rays, x–rays, gama rays and cosmic
rays.

44
Atomic Structure

decreasing frequency
1026 1024 1022 1020 1018 1016 1014 1012 1010 108 106 104 102 100
v(Hz)

Cosmic g rays X rays UV IR Microwave FM AM


rays Long radio waves
Radio waves
l(m)
10–18 10–16 10–14 10–12 10–10 10–8 10–6 10–4 10–2 100 102 104 106 108

increasing wavelengths
V I B G Y O R

let igo e e n low nge


Vio Ind Blu Gr e Ye l O r a Red
3800 4300 4500 4900 5500 5900 6500 7600
Visible spectrum (in Å)
Wavelength l (nanometers)

Illustration 2. The vividh Bharti station of All India Radio broadcast on a frequency of 1368 Kilo Hertz. Calculate
the wave length of the Electromagnetic waves emited by the transmitter.
Solution As we know velocity of light (C)
C = 3 × 108 m/sec.
Given n (frequency)
= 1368 kHz
= 1368 × 103 Hz
= 1368 × 103 sec–1

C 3 ´ 108 m sec -1
Q l= \l= Þ l = 219.3 m
n 1368 ´ 10 3 sec -1

Illustration 3. Calculate n in cm–1 and n of yellow radiations have wavelength of 5800 Å


1
Solution. As we known n =
l
1
n=
5800Å

1
n=
5800 ´ 10 -8 cm {Q 1Å = 10–8 cm}

108
= cm–1 = 17241.4 cm–1
JPR\COMP.251\D\Allen(IIT-JEE Wing)\2020–21\Nurture\Che\Unit-01\Atomic Structure

5800
n = cn
= 3 × 1010 cm sec–1× 1.7 × 104 cm–1
= 3 × 1.7 × 1014
= 5.1 × 1014 sec-1

Illustration 4. How long would it take a radio wave of frequency 6 × 103 sec–1 to travel from mars to the earth, a
distance of 8 × 107 km ?
Solution. Distance to be travelled from mars to earth
= 8 × 107 km
= 8 × 1010 m
Q Velocity of EM waves = 3 × 108 m/sec

45
JEE-Chemistr y

Dis tan ce 8 ´ 1010 m


\ Time= = = 2.66 × 102 sec.
Velocity 3 ´ 108 m / sec -1

Illustration 5. What will be the frequency of photon of wavelength 2225 Å traveling in vacuum ?
Solution. Velocity of light in vacuum = 3 × 108 m sec–1
Wavelength = 2225 × 10–10 meter

Velocity 3 ´ 108 meter / sec 3000


Frequency = = = ´ 1015 sec -1 = 1.349 × 1015 sec–1
Wavelength 2225 ´ 10 -10 meter 2225

Earlier Model of Atom/Wave Theory


1. The mass to charge ratio for A+ iobns is 1.97 × 10–7 kg C–1. Calculate the mass of A atom.

2. Atomic radius is of the order of 10–8 cm and nuclear radius is of the order of 10–13 cm. Calculate what fraction
of atom is occupied by nucleus ?

3. With what velocity should an a–particles travel towards the nucleus of a Cu atom so as to arrive at a distance
of 10–13 m.

4. How long would it take a radio wave of frequency 6 × 103 sec–1 to travel from mars to the earth, a distance
of 8 × 107 km ?

5. Rutherfords experiment , which established the nuclear model of atom, used a beam of :–
(A) b - particles, which impinged on a metal foil and get absorbed.
(B) g - rays, which impinged on a metal foil and ejected electron.
(C) Helium atoms, which impinged on a metal foil and got scattered.
(D) Helium nuclie, which impinged on a metal foil and got scattered.

6. When alpha particles are sent through a thin metal foil, most of them go straight through the foil because -
(A) alpha particles are much heavier than electrons (B) alpha particles are positively charged
(C) most part of the atom is empty space (D) alpha particles move with high speed

7. For a wave, frequency is 10 Hz and wavelength is 2.5 m. How much distance it will travel in 40 seconds.

6.0 PLANCK'S QUANTUM THEORY


SL AL

The electromagnetic theory of radiations given by Maxwell regarded radiant energy as a continuous flow of
energy in the form of waves. This theory, no doubt , explained certain phenomena associated with light. But at
the same time , it failed to account for a few more phenomena.
These are listed as follows :
JPR\COMP.251\D\Allen(IIT-JEE Wing)\2020–21\Nurture\Che\Unit-01\Atomic Structure

(a) The nature of radiations which are emitted by a black body also known as black body radiations.
(b) The ejection of electrons from the surface of certain metals when light radiations strike the surface. This
is known as photoelectric effect.
(c) The spectral lines emitted by excited atomic gases. These are known as line spectrum and help in the
identification of the elements.
Features of planck's quantum theory :
(i) The radiant energy emitted or absorbed by a body not continuously but discontinuously in the form of
small discrete packets of energy and these packets are called quantum.
(ii) In case of light, the smallest packet of energy is called as 'photon' but in general case the smallest packet
of energy called as quantum.
(iii) The energy of each quantum is directly proportional to frequency of the radiation i.e.

46
Atomic Structure

hc ì cü
Eµn Þ E = hn or E= íQ n = ý
l î l þ
Proportionality constant or Planck's constant (h)
h = 6.626 × 10–37 kJ sec.
or 6.626 × 10–34 J sec (1erg = 10–7J)
or 6.626 × 10–27 erg sec.
(iv) Total amount of energy transmitted from one body to another will be some integral multiple of energy of
a quantum.
E = nhn
Where n is an integer and n = number of quantum
hc
E = hn = = hcn
l

Illustration 6. Calculate the energy of a photon of sodium light of wave length 5.862 × 10–16 m in Joules.
Solution. l = 5.862 × 10–16 m
c = 3 × 108 m sec–1
nhc hc
E = nhn or {Q n = 1} \E=
l l

1 ´ 6.6 ´ 10 -34 Joule ´ 3 ´ 10 8 m sec -1


E=
5.862 ´ 10 -16 m

= 6.6 ´ 3 ´ 10 -10 Joules = 3.38 × 10–10 Joules.


5.862

Illustration 7. Calculate the frequency & energy of a photon of wave length 4000 Å
Solution. (a) Calculation of frequency :
l = 4000 Å
l = 4000 × 10–10 m
C
Q n=
l
3 ´ 108 m / sec
\ n=
4 ´ 10 -7 m
= 0.75 × 1015 sec–1 = 7.5 × 1014 sec–1
(b) Calculation of energy :
E = hn
= 6.626 × 10–34 Joule × 7.5 × 1014 sec–1
= 4.96 × 10–19 Joule

Illustration 8. Calculate the l and frequency of a photon having an energy of 2 electron volt
Solution. Q 1ev = 1.602 × 10–19 J
\ 2ev = 3.204 × 10–19 J = E
(a) Calculation of wavelength (l) :
JPR\COMP.251\D\Allen(IIT-JEE Wing)\2020–21\Nurture\Che\Unit-01\Atomic Structure

hc hc
E= or l =
l E
6.626 ´ 10-34 Js ´ 3 ´ 108 m sec -1
=
3.204 ´ 10-19 J
= 6.204 × 10–7 m
(b) Calculation of frequency (n) :
c 3 ´ 108 m sec -1
n= =
l 6.204 ´ 10-7 m
= 0.48 × 1015 sec–1
= 4.8 × 1014 sec–1

47
JEE-Chemistr y
Illustration 9. Which has a higher energy ?
(a) A photon of violet light with wave length 4000 Å
or
(b) A photon of red light with wave length 7000 Å
Solution. (a) Violet light :
hc
Eviolet =
l

6.626 ´ 10 -34 J sec´ 3 ´ 108 m sec -1


= = 4.97 × 10–19 Joule
4000 ´ 10 -10 m
(b) Red light :
hc
Ered =
l

6.626 ´ 10 -34 J sec´ 3 ´ 108 m sec -1


= = 2.8 × 10–19 Joule
7000 ´ 10 -10 m
So, Eviolet > Ered

Illustration 10. How many photons of lights having a wave length of 5000 Å are necessary to provide 1 Joule of
energy.
nhc
Solution. Q E =
l
E´l
\ n=
hc

1Joule ´ 5000 ´ 10 -10 m


= = 2.5 × 1018 photons
6.626 ´ 10-34 Joule sec ´ 3 ´ 108 m sec -1

6.1 Photoelectric effect


SL AL

(i) Sir J.J. Thomson observed that when a light of certain frequency strikes the surface of a metal, electrons
are ejected from the metal. This phenomenon is known as photoelectric effect and the ejected electrons
are called photoelectrons.
(ii) A few metals, which are having low ionisation energy like Cesium, show this effect under the action of
visible light but many more show it under the action of more energetic ultraviolet light.
Light

– electrons
+

Evacuated quartz tube


V

– +
A
JPR\COMP.251\D\Allen(IIT-JEE Wing)\2020–21\Nurture\Che\Unit-01\Atomic Structure

The experimental findings are summarized as below :


• Electrons come out as soon as the light (of sufficient energy) strikes the metal surface.
• The light of any frequency will not be able to cause ejection of electrons from a metal surface. There
is a minimum frequency, called the threshold (or critical) frequency, which can just cause the ejection.
This frequency varies with the nature of the metal. The higher the frequency of the light, the more energy
the photoelectrons have. Blue light results in faster electrons than red light.
• Photoelectric current is increased with increase intensity of light of same frequency, if emission is permitted,
i.e. a bright light yields more photoelectrons than a dim one of the same frequency, but the electron energies
remain the same.
• Light must have stream of energy particles or quanta of energy (hn). Suppose, the threshold frequency
of light required ejecting electrons from a metal is n0, when a photon of light of this frequency strikes
a metal it imparts its entire energy (hn0) to the electron.
48
Atomic Structure

E=hn0 E>hn0

K.Emax = hn – h n0
K.E.=0
Metal

"This energy enables the electron to break away from the atom by overcoming the attractive influence of the
nucleus". Thus each photon can eject one electron. If the frequency of light is less than n0 there is no ejection
of electron. If the frequency of light is higher than n0 (let it be n), the photon of this light having higher energy
(hn), will impart some energy to the electron that is needed to remove it away from the atom. The excess energy
would give a certain velocity (i.e. kinetic energy) to the electron.
hn = hn0 + K.E.
hn = hn0 + ½ mn2
½ mn2 = hn – hn0
where n = frequency of the incident light
n0 = threshould frequency
hn0 is the threshold energy (or) the work function denoted by f = hn0 (minimum energy of the photon to liberate
electron). It is constant for particular metal and is also equal to the ionization potential of gaseous atoms.
The kinetic energy of the photoelectrons increases linearly with the frequency
of incident light. This, if the energy of the ejected electrons is plotted as K.E.
a function of frequency, it result in a straight line whose slope is equal to of
Planck's constant 'h' and whose intercept is hn0. Photoelectrons

n
v Stopping Potential (V0)
Light

– electrons
+

Evacuated quartz tube


V

– + µA

When a suitable radiation is incident on the surface of metal plate, electrons are ejected from its surface. If the
collector is at a positive terminal w.r.t. emitter E, the electrons are attracted by it. It leads to flow of current,
called Photocurrent in the circuit, which is measured by a micro ammeter (µA). It will depend on no. of
electrons ejected and K.E. of ejected electrons.
If the collector is maintained at a negative potential w.r.t. the emitter,
the electrons are repelled by it. The electrons , which have sufficient
kinetic energy, reach the collector despite its negative polarity.
Now , if we increase the reverse potential , the photo V0
JPR\COMP.251\D\Allen(IIT-JEE Wing)\2020–21\Nurture\Che\Unit-01\Atomic Structure

current gradually decreases and becomes zero at a particular rever se


potential. This minimum applied reverse potential to stop photocurre
nt is called Stopping Potential (V0). Hence the maximum kinetic
v0
ene rgy of photo electrons can be written as -
K.E.max = eV0 = hu – f v (frequency)

49
JEE-Chemistr y

Illustration 11. A photon of wavelength 3000 Å strikes a metal surface, the work function of the metal being 2.20
eV. Calculate
(i) the energy of the photon in eV
(ii) the kinetic energy of the emitted photo electron and
(iii) the velocity of the photo electron.
Solution. (i) Energy of the photon

E = hn =
hc
=
( )(
6.6 ´ 10 -34 Js 3 ´ 108 ms -1 )
= 6.6 × 10–19 J
l 3 ´ 10 -7 m
1eV = 1.6 × 10–19 J
6.6 ´ 10 -19 J
Therefore E = = 4.125 eV
1.6 ´ 10 -19 J / ev
(ii) Kinetic energy of the emitted photo electron
Work function = 2.20 eV
Therefore, KE = 2.475 – 2.20 = 1.925 eV = 3.08 × 10–19 J
(iii)Velocity of the photo electron
1
KE = mv2 = 3.08 × 10–19 J
2

2 ´ 3.08 ´ 10 -19
Therefore, velocity (v) = = 8.22 × 105 ms–1
9.1 ´ 10 -31

Illustration 12. Photoelectrons are liberated by ultra violet light of wavelength 2000 Å from a metallic surface for
which the photoelectric threshold is 4000 Å. Calculate the de Broglie wavelength of electrons emitted
with maximum kinetic energy.
Solution. K.E. = Quantum Energy – Threshold energy

6.626 ´ 10 -34 ´ 3 ´ 108 6.626 ´ 10 -34 ´ 3 ´ 108


= -10 –
2000 ´ 10 4000 ´ 10 -10

6.626 ´ 10 -34 ´ 3 ´ 108 æ 1 - 1 ö


= ç ÷ = 4.969 × 10–19 Joule.
10 -10 è 2000 4000 ø

1
mv 2 = 4.969 × 10–19 Þ m2v2 = 2 × 4.969 × 10–19 × 9.1 × 10–31
2

h 6.626 ´ 10 -34
mv = 9.51 × 10–25 Þ l = = = 0.696 × 10–9 m
mv 9.51 ´ 10 -25
JPR\COMP.251\D\Allen(IIT-JEE Wing)\2020–21\Nurture\Che\Unit-01\Atomic Structure

Plancks Quantum Theory/Photoelectric Effect


1. Assume that 10–17 J of light energy is needed by the interior of the human eye to see an object. How many
photons of green light (l = 495 nm) are needed to generate this minimum energy.
(A) 25 (B) 30 (C) 35 (D) 40

2. A certain dye absorbs light of l = 4530 Å and then fluorescence light of 5080 Å. Assuming that under given
conditions 47% of the absorbed energy is re-emitted out as fluorescence, calculate the ratio of number of quanta
emitted out to the number of quanta absorbed.

3. A photon of 300 nm is absorbed by a gas and then re-emits two photons. One re-remitted photon has wave-
length 500 nm. Calculate energy of other photon re-emitted out.

50
Atomic Structure
4. Find out the number of photons emitted by a 60 watt bulb in one minute if wavelength of an emitted photon is
620 nm.

5. (a) Calculate the kinetic energy of a photoelectron emitted by a sodium surface when light of wavelength 400
nm is incident on it. The work function of sodium is 2.28 eV.
(b) Calculate the value of the longest wavelength which can result in the emission of a photoelectron from a
sodium surface.

6. Ultraviolet light of wavelength 280 nm is used in an experiment of photoelectric effect with lithium
(w = 2.5 eV) cathode. Find
(a) Maximum K.E. (b) Stopping potential

7. The therehold frequency n0 for a metal is 6 × 1014 s–1. Calculate the kinetic energy of an electron emitted when
radiation of frequency n = 1.1 × 1015 s–1 hits the metal.

8. When electromagnetic radiation of wavelength 310 nm fall on the surface of Sodium, electrons are emitted with
K.E. = 1.5 eV. Determine the work function (W0) of Sodium.
(A) 2.5 eV (B) 5 eV (C) 10 eV (D) 12.5 eV

9. When electromagnetic radiation of wavelength 300 nm falls on the suface of sodium, electrons are emitted with
a kinetic energy of 1.68 × 105 J mol–1. What is the minimum energy needed to remove an electron from
sodium ? What is the maximum wavelength that will cause a photoelectron to be emitted?
(A) 908 nm (B) 517 nm (C) 1010 nm (D) 205 nm

10. The thereshold frequency n0 for a metal for a metal is 7.0 × 1014 s–1. Calculate the kinetic energy of an electron
emitted when radiation of frequency n = 1.0 × 1015 s–1 hits the metal.

7.0 BOHR'S ATOMIC MODEL


SL AL

l Some Important formulae


kq1q 2
(i) Coulombic force =
r2

mv 2
(ii) Centrifugal force =
r
(iii) Angular momentum = mvr
It is a quantum mechanical model. This model was based on quantum theory of radiation and Classical law of
physics.

l The important postulates on which Bohr's Model is based are the following
1st Postulate:
• Atoms has a nucleus where all protons and neutrons are present.
• The size of nucleus is very small and it is present at the centre of the atom.

2nd Postulate :
JPR\COMP.251\D\Allen(IIT-JEE Wing)\2020–21\Nurture\Che\Unit-01\Atomic Structure

• Negatively charged electron are revolving around the nucleus in the same way as the planets are revolving
around the sun.
• The path of electron is circular.
• The attraction force (Coulombic or electrostatic force) between nucleus and electron is equal to the
centrifugal force on electron.
i.e. Attraction force towards nucleus = centrifugal force away from nucleus.

3rd Postulate :
h
• Electrons can revolve only in those orbits whose angular momentum (mvr) is integral multiple of .
2p

51
JEE-Chemistr y

nh
i.e. mvr =
2p
n = Whole number = 1, 2, 3..... h =h
Where h = Plank's constant, 2p
p = Constant
h h h h h
• Angular momentum can have values such as , 2 , 3 , 4 , 5 ........but can not have
2p 2p 2p 2p 2p
h h h
fractional values such as 1.5 , 1.2 , .5 .......
2p 2p 2p

4th Postulate :
• The orbits in which electron can revolve are known as stationary Orbits because in these orbits energy
of electron is always constant.
5th Postulate :
• Each stationary orbit is associated with definite amount of energy therefore these orbits are also called as
energy levels and are numbered as 1, 2, 3, 4, 5, .... or K, L, M, N, O, ..... from the nucleus outwards.

6th Postulate :
• The emission or absorbtion of energy in the form of photon can only occur when electron jumps from one
stationary state to another & it is D E = Efinal state – Einitial state
• Energy is absorbed when electron jumps from inner to outer orbit and is emitted when electron moves
from outer to inner orbit.

7.1 Radii of various orbits of hydrogen atom


SL AL

Consider, an electron of mass 'm' and charge 'e' revolving around a nucleus of charge Ze (where, Z = atomic
number and e is the charge of the proton) with a tangential velocity v. r is the radius of the orbit in which electron
is revolving.
By Coulomb's law, the electrostatic force of attraction between the moving electron and nucleus is Coulombic
JPR\COMP.251\D\Allen(IIT-JEE Wing)\2020–21\Nurture\Che\Unit-01\Atomic Structure

KZe 2 v
force =
r2 m
s
1 diu
K= (where Î0 is permittivity of free space) ra
4p Î0 Å
9
K = 9 × 10 Nm C 2 –2

In C.G.S. units, value of K = 1 dyne cm2 (esu)–2


mv 2
The centrifugal force acting on the electron is
r
Since the electrostatic force balance the centrifugal force, for the stable electron orbit.

52
Atomic Structure

mv 2 KZe 2
= 2 ...(1)
r r
KZe 2
(or) v2 = ...(2)
mr
According to Bohr's postulate of angular momentum quantization, we have
nh
mvr =
2p
nh
v=
2pmr
n 2h 2
v2 = ...(3)
4 p2m 2r 2
Equating (2) and (3)
KZe 2 n 2h 2
=
mr 4 p2m 2r 2
n 2h 2
Solving for r we get r =
4 p mKZe 2
2

where n = 1, 2, 3, ......, ¥
Hence, only certain orbits whose radii are given by the above equation are available for the electron. The greater
the value of n, i.e., farther the energy level from the nucleus the greater is the radius.
The radius of the smallest orbit (n = 1) for hydrogen atom (Z = 1) is r0.

( )
2
n 2h 2 12 ´ 6.626 ´ 10 -34
r0 = 2 2 = = 5.29 × 10–11 m = 0.529 Å
( )
2
4 p me K 4 ´ ( 3.14 ) ´ 9 ´ 10 -31 ´ 1.6 ´ 10 -19
2
´ 9 ´ 10 9

Radius of nth orbit for an atom with atomic number Z is simply written as
n2
rn = 0.529 × Å
Z

7.2 Calculation of energy of an electron


SL AL

The total energy (E) of the electron is the sum of kinetic energy and potential energy.
Kinetic energy of the electron = ½ mv2

KZe 2 -KZe 2
Potential energy = ò columbic force.dr =
ò r 2
.dr =
r

-KZe 2
Total energy = 1/2 mv2 – ...(4)
r
From equation (1) we know that
JPR\COMP.251\D\Allen(IIT-JEE Wing)\2020–21\Nurture\Che\Unit-01\Atomic Structure

mv 2 KZe 2
= 2
r r

KZe 2
Q ½ mv2 =
r2
Substituting this in equation (4)
KZe 2 KZe 2 KZe2
Total energy (E) = - = -
2r r 2r

53
JEE-Chemistr y
Substituting for r, gives us
2p2 mZ 2e4 K 2
E= where n = 1, 2, 3, .....
n 2h 2
This expression shows that only certain energies are allowed to the electron. Since this energy expression consist
of so many fundamental constant, we are giving you the following simplified expressions.
Z2
E = –21.8 × 10–12 × erg per atom
n2

Z2 Z2
= –21.8 × 10–19 × J per atom = –13.6 × eV per atom
n2 n2
(1 eV = 3.83 × 10–23 Kcal)
1 eV = 1.602 × 10–12 erg
(1 eV = 1.602 × 10–19 J)
Z2
[E = –313.6 × Kcal/mole (1 cal = 4.18 J)]
n2
The energies are negative since the energy of the electron in the atom is less than the energy of a free electron,
i.e. the electron is at infinite distance from the nucleus which is taken as zero. The lowest energy level of the
atom corresponds to n = 1, and as the quantum number increases, E becomes less negative.
When n = ¥, E = 0, which corresponds to an ionized atom, i.e. the electron and nucleus are infinitely separated.
H ¾® H+ + e– (ionization)
Potential Energy
Total Energy = - Kinetic energy =
2

7.3 Calculation of velocity


SL AL
We know that
nh nh
mvr = ; v=
2p 2pmr
By substituting for r we are getting
2pKZe 2
v=
nh
where excepting n and Z all are constants
Z
v = 2.18 × 108 cm/sec.
n

7.4 Terms associated with Bohr's model as per energy considerations


SL AL

Ground state :
lowest energy state of any atom or ion is called ground state of the atom.
Ground state energy of H–atom = – 13.6 ev
Ground state energy of He+ Ion = – 54.4 ev
JPR\COMP.251\D\Allen(IIT-JEE Wing)\2020–21\Nurture\Che\Unit-01\Atomic Structure

Excited State :
States of atom other than the ground state are called excited states :
n=2 first exited state
n=3 second exited state
n=4 thired exited state
n = n + 1 nth exited state
Ionisation energy (IE) :
Minimum energy required to move an electron from ground state to
n = ¥ is called ionisation energy of the atom or ion.
Ionisation energy of H–atom = 13.6 ev
Ionisation energy of He+ ion = 54.4 ev
Ionisation energy of Li+2 ion = 122.4 ev

54
Atomic Structure
Ionisation Potential (I.P.) :
Potential difference through which a free electron must be accelerated from rest, such that its kinetic energy
becomes equal to ionisation energy of the atom is called ionisation potential of the atom.
I.P. of H atom = 13.6 v
I.P. of He+ Ion= 54.4 v
Excitation Energy :
Energy required to move an electron from ground state of the atom to any other state of the atom is called
excitation energy of that state. excitation energy of 2nd state = excitation energy of 1st excited state = 1st excitation
energy = 10.2 ev.
Excitation Potential :
Potential difference through which an electron must be accelerated from rest to so that its kinetic energy become
equal to excitation energy of any state is called excitation potential of that state.
excitation potntial of thired state = excitation potntial of second exicited state = second excitation potential =
12.09 v.
Binding Energy ‘or’ Seperation Energy :
Energy required to move an electron from any state to n = ¥ is called binding energy of that state.
Binding energy of ground state = I.E. of atom or Ion.

Illustration 13. Calculate the radius of 1st,2nd,3rd,4th Bohr's Orbit of hydrogen.


n2
Solution. Radius of Bohr's orbit : r = 0.529 ×
Z

12
(a) Radius of 1st orbit : r = 0.529 × = 0.529 Å
1

22
(b) Radius of 2nd orbit : r = 0.529 × = 0.529 × 4 = 2.116 Å
1

32
(c) Radius of 3rd orbit : r = 0.529 × = 0.529 × 9 = 4.761 Å
1

42
(d) Radius of 4th orbit : r = 0.529 × = 0.529 × 16 = 8.464 Å
1

Illustration 14. Calculate the radius ratio of 2nd orbit of hydrogen and 3rd orbit of Li+2
Solution. Atomic number of H = 1
Atomic number of Li2+ = 3
2nd orbit radius of hydrogen

22
(r2)H = 0.529 ×
1
3rd orbit radius of Li+2
JPR\COMP.251\D\Allen(IIT-JEE Wing)\2020–21\Nurture\Che\Unit-01\Atomic Structure

32
(r3)Li+2 = 0.529 ×
3

22
( r2 )H 0.529 ´
Q = 1 = 4 Þ \ ( r2 )H : ( r3 )Li+2 =4:3
( r3 )Li+2 0.529 ´
32 3
3

55
JEE-Chemistr y
Illustration 15. The ratio of the radius of two Bohr's orbit of Li+2 is 1:9. what Would be their nomenclature.
(1) K & L (2) L & M (3) K & M (4) K & N

nx2
0.529 ´ n x2 1
rx 1 3
Solution. = = Þ Þ
ry 9 n 2y n 2y 9
0.529 ´
3

nx 1 K Shell
= =
ny 3 M Shell

Illustration 16. Calculate the energy of Li+2 atom for 2nd excited state.
Z2
Solution. E = –13.6 ×
n2
Q Z = 3 and e– exist in 2nd excited state, means e– present in 3rd shell i.e. n = 3

( 3 )2
\ E = –13.6 × = – 13.6 eV/atom
( 3 )2

Illustration 17. Calculate the ratio of energies of He+ for 1St & 2nd excited state .
Solution. (He+) 1st Excited state : (He+)2 2nd Excited state
+ nd
i.e. (He )2 shell : (He+)3rd shell

( 2)2 ( 2)2
–13.6 × : – 13.6 ×
( 2)2 ( 3 )2
4 4
:
4 9
1 1
:
4 9
9 : 4

Illustration 18. The ionization energy for the hydrogen atom is 13.6 ev then calculate the required energy in ev to
excite it from the ground state to 1st excited state.
Solution. Ionization energy = 13.6 eV
i.e. 1st energy state = – 13.6 eV
Energy of 1st excited state
i.e. 2nd orbit = –3.4 eV
so, E2 – E1 = – 3.4 + 13.6 = 10.2 eV

Illustration 19. If the total energy of an electron is –1.51 ev in hydrogen atom then find out K.E, P.E, orbit radius
and velocity of the electron in that orbit.
Solution. Given E = –1.5 eV
(i) E = – KE
JPR\COMP.251\D\Allen(IIT-JEE Wing)\2020–21\Nurture\Che\Unit-01\Atomic Structure

K.E = – E { Q Z = 1}
= 1.51 eV

(ii) PE = – 2 × 1.51
= – 3.02 eV
(iii) Orbit = 3rd
Z2 12
Q E = – 13.6 × ev Þ – 1.51 = – 13.6 ×
n2 n2
-13.6
n2 =
=9
-1.51
\ n=3

56
Atomic Structure

3´ 3
(iv) r = 0.529 × Å
1
= 0.529 × 9 = 4.761 Å
1 2.188 ´ 108
v = 2.188 × 108 × cm/sec = = 0.729 × 108 cm / sec
3 3

Illustration 20. Calculate the velocity of an electron placed in the 3rd orbit of the Li2+ ion. Also calculate the number
of revolutions per second that it makes around the nucleus.
(n)2
Solution. Radius of 2nd orbit = r1x ×10–8 cm
Z

= 0.529 × 10–8 ×
( 3 )2
= 1.587 × 10–8 cm
3
Velocity of electron in 2nd orbit,
Z 2.18 ´ 108 ´ 3
v = 2.18 × 108 cm/sec = = 2.18 × 108 cm/sec
n 3
1 v 2.18 ´ 108 cm / sec
No. of revolutions/sec = = = = 0.2187 × 1016
2pr / v 2pr 2 ´ 3.14 ´ 1.587 ´ 10 -8 cm
= 2.187 × 1015 rev/sec

Bohr's Atomic model (Radius, Velocity & Energy Derivation)


1. If the radius of the first Bohr orbit of the H atom is r, then for the Li2+ ion it will be :
(A) 3r (B) 9r (C) r/3 (D) r/9

2. The radii of two of the first four Bohr’s orbits of the hydrogen atom are in the ratio 1 : 4. The energy difference
between them may be:
(A) Either 12.09 eV or 3.4 eV (B) Either 2.55eV or 10.2 eV
(C) Either 13.6 eV or 3.4 eV (D) Either 3.4eV or 0.85 eV

3. The potential energy of the electron present in the ground state of Li2+ ion is represented by:
3e 2 3e 3e 2 3e 2
(A) + (B) - (C) - (D) -
4 p Î0 r 4p Î0 r 4 p Î0 r 2 4p Î0 r

4. In a certain electronic transition in the hydrogen atom from an initial state i to a final state f, the difference in the
orbit radius (ri - rf) is seven times the Bohr radius. Identify the transition.
(A) 4 ® 1 (B) 4 ® 2 (C) 4 ® 3 (D) 3 ® 1

5. The velocity of electron in the ground state of H atom is 2.185 × 108 cm/sec. The velocity of electron in the
second orbit of Li2+ ion in cm/sec would be
(A) 3.276 × 108 (B) 2.185 × 108 (C) 4.91 × 108 (D) 1.638 × 108
JPR\COMP.251\D\Allen(IIT-JEE Wing)\2020–21\Nurture\Che\Unit-01\Atomic Structure

2h
6. If the angular momentum of an electron in a Bohr orbit is , then the value of potential energy of electron
p
present in He+ ion is :
(A) – 13.6 eV (B) – 3.4 eV (C) – 6.8 eV (D) – 27.2 eV.

7. For which of the following species, Bohr model is not valid :


(A) He+ (B) H (C) Li2+ (D) H+

8. The ratio of radius of two different orbits in a H-atom is 4 : 9, then the ratio of the frequency of revolution of
electron in these orbits is :
(A) 2 : 3 (B) 27 : 8 (C) 3 : 2 (D) 8 : 27

57
JEE-Chemistr y
9. The ratio of the difference in energy between the first and second Bohr orbit to that between the second and third
Bohr orbit in a H-like species is
1 1 4 27
(A) (B) (C) (D)
2 3 9 5

10. When an electron is brought close to the nucleus of the atom from an infinite distance, the energy of the
electron-nucleus system changes such that it
(A) increases to a smaller negative value (B) increases to a greater positive value
(C) decreases to a smaller positive value (D) decreases to a greater negative value

8.0 SPECTRUM
SL AL

l Electromagnetic spectrum or EM spectrum


The arrangement obtained by arranging various types of EM waves in ordes of their increasing frequency or
decreasing wave length is called as EM SPECTRUM

low n RW MW IR Visible U.V X-ray g Cosmic high n


low E Rays high E
longer l Shorter l
When a radiation is passed through a spectroscope (Prism) for the dispersion of the radiation, the pattern
(photograph) obtained on the screen (photographic plate) is called as spectrum of the given radiation

Classification of Spectrum

(1) Emission (2) Absorption

(a) Continuous (b) line (c) band (a) line (b) band

8.1 Emissions spectrum


SL AL
When the radiation emitted from incandescence source (eg. from the candle, sun, tubelight, burner, bulb, or by
passing electric discharge through a gas at low pressure, by heating some substance at high temp) is passed
directly through the prism and then received on the screen then the obtained spectrum is called as emission
spectrum.
(a) Emission continuous spectrum or continuous spectrum :
When a narrow beam of white light is passed through a prism, it is dispersed into 7 colours from violet to
Red.
JPR\COMP.251\D\Allen(IIT-JEE Wing)\2020–21\Nurture\Che\Unit-01\Atomic Structure

(b) Emission line spectrum :


When an atomic gas is raised to incandescence source or subjected to electrical excitation, it first absorbs
energy & then gives it out as radiation. On examining these radiation through a spectroscope a spectrum
is obtained which have well defined lines,each corresponding to a definite wave length & these lines are
separated from each other by dark space. This type of Emission spectrum is called as Emission line
spectrum.

58
Atomic Structure

Ex. Emission atomic spectrum of Hydrogen :


Lyman Balmer Paschen Brackett Pfund
series series series series series

(c) Emission band spectrum :


If molecular form of the gas is used, it first absorbs energy for not only electron transition but for rotational,
vibrational and translational then emits radiations.
On examining these radiations through a spectroscope a spectrum is obtained on the screen, which are
group of closely packed lines called Bands, therefore this type of Emission spectrum is called as emission
band spectrum. Bands are separated from each other by dark space.

Note: Since band spectrum are caused by molecules therefore band spectrum are also called as molecular
spectrum.

8.2 Absorption spectrum


SL AL
When white light is first passed through a solution or vapours of chemical substance or gas and then analyzed by
spectroscope, it is observed that some dark lines are obtained in otherwise continuous spectrum.
This type of spectrum is called as Absorption spectrum.
• If white light is passed through atomic gas then the obtained spectrum is called as Absorption line
spectrum.
• If white light is passed through molecular gas then the obtained spectrum is called as Absorption band
spectrum.
JPR\COMP.251\D\Allen(IIT-JEE Wing)\2020–21\Nurture\Che\Unit-01\Atomic Structure

8.3 Hydrogen line spectrum or Hydrogen spectrum


SL AL
When an electric excitation is applied on atomic hydrogen gas at Low pressure,a bluish light is emitted. when
a ray of this light is passed through a prism, a spectrum of several isolated sharp line is obtained.The wavelength
of various lines show that spectrum lines lie in Visible, Ultraviolet and Infra red region. These lines are grouped
into different series.
59
JEE-Chemistr y
6 lines 5 lines 4 lines 3 lines 2 lines 1 line
7 Q

{
6
Far I.R. region P
Humphery series
5 O
I.R. region 0.31 eV
Pfund series
4 N
I.R. region 0.66 eV
Brackett series
Energy 3 M
Levels
Infra Red region 1.89 eV
or
Paschen series
L
2
Visible region 10.2 eV
or
Balmer series
1 K
Ultra violet region
or
Lyman series
Series Discovered by regions n 2® n 1 Number of lines
lyman lyman U.V. region n2 = 2,3,4 ... / n1=1 n2 – 1
Balmer Balmer Visible region n2 = 3,4,5 ... / n1=2 n2 – 2
Paschen Paschen Infra red (I.R.) n2 = 4,5,6 ... / n1=3 n2 – 3
Bracket Bracket I.R. region n2 = 5,6,7 ... / n1=4 n2 – 4
Pfund Pfund I.R. region n2 = 6,7,8 ... / n1=5 n2 – 5
Humphery Humphery far I.R. region n2 = 7,8,9 ... / n1=6 n2 – 6
KEY POINTS :
• First line / Starting line / Initial line ( l max. and n min )
• Last line / limiting line / Series limit ( l min and n max. )
• First line of any series = a line
Second line of any series = b line
Third line of any series = g line
(n 2 - n1 )(n 2 - n1 + 1)
• Total no. of emission lines between n2 & n1 = , (n2 > n1)
2
n(n - 1)
• For transition from any orbit 'n' to n = 1 , total no. of emission lines =
2

1 é1 1ù
• = v = RH ê 2 - 2 ú × Z2 where RH represents Rydberg's constant
l ë n1 n 2 û
value of R = 109677 cm–1 = 10967700m–1
; 109700 cm–1 ; 10970000 m–1
1
= 912Å
JPR\COMP.251\D\Allen(IIT-JEE Wing)\2020–21\Nurture\Che\Unit-01\Atomic Structure

Illustration 21. In a hydrogen spectrum if electron moves from 7 to 1 orbit by transition in multi steps then find out
the total number of lines in the spectrum.
Solution. Lyman = (n1 – 1) = 7 – 1 = 6
Balmer = (n2 – 2) = 7 – 2 = 5
Paschen = (n2 – 3) = 7 – 3 = 4
Bracket = (n2 – 4) = 7 – 4 = 3
Pfund = (n2 – 5) = 7 – 5 = 2
Humphrey = (n2 – 6) = 7 – 6 = 1
Total = 21

60
Atomic Structure
Total number of lines can be calculated as follows :

( n 2 -n1 ) éë( n 2 -n1 )+1ùû (7-1)( 6+1) 42


Total number of lines = = = =21
2 2 2

Illustration 22. In a hydrogen spectrum if electron moves from 6th to 3rd orbit by transition in multi steps then find
out the following steps :
(a) Total number of lines in spectrum (b) Total number of lines in U.V. region
(c) Total number of lines in visible region (d) Total number of lines in IR region
Solution. (a) Calculation of total number of lines :

( n 2 - n1 ) éë( n2 - n1 ) + 1ùû ( 6 - 3 ) éë( 6 - 3 ) + 1ùû


3´ 4
= = = =6
2 2 2
(b) Calculation of number lines present in U.V. region.
e– moves from 6th to 3rd orbit in multisteps.
For U.V. region, e– should be comes into 1st shell. So the number of lines in U.V. region zero.
(c) Calculation of total number of lines in visible region.
For visible region, e– should be comes into 2nd shell, so the number of lines in visible region
zero.
(d) Calculation of total number of lines in I.R. region.
In I.R. region, Paschen, Bracket and Pfund series are present.
Number of lines in Paschen series = n2 – 3
= 6–3
= 3
Number of lines in Bracket series = n2 – 4
= 6–4
= 2
Number of lines in Pfund series = n2 – 5
= 6–5
= 1
So total number of lines = 3+2+1=6

Illustration 23. In Balmer series of H atom/spectrum, which electronic transitions represents 3rd line ?
Solution. In Balmer series
3rd to 2nd ® 1line
4th to 2nd ® 2 line
5th to 2nd ® 3 line
Infinite to 2nd ® Last line or limiting line
So, Ans is
5th to 2nd line ® 3rd line

Illustration 24. In H atom if e– moves, from nth orbit to 1st orbit by transition in multi steps, if there are total number
of lines in spectrum are 10 then find out the value of n.
( n 2 - n1 ) éë( n2 - n1 ) + 1ùû
Solution. Total number of lines =
2
JPR\COMP.251\D\Allen(IIT-JEE Wing)\2020–21\Nurture\Che\Unit-01\Atomic Structure

( n 2 - n1 )( n 2 )
So, 10 =
2
n 22 – n2 – 20 = 0
n 22 – 5n2 + 4n2 – 20 = 0
n2 (n2 –5) + 4 (n2 – 5) = 0
(n2 + 4) (n2 – 5) = 0
n2 = 5

Illustration 25. Calculate the wavelength of 1st line of Balmer series in Hydrogen spectrum.
Solution. For first line of Balmer series
n1 = 2, n2 = 3

61
JEE-Chemistr y

1 é1 1ù 1 é9 - 4 ù 1 é5 ù
= R(1)2 ê - ú Þ = Rê ú Þ = Rê ú
l ë 4 9 û l ë 36 û l ë 36 û
36 36 1 36
= l= Þ l= ´ = ´ 912 Å
5R 5 R 5
= 6566.4 Å

Illustration 26. Calculate the frequency of the last line of the lyman series in hydrogen spectrum.
Solution. For last line of Lyman series n1= 1, n2 = ¥

1 æ 1 1 ö
= RZ2 çç 2 - 2 ÷÷
l è n1 n 2 ø

1 æ1 ö
= R ç - 0÷
l è1 ø
1
= R
l
1
= 109700 cm–1
l
C
u =
l
1
= C×
l
= C× R
= 3 × 108 m sec–1 × 109700 cm—1
= 3 × 1010 cm sec–1 × 109700 cm–1
= 3.29 × 1015 sec–1

Illustration 27. What will be the shortest and longest wavelength of absorption lines of hydrogen gas containing atoms
in ground state ? Give Z = 1, R = 109737.5

1 é1 1ù
Solution. = RZ 2 ê 2 - 2 ú
l ë n1 n 2 û
For shortest wavelength DE should be maximum for that n1 = 1, n2 = ¥
1
= 109737.5cm -1 × 1 é 1 - 1 ù = 109737.5 cm–1
l êë 12 ¥ úû

l = 911 × 10–8 cm
For largest wavelength DE should be minimum so n1 = 1, n2 = 2

1 é1 1ù
= 109737.5 cm–1 × ê 2 - 2 ú
l ë1 2 û
l = 1215 Å
JPR\COMP.251\D\Allen(IIT-JEE Wing)\2020–21\Nurture\Che\Unit-01\Atomic Structure

Illustration 28. A series of lines in the spectrum of atomic hydrogen lies at wavelengths 656.46, 482.7, 434.17, 410.29
nm. What is the wavelength of next line in this series.
Solution. The give series of lines are in the visible region and thus appears to be Balmer series
Therefore, n1 = 2 and n2 =? for next line
If l = 410.29 × 10–7 cm and n1 = 2
n2 may be calculated for the last line

1 é1 1ù
= R ê 2 - 2ú
l ë n1 n 2 û

62
Atomic Structure

1 é1 1ù
- 2ú n = 6
-7 = 109737 ê 2
410.29 ´ 10 ë2 n2 û 2

Thus next line will be obtained during the jump of electron from 7th to 2nd shell, i.e.
1 é1 1ù é1 1 ù
= R ê 2 - 2 ú = 109737 ê - ú
l ë2 7 û ë 4 49 û
–7
l = 397.2 × 10 cm = 397.2 nm

Illustration 29. The wave number of 1St line of Balmer series of hydrogen spectrum is 15200 cm–1 . The wave
number of 1St line of Balmer series of Li+2 spectrum will be ?
Solution. Wave number of 1st line of Balmer series of hydrogen spectrum.
1 æ 1 1 ö
n= = RZ 2 çç 2 - 2 ÷÷
l è n1 n 2 ø
æ 1 1 ö
or n = Z 2 ´ R çç 2 - 2 ÷÷ for H, Z = 1
è n1 n 2 ø
æ 1 1 ö
n = R çç 2 - 2 ÷÷ = 15200 cm–1
è n1 n 2 ø
Wave number of 1st line of Balmer series of Li+2 ion is.
æ 1 1 ö
n = Z 2 ´ R çç 2 - 2 ÷÷ {Q Z = 3 for Li+2 }
è n1 n 2 ø
\ n = 32 × 15200 = 9 × 15200 = 136800 cm–1

Illustration 30. Calculate the ratio of maximum l of Lyman & Balmer series ?
1
Solution. E a nµ
l
Maximum l of Lyman series 1st line of Lyman series
=
Maximum l of Balmer series 1st line of Balmer series

1 é1 1ù é1 1 ù é3ù
Rê 2 - 2ú Rê - ú Rê ú
Lyman l L ë1 2 û = ë1 4 û ë4 û
= 1 Þ =
Balmer é1 1ù é1 1ù é5 ù
Rê 2 - 2ú Rê - ú Rê ú
lB ë2 3 û ë4 9û ë 36 û
lB 27 lL 5
= Þ =
lL 5 lB 27

Illustration 31. A certain electronic transition from an excited state to ground state of the Hydrogen atom in one or
more steps gives rise of 5 lines in the ultra violet region of the spectrum.How many lines does this
transition produce in the infra red region of the spectrum?
JPR\COMP.251\D\Allen(IIT-JEE Wing)\2020–21\Nurture\Che\Unit-01\Atomic Structure

Solution. (Lyman Series) ultra violet region : 5 Lines i.e. e– is coming from 6th to 1st Orbit
n2 –1 = 5 n2 = 6
Infrared region line
(i) Paschen series = (6 – 3) = 3
(ii) Bracket = (6 – 4) = 2
(iii) Pfund = (6 – 5) = 1
Total Number of lines are = 6

63
JEE-Chemistr y
l Limitation of the Bohr's model
(i) Bohr's theory does not explain the spectrum of multi electron atom.

nh
(ii) Why the Angular momentum of the revolving electron is equal to , has not been explained by Bohr's
2p
theory.
(iii) Bohr interrelate quantum theory of radiation and classical law of physics with out any theoritical
explanation.This was the biggest drawback of this model.
(iv) Bohr's theory does not explain the fine structure of the spectral lines. Fine structure of the spectral line is
obtained when spectrum is viewed by spectroscope of more resolution power.
(v) Bohr theory does not explain the spiliting of spectral lines in the presence of magnetic field (Zemman's
effect) or electric field (Stark's effect)

Bohr's Atomic model & Spectrum


1. Wavelength of radiations emitted when an electron jumps from a state A to C is 2000 Å and it is 6000 Å when
the electron jumps from state B to state C. Wavelength of the radiations emitted when an electron jumps from
state A to B will be
(A) 2000 Å (B) 3000 Å (C) 4000 Å (D) 6000 Å

2. Wave number of a spectral line for a given transition is x cm–1 for He+, then its value for Be3+ (isoelectronic of
He+) for same transition is :
x
(A) x cm–1 (B) 4x cm–1 (C) cm–1 (D) 2x cm–1
4

3. The ionization energy of a hydrogen like Bohr atom is 4 Rydberg. What is the radius of the first orbit of this
atom? (Bohr radius of hydrogen = 5 × 10-11 m ; 1 Rydberg = 2.2 × 10-18 J)

4. The energy required to remove an electron from metal X is E = 3.31 × 10–20 J. Calculate the maximum
wavelength of light that can photoeject an electron from metal X.
(A) 4 mm (B) 6 mm (C) 7 mm (D) 5 mm

5. An electron in a hydrogen like species is in the excited state n2. The wavelength for the transition from
n2 to n1 = 2 is 48.24 nm. The corresponding wavelength for the transition n2 to n1 = 3 is 142.46 nm. Find the
value of n2 and Z and report the H like atom.

6. The hydrogen atoms in the ground state is excited by means of monochromatic radiations of wavelength l Å.
The resulting spectrum consists of maximum 15 different lines. What is the value of l. RH = 109737 cm–1.

7. Monochromatic radiation of wavelength l is incident on a hydrogen sample in ground state. Hydrogen atoms
absorb a fraction of light and subsequently emit radiations of six different wavelengths. Find the wavelengthl
JPR\COMP.251\D\Allen(IIT-JEE Wing)\2020–21\Nurture\Che\Unit-01\Atomic Structure

(A) 97.2 nm (B) 121.6 nm (C) 110.3 nm (D) none of these

9.0 THE DUAL NATURE OF MATTER (THE WAVE NATURE OF ELECTRON)


SL AL

(i) In 1924. a French physicist, Louis De Broglie suggested that if the nature of light is both that of a
particle and of a wave, then this dual behavior should be true for the matter also.
(ii) According to De Broglie, the wavelength l of an electron is inversely proportional to its momentum p.
1 1
lµ or lµ
p mv

64
Atomic Structure

h
l= Here h = Planck's constant
p
p = momentum of electron
\ Momentum (p) = Mass (m) × Velocity (c)
h h
• l= =
mv 2m(K.E.)
From the de-Broglie equation it follows that wavelength of a particle decrease with increase in velocity of the
particle. Moreover, lighter particles would have longer wavelength than heavier particles, provided velocity is equal.
• If a charged particle Q is accelerated through potential difference V from rest then de-broglie wavelength
is
h
l=
2mQV
• de-Broglie concept is more significant for microscopic or sub-microscopic particles whose wavelength can
be measured.
• The circumference of the nth orbit is equal to n times the wavelength of the electron.
2prn = nl
• Wavelength of electron is always calculated using De-broglie calculation.

Illustration 32. Two particles X and Y are in motion. If the wavelength associated with particle X is 4 × 10–8 m,
calculate the wavelength associated with particle Y if its momentum is half of X.
Solution. According to de Broglie equation
h h
lx = and ly =
px py

l x py
=
l y px

But py = ½ px (given)
l x 1 / 2px
= =½
ly px

ly = 2lx = 2 × 4 × 10–8m = 8 × 10–8m

Illustration 33. Calculate the de Broglie wavelength of a ball of mass 0.1 kg moving with a speed of 30 ms–1.
h 6.6 ´ 10 -34
Solution. l= =
mv 0.1 ´ 30
l = 2.2 × 10–34 m
This is apparent that this wavelength is too small for ordinary observation.
Although the de Broglie equation is applicable to all material objects but it has significance only
in case of microscopic particles.
JPR\COMP.251\D\Allen(IIT-JEE Wing)\2020–21\Nurture\Che\Unit-01\Atomic Structure

Since, we come across macroscopic objects in our everyday life, de Broglie relationship has no
significance in everyday life.

Illustration 34. Which of the following should be the wavelength of an electron if its mass is 9.1× 10–31 kg and its
velocity is 1/10 of that of light and the value of h is 6.6252× 10–34 joule second?
(1) 2.446 × 10–7 metre (2) 2.246 × 10–9 metre (3) 2.246 × 10–11 metre
(4) 2.246 × 10–13 metre
Solution. Given that
m = 9.1 × 10–31 kg
1
c= of velocity of light
10

65
JEE-Chemistr y

1
or c= × 3 × 108 metre/second i.e. 3 × 107 metre/second
10
h = 6.6252 × 10–34 joule second
h 6.6252 ´ 10 -34 6.6252 ´ 10 -34
l= = =
mc 9.1 ´ 10 -31 ´ 3 ´ 107 27.3 ´ 10 -24
–10
or 0.2426 × 10 metre
or 2.426 × 10–11 metre

10.0 HEISENBERG UNCERTAINTY PRINCIPLE


SL AL
• Bohr's theory considers an electron as a material particle. Its position and momentum can be determined
with accuracy. But, when an electron is considered in the form of wave as suggested by de-Broglie, it is
not possible to ascertain simultaneously the exact position and velocity of the electron more precisely at
a given instant since the wave is extending throughout a region of space.
• In 1927, Werner Heisenberg presented a principle known as Heisenberg uncertainty principle which states
as : "It is impossible to measure simultaneously the exact position and exact momentum of a body as
small as an electron."
• The uncertainty of measurement of position, Dx, and the uncertainty of momentum Dp or mDv, are
related by Heisenberg's relationship as : ( p = mv, Dp = mDv)
h h h
Dx . Dp > or Dx . mDv > or Dx.Dv ³
4p 4p 4 pm
where h is Planck's constant.
Dx Dv = uncertainty product
For an electron of mass m (9.10 × 10–28 g), the product of uncertainty is quite large.
6.624 ´ 10 -27
Dx . Dv >
4 pm
6.624 ´ 10 -27
> = 0.57 erg sec per gram approximately
4 ´ 3.14 ´ 9.10 ´ 10 -28
When Dx = 0, Dv = ¥ and vice-versa.
In the case of bigger particles (having considerable mass), the value of uncertainty product is negligible. If the
position is known quite accurately, i.e., Dx is very small, Dv becomes large and vice-versa.
• In terms of uncertainty in energy DE, and uncertainty in time Dt, this principle is written as,
h
DE.Dt ³
4p
• Heisenberg replaced the concept of definite orbits by the concept of probability.

Illustration 35. Why electron cannot exist inside the nucleus according to Heisenberg's uncertainty principle ?
Solution. Diameter of the atomic nucleus is of the order of 10–15 m
The maximum uncertainty in the position of electron is 10–15 m.
JPR\COMP.251\D\Allen(IIT-JEE Wing)\2020–21\Nurture\Che\Unit-01\Atomic Structure

Mass of electron = 9.1 × 10–31 kg.


h
Dx. Dp =
4p
Dx × (m.Dv) = h/4p

h 1 6.63 ´ 10 -34 1
Dv = ´ = ×
4 p Dx.m 4´
22 10 ´ 9.1 ´ 10 -31
-15

7
10
Dv = 5.80 × 10 ms –1

This value is much higher than the velocity of light and hence not possible.

66
Atomic Structure
Illustration 36. What should be the uncertainty in the velocity of an electron if the uncertainty in its position is 0.005
nm, the mass of electron is 9.109 × 10–31 kg and the value of h is 6.6252 × 10–34 joule/second?
(1) 2.316 × 105 (2) 1.158 × 107 (3) 2.316 × 109 (4) 2.316 × 1011
Solution. Uncertainty in position (Dx) = 0.005 nm = 0.005 × 10–9 m = 5 × 10–12 m
Mass of electron (m) = 9.109 × 10–31 kg.

h 6.6252 ´ 10 -34
\ Dv = = m/sec.
4 pm ´ Dx 4 ´ 3.14 ´ 5 ´ 10 -12 ´ 9.109 ´ 10 -31

6.6252 ´ 10 -34
or Dv =
4 ´ 3.14 ´ 5 ´ 9.109 ´ 10 -43
\ Dv = 1.15816 × 107 m/sec.
Illustration 37. If uncertainty in position of an e– is same as the Dx of He atom. If Dp of e– is 32 ×105 then find Dp
in He atom.
h
Solution. Dx × Dp =
4p
Since Dx is same for both.
therefore Dp will be same by
h
Dx e ´ Dpe ³
4p DPe
h = DPHe = 1
Dx(He) ´ DPHe ³
4p
\ DPe = DPHe
32 × 105 = 32 × 105
D PHe = 32 × 105
Illustration 38. Calculate the uncertainty in the position of a particle when the uncertainty in momentum is
(a) 1 × 10–3g cm sec–1 (b) Zero.
Solution. Given
Dp = 1 × 10–3 g cm sec–1
h = 6.62 × 10–27 erg sec.
p = 3.142
According to uncertainty principle

h h 1 6.62 ´ 10 -27 1
Dx.Dp ³ So, Dx ³
. ³ ´ -3 = 0.527 × 10–24 cm
4p 4 p Dp 4 ´ 3.142 10
(b) When the value of Dp = 0, the value of Dx will be infinity.

De-Broglie and Heisenberg uncertainty principle


1. Alveoli are the tiny sacs of air in the lungs whose average diameter is 50 pm. Consider an oxygen molecule
trapped within a sac. Calculate uncertainty in the velocity of oxygen molecule?
(A) 1.98 × 10–2 ms–1 (B) 19.8 ms–1 (C) 198 × 10–4 ms–1 (D) 19.8 × 10–6ms–1
2. Determine the de-Broglie wavelength associated with an electron in the 3rd Bohr's orbit of He+ ion?
JPR\COMP.251\D\Allen(IIT-JEE Wing)\2020–21\Nurture\Che\Unit-01\Atomic Structure

(A) 10 Å (B) 2 A (C) 5 Å (D) 1 Å


3. If E1, E2 and E3 represent respectively the kinetic energies of an electron, an alpha particle and a proton each
having same de Broglie wavelength then:
(A) E1 > E3 > E2 (B) E2 > E3 > E1 (C) E1 > E2 > E3 (D) E1 = E2 = E3
4. Which of the following statement is true referred to the above paragraph :
I. Higher is the mass of the particle, error in measurement of velocity reduces.
II. For an electron if uncertainly in position tends to zero then uncertainly in momentum is extremely small.
III. If l1 > l2 are the two different wavelength used to detect the position of electron and uncertainty in
velocity be v1 and v2 respectively then v2 > v1
(A) only I (B) I and II (C) I,II & III (D) I and III

67
JEE-Chemistr y
5. The uncertainty in position and velocity of the particle are 0.1 nm and 5.27×10–24 ms–1 respectively, then the
mass of the particle is : (h = 6.625 × 10–34Js)
(A) 200 g (B) 300 g (C) 100 g (D) 1000 g
6. In H-atom, if ‘x’ is the radius of the first Bohr orbit, de Broglie wavelength of an electron in 3rd orbit is :
9x x
(A) 3 p x (B) 6 p x (C) (D)
2 2
7. Which of the following has least De Broglie l if they have same velocity.
(A) e– (B) p (C) CO2 (D) SO2
8. What should be the momentum (in gram cm per second) of a particle if its De Broglie wavelength is 1 Å and the
value of h is 6.6252 × 10–27 erg second ?
(A) 6.6252 × 10–19 (B) 6.6252 × 10–21 (C) 6.6252 × 10–24 (D) 6.6252 × 10––27
9. What should be the uncertainty in position if uncertainty in momentum is 1 × 10–2 g cm/sec. and value of h is
6.6252 × 10–34 Joule sec. ?
(A) 1.054 × 10–22 m (B) 1.054 × 10–25 m (C) 0.525 × 10–27 m (D) 1.054 × 10–32 m

11.0 WAVE MECHANICAL MODEL OF ATOM


11.1 Concept of Orbitals
SL AL

According to Heisenberg's uncertainty principle it is not possible to determine precisely the position and
momentum of an electron in the atom simultaneously. Therefore , Bohr's concept of well defined orbits is
ruled out. In quantum mechanical model, we speak of probability or possibility of an electron with a particular
energy being present in a certain region of space around the nucleus. The probability of finding the electron at
a particular location is given by the square of wave function (|y|2) corresponding to that location. There are
certain regions around the nucleus where probability of finding the electron is high and there are certain
regions where probability of finding the electron is low. The probability of finding the electron does not become
zero even at large distances from the nucleus, although it may become negligible. Therefore, is not possible to
draw a boundary that will enclose the region of 100 % probability.
However, for the sake of simplicity we draw arbitrary boundaries which enclose the regions where probability of
finding the electron is maximum (about 90-95%). These regions of space around the nucleus where probability
of finding the electron is maximum are called orbitals.
An orbital may be defined as that region of space around the nucleus where the probility of finding
an electron is maximum (90-95%).
It is rather difficult to represent an orbital by a simple picture. Different methods are employed for this purpose.
But the most common method is to represent an orbital as an electron cloud in terms of small dots, the intensity
of dots being more in some region and less in other.

Orbital Orbit
Electron-cloud representation Bohr's orbit
JPR\COMP.251\D\Allen(IIT-JEE Wing)\2020–21\Nurture\Che\Unit-01\Atomic Structure

of an orbital
In terms of electron cloud representation, the probability of finding an electron in a particular region of space is
directly proportional to the density of dots in that region.
The important differences between an orbit and an orbital are given in table.

68
Atomic Structure
Differences between an Orbit and an Orbital :
Orbit Orbital
1. It is a well-defined circular path 1. It is a region of space around the nucleus
followed by revolving electron of the atom where the electron is most
around the nucleus. likely to be found.
2. It represents planar motion of an 2. It represents three-dimensional motion of
electron. an electron around the nucleus.
3. The maximum number of electrons in 3. An orbital cannot accommodate more than
an orbit is 2n2 , where n stands for two electrons.
4. Orbits are circular in shape. 4. Orbitals have different shapes, e.g., s-orbitals
are spherically symmetrical whereas
p-orbitals are dumb-bell shaped.
5. Orbits are non-directional in 5. Orbitals (expects s-orbitals) have directional
character hence they cannot character and hence they can account for
explain shapes of molecules. shapes of molecules.
6. Concept of well-defined orbit is 6. Concept of orbital is in accordance with
against Heisenberg's principle. Heisenberg's principle

11.2 The wave function


SL AL
Each "Particle" is represented by a wave function Y(position, time) such that Y*Y = the probability of finding
the particle at that position at that time.
The wavefunction is used in the Schrodinger equation. The Schrodinger equation plays the role of Newtons' laws
and conservation of energy in classical mechanics - i.e., it predicts the future behaviour of a dynamic system. It
does not predict analytically and precisely detailed outcome of an experiment, but given a large number of
events, the Schrodinger equation will predict the distribution of results.
• Contains all the measurable information about the particle.
• Y*Y summed over all space = 1,
(If particle exists, probability of finding it somewhere must be one)
• It is continuous.
• Allows energy calculations via the Schrodinger equation.
• Establishes the probability distribution in three dimensions.
• Permits calculation of most probable value (expectation value) of a given variable.

11.3 Schrodinger Equation in Cartesien Coordinates


SL AL

æ ¶ 2Y ¶ 2Y ¶ 2Y ö 8 p2m
ç 2 + 2 + 2 ÷+ (E - V)Y = 0 Three dimension Schrodinger equation
è ¶x ¶y ¶z ø h2

¶ 2y 8 p 2 m
+ (E - V) y = 0 One dimension Schrodinger equation
¶x 2 h2
Y = Wave function = f(x, y, z)
E = Total energy of particle
JPR\COMP.251\D\Allen(IIT-JEE Wing)\2020–21\Nurture\Che\Unit-01\Atomic Structure

V = Potential energy of particle.


• The Schrodinger equation is a second order differential equation. It has infinite solutions.
• Out of infinite solutions only those solution are acceptable in which Y is continuous, finite and single
valued.
• When solved with certain restriction , called boundary condition - only finite number of solutions are
obtained.
• The acceptable solution corresponds to different states of electron in an atom.
Significance of y
(i) y gives the complete description of state of system (like electron in an atom)
(ii) y represents probability amplitude of particle wave.
(iii) As such y has no physical significance.
(iv) In an atom, each y represent an orbital like y1s , y2s , y2p , etc.
69
JEE-Chemistr y
Significance of y2
The y2 gives probability density of finding a particle at a point in space.
For electromagnetic waves intensity of light (I) is proportional to square of amplitude.
I µ A 2 µ no. of photons
Similarity y2 µ probability of finding a particle in space.
This interpretation was given by Born hence known as Born interpretation.
Probability of finding a particle in space in infinitesimally small volume element dv = dx dy dz, situated at
coordinate (x, y, z) is :
Probability = [Y(x, y, z)]2 (dx dy dz)
Probability density at a point (x,y,z) in space = [ y(x, y, z) ]
2

11.4 Schrodinger Equation in Polar Coordinates


SL AL
It is difficult to solve the schrodinger equation for H-atom in cartesian coordinates. Hence the equation is
transformed from cartesian to polar coordinates.
z = rcosq
y = rsinq sinf
x = rsinq cosf

q
y
f
x

y (x , y, z) = R(r).Q(q) F(f)
The differential equation is transformed into polar coordinates by proper substitution, the resulting differential
equation in polar coordinates is broken into three independent equations.
The three independent equations are :
(a) Radial equation,
(b) Zenith equations,
(c) Azimuthal equation
The three equations are solved separately. The solution to radial equation which contain only 'r' terms gives radial
function R. The solution to angular equation which contain only q term give angular wave function Q. The solution
to azimuthal equation to give azimuthal function F. These three independent solution are combined to give complete
wave function.
y = r.q.f
Probability density at a point (r,q,f) in space = [ R(r)Q( q).F (f) ]
2

Probability of finding an electron at coordinate (r,q,f) in infinitesimally small volume element dv = r2 sinq dq df
dr is :
Probability = [ R(r) Q(q) F( f) ] .r . sin q.dq df dr
2 2
JPR\COMP.251\D\Allen(IIT-JEE Wing)\2020–21\Nurture\Che\Unit-01\Atomic Structure

Radial probability density is probability density at distance r in a specified direction. Radial probability
density at a constant angle is = [R(r)]2.
Radial probability in a volume 4pr2.dr at distance 'r' from nucleus = [R(r)]2 . 4pr2.dr
Radial probability distribution [P(r)] = [R(r)]2 4pr2
Radial probability distribution gives spherical sense of probability without any specified direction.

11.5 Quantum Numbers


SL AL

In an atom a large number of electron orbitals are permissible. These orbitals are designated by a set of numbers
known as quantum numbers. In order to specify energy, size, shape and orientation of the electron orbital three
quantum numbers are required. These are, principal quantum number, azimuthal quantum number and magnetic
quantum number. These quantum numbers arise as a natural consequence during the solution of the Schrodinger

70
Atomic Structure
wave equation. In order to designate the electron, an additional quantum number called spin quantum number
is needed to specify spin of the electron. Thus, each orbital in an atom is designated by a set of three quantum
numbers and each electron is designated by a set of four quantum numbers.
These quantum numbers are discussed below :
(I) Principal quantum number (n) ® Shell (Orbit) (II) Azimuthal quantum number (l) ® Sub shell
(III) Magnetic quantum number (m) ® Orbital (IV) Spin quantum number (S) ® Spin of e–

Principal quantum number : Shell


l Given by Bohr.
l It represents the size and energy of the shell to which e– belongs
l It is noted that, theoretically value of n may be from 1 to ¥ , but only value of n used for known elements
is from 1 to 7.
l Greater the value of n greater is the distance from the nucleus.

n2
r = 0.529 × A°
z
r1 < r2 < r3 < r4 < r5 – – – – – – – –
l Greater the value of n greater is the energy of shell

z2
E = – 13.6 × ev/atom
n2
E1 < E2 < E3 < E4 – – – – – – – –
nh
l The angular momentum of a revolving electron is mvr =
2p
Where n = Principal quantum number.
l Max. number of electrons in a particular shell is equal to 2n2
Shell K L M N
Principal quantum number (n) 1 2 3 4
Maximum number of electrons 2 8 18 32
Azimuthal quantum number / Angular quantum number / Secondary quantum number / Subsidiary
quantum number :
Represented by 'l' (Given by – Sommerfeld)
l It represents the shape of the subshell and orbital.
l Value of l between 0 to (n – 1)
i.e. l = 0, 1, 2 – – – – – – – – – (n–1)
l = 0(s Subshell)
l = 1(p Subshell)
l = 2(d Subshell)
l = 3(f Subshell)
Ex. If n = 1 then l = 0 Þ 1s i.e. in n =1 shell, only one subshell 's' is present.
If n = 2 then l = 0,1 Þ 2s,2p i.e. in n =2 shell, two subshell 's' & 'p' are present.
If n = 3 then l = 0,1,2 Þ3s, 3p, 3d i.e. in n =3 shell, three subshell 's' , 'p' & 'd' are present.
JPR\COMP.251\D\Allen(IIT-JEE Wing)\2020–21\Nurture\Che\Unit-01\Atomic Structure

If n = 4 then l = 0,1,2,3 Þ 4s,4p, 4d, 4f i.e. in n =2 shell, four subshell 's' , 'p' , 'd' & 'f' are present.
l If the value of n is same then the order of energy of the various subshell will be s < p < d < f
Ex. 4s < 4p < 4d < 4 f
3s < 3p< 3d
2s < 2p
l If value of l is same but value of n is different then the order of energy will be -
1s < 2s < 3s < 4s < 5s < 6s
3d < 4d < 5d < 6d
4P < 5P <6p
l It gives the quantized values of orbital angular momentum of electron and it is denoted by L and expression
for L is as

71
JEE-Chemistr y

h ì hü
L= l(l + 1)
2p
= l(l + 1) h where íQ h = ý { h is called as 'hash'}
î 2p þ
Orbital angular momentum
For s subshell = 0
h
For p Subshell = 2 or 2h
2p
l The number of electrons in a particular subshell is equal to 2(2l +1)
for s Subshell = 2 e– for p subshell = 6 e–

for d subshell = 10 e for f subshell = 14 e–
Shape of the subshell :
s ® spherical p ® dumb bell shape
d ® double dumb bell shape f ® complex shape
Magnetic quantum number /Orientation quantum number (m) :
It was introduced by Zeeman to explain splitting of lines in magnetic field. Magnetic quantum number determines
orientation of orbitals in space and it is denoted by m and m is dependent upon the value of l and it has only
integral values from +l to –l through zero.
Each m represent one orbital within the sub energy level. It defines orientation of orbital in space.
Total value of m = 2l + 1 = total number of orbitals in a subshell
l for s subshell, l = 0, m = 0 (only one orientation of 'S' orbital is possible)
l for p subshell, l = 1, m = 2 × 1 + 1 = 3, i.e, – 1 to +1
i.e., –1, 0, +1 (three orientation of 'p' orbitals are possible can be designate as px, py, pz)
l for d subshell, l = 2, m = 2 × 2 + 1 = 5, (i.e, only 5 orbitals)
= –2 to +2 = –2, –1, 0, +1, +2
l for f subshell, l = 3, m = 2 × 3 + 1 = 7, (i.e, only 7 orbitals)
= –3 to +3
= –3, –2, –1, 0, +1, +2, +3
i.e., s subshell p subshell d subshell f subshell

0 –1 0 +1 –2 –1 0 +1 +2 –3 –2 –1 0 +1 +2 +3
Spin Quantam number (s) : (Given by Gold schmidt) :
It represents the direction of electron spin around its own axis
ì 1ü
l for clock wise spin/spin up (­) electron ® í± ý
î 2þ

ì 1ü
l for anticlock wise spin/spin down (¯) electron ® ím ý
î 2þ
h
Spin angular momentum of an e –– = s ( s + 1).
or s ( s + 1) h
2p
JPR\COMP.251\D\Allen(IIT-JEE Wing)\2020–21\Nurture\Che\Unit-01\Atomic Structure

l Each orbital can accomodate 2 electrons with opposite spin or spin paired.
S

­¯ Spin paired e

­­ Spin parallel e

Illustration 39. How many sub-shells are there in N shell ? How many orbitals are there in d sub-shell ?
Solution For N shell, principal quantum number, n = 4.
Number of sub-shells, in an energy level = n
\ Number of sub-shells, in N shell = 4

72
Atomic Structure
For d subshell l = 2
Number of orbitals in a sub-shell = 2 l + 1
Number of orbitals in a d-sub-shell = 2 × 2 + 1 = 5

Illustration 40. State which of the following sets of quantum numbers would be possible and which would not be
permissible for an electron in an atom.
1 1
(i) n = 3 , l = 3 , ml = +3 , ms = + (ii) n = 3 , l = 0 , ml = +1 , ms = –
2 2
1
(iii) n = 5 , l = 4 , ml = +3 , ms = 1 (iv) n = 0 , l = 0 , ml = 0 , ms = +
2
1
(v) n = 4 , l = 3 , ml = –2 , ms = –
2
Solution
(i) Not possible because when n = 3, l cannot be equal to 3. The value of l cannot be numerically equal to the
value of n.
(ii) Not possible because when l = 0, ml cannot be +1. The value of ml cannot be numerically more than the
value of l.
1 1
(iii) Not possible because s cannot be 1. The only allowed values of ms are + and – .
2 2
(iv) Not possible because n cannot be 0.
(v) This set is permitted because when n = 4, l can be 3. When l = 3, ml can be –2. ms can have
1
– value.
2

Illustration 41.
What is the maximum number of electrons that can be accommodated :
(i) in the shell with n = 3
(ii) in the subshell with l = 3
(iii) in an orbital with ml = +3
Solution
(i) For a shell with n = 3 the number of orbitals = n2 = 32 = 9
\ Number of electrons = 9 × 2 = 18
(ii) Number of orbitals = 2l +1 = 2 × 3 + 1 = 7
\ Number of electrons = 7 × 2 = 14
(iii) This corresponds to only one orbital and hence it can accomodate only 2 electrons.

Illustration 42.
Which of the following orbitals are not possible ?
2d, 4f, 4g and 6d
Solution
2d is not possible. For d sub-shell l = 2. When n = 2, l can be 0 and 1. Since l cannot be 2 for n = 2 hence
2d sub-shell is not possible.
JPR\COMP.251\D\Allen(IIT-JEE Wing)\2020–21\Nurture\Che\Unit-01\Atomic Structure

4f is possible. For f-sub-shell l = 3. When n = 4, l can be equal to 3.


4g is not possible. For g-sub-shell l = 4. When n = 4, l cannot be equal to 4.
6d is possible. For d-sub-shell l = 2. When n = 6, l can be equal to 2.

Illustration 43. What are the possible values of ml for an electron with l = 2 ?
Solution The value of ml can vary from –l to +l ; in this case ml can have values –2, –1, 0, +1
and +2.

Illustration 44. What are the possible values of ms for an electron with ml = 0 ?
Solution ms must be –1/2 or +1/2. These are the only permitted values, regardless of the values of the other
quantum numbers.

73
JEE-Chemistr y
Illustration 45. Match each of the following quantum number values with the proper letter designation
(K , L , M , N , s , p , d , f) :
(a) n = 1 (b) l = 2 (c) l = 0 (d) n = 3
Solution (a) K (b) d (c) s (d) M

Quantum Numbers
1. Which d -orbital does not have four lobes?
(A) d x 2 – y 2 (B) dxy (C) dyz (D) d z 2

2 The total number of subshells in nth main energy level are :


(A) n2 (B) 2n2 (C) (n–1) (D) n.
3. Which of the following orbital does not make sense?
(A) 3d (B) 3f (C) 5p (D) 7s.
4. The maximum number of electrons in s, p and d-subshells are :
(A) 2 in each (B) 2, 6 and 6 (C) 2, 6 and 10 (D) 2, 6 and 12.
5. Any p-orbital can accommodate up to :
(A) four electrons (B) two electrons with parallel spin
(C) six electrons (D) two electrons with opposite spin.
6. Which of the following sets of quantum numbers represent an impossible arrangement
n l m ms n l m ms
1 1
(A) 3 2 –2 (B) 4 0 0
2 2
1 1
(C) 3 2 –3 (D) 5 3 0
2 2
7. Which quantum number will determine the shape of the subshell
(A) Principal quantum number (B) Azimuthal quantum number
(C) Magnetic quantum number (D) Spin quantum number

11.6 Radial probability distribution function (rpdf), 4p r2y 2(r)


SL AL
It is often useful to know the likelihood of finding the electron in an orbital at any given distance away from the
nucleus. This enables us to say at what distance from the nucleus the electron is most likely to be found, and
also how tightly or loosely the electron is bound in a particular atom. This is expressed by the radial distribution
function. For s-orbitals, the radial distribution function is given by multiplying the electron density by
4p r2y 2(r). Radial distribution function is the measure of the probability of finding the electron in a spherical
shell between thickness r and (r + dr) from the nucleus, irrespective of the direction. For 1s orbital of H-atom :
The wave function y1,is independent of the angles q and f , and the desired probability is found by multiplying
|y2| by volume of the thin shell.

v Volume of radial shell :


JPR\COMP.251\D\Allen(IIT-JEE Wing)\2020–21\Nurture\Che\Unit-01\Atomic Structure

é Volume of sphere ù é Volume of sphere ù


= ê ú-ê ú r
ë with radius(r + dr) û ë with radius r û
4 4 r+dr
= p(r + dr)3 - pr 3
3 3
4 3 4 4
= p(r + 3r 2dr + 3rdr 2 + dr 3 ) - pr 3 = p[r 3 + 3r 2dr - r 3 ]
3 3 3
(As dr represents an extremely small thickness, the higher powers of dr such as dr2 and dr3 may be neglected.)
4
\ Volume of shell = p ´ 3r 2dr) = 4 pr 2dr
3

74
Atomic Structure
Radial probability distribution = Radial probability density × Volume of radial shell
= 4p r2y 2(r)dr
If we observe the radial probability distribution curve of 1s orbital, we find that it is quite different from radial
probability density graph (y2 vs. r). The radial probability density (y2) is maximum close to the nucleus but the
total probability is least corresponding to this volume. This is due to the reason that the volume of the shell near
the nucleus is very small resulting in a small value of total probability. It is zero at the nucleus. As the distance
from the nucleus increases the volume of the radial shell increases while y2 decreases rapidly.
As a result of this, the probability keeps on increasing gradually and reaches a maximum value and then
decreases gradually. The maximum in the curve indicates the most probable value and the corresponding
distance from the nucleus is called distance of maximum probability (r0). For hydrogen atom, this radius has a
value of 53 pm.It is important to note that Bohr predicted that the electron will always be at r = 53 pm for
hydrogen atom. However, according to wave mechanical model the electron is most likely to be found at this
distance and there is probability of finding the electron at distances both shorter and larger than 53 pm.

v Chracteristics of radial distribution function :


(i) The number of maxima in radial distribution function plot are (n – 1).
(ii) The maximum probability of finding the electron, for the ground state hydrogen atom (1s) is found to be at
a0 (first Bohr radius).
(iii) For 2s, 3s, 3p orbitals , the number of maxima is more than one, indicating that there is maximum
probability of finding the electron at the distance corresponding to the highest value of peak.
However, there is lesser probability of finding the electron at the other peaks. It shows that in a certain
state, the electron spends some portion of its time very close to the nucleus.

Ex. Find the distance at which probability of finding electron is maximum for 1s orbital in a He atom. The
2r
-
4 a0
wave function orbital given as. y 1s = .e
a 03 / 2
æ 4r ö -4r
- ÷
æ 16 ö çè a0 ø
Sol. Probability distribution function is P(r) = y .4pr = ç 3 ÷ e
2 2 .4 pr 2 Þ 2
P(r) = k . r . e a0
è a0 ø
4r 4r
dP(r) - æ 4 ö 2 - a0 2r a
differentating = 2r.e a0 – ç ÷ r .e =0 Þ 1 = Þr= 0
dr è a0 ø a 0 2

a0
Þ probability of finding electron is maximum at distance from nucleus.
2
JPR\COMP.251\D\Allen(IIT-JEE Wing)\2020–21\Nurture\Che\Unit-01\Atomic Structure

75
JEE-Chemistr y
v Electron-density distribution in 1s, 2s and 3s-orbitals
The lower part of the fig. shows how the electron density, represented by y2 varies as a function of distance from
the nucleus. In the 2s and 3s - orbitals, the electron-density function drops to zero at certain distances from the
nucleus. The spherical surfaces around the nucleus at which y2 is zero are called nodes.

1s 2s 3s
n=1, l=0 n=2, l=0 n=3, l=0

y(r) y(r) y(r)

1s 2s 3s
r r r

Height of graph Node Node


y (r) indicates density
y (r) y (r)
2 2 2

of dots as we
move away from
origin
1s 2s 3s
r r r

1s 2s 3s
JPR\COMP.251\D\Allen(IIT-JEE Wing)\2020–21\Nurture\Che\Unit-01\Atomic Structure
4p r2 y2(r)

4p r2 y2(r)

4p r2 y2(r)

r r r
rmax rmax rmax

76
Atomic Structure
v Electron-density distribution in 2px , 3px and 4px-orbitals :

2px 3px 4px

y(r) y(r) y(r)

r r r

y (r)
2
y (r)
2
y2(r)

r r r
4pr y (r)

4pr y (r)

4pr2y2(r)
2

2
2

r r r
rmax rmax rmax
JPR\COMP.251\D\Allen(IIT-JEE Wing)\2020–21\Nurture\Che\Unit-01\Atomic Structure

77
JEE-Chemistr y

2 8

[(R)24pr2]
Rnl(r)
1 4
1s 1s
0 0 1a
5 10 15 20a0 0 10 20 24a0
0.8 3
0.4 2
2s 1 2s
0 0

0.2
3
0.1 2
2p 2p
0 1
0

0.3 2.0
0.2 1.5
0.1 3s 1.0 3s
0 0.5
0

0.1 1.5
1.0
0.05 0.5
3p 3p
0 0

0.06 1.5
1.0
0.03 3d 3d
0.5
0 0

0.2 0.6
0.1 0.4
4s 0.2 4s
0 0
0.06 1.0
0.03 4p 0.5 4p
0 0

0.03 1.0
0.02 0.5
0.01 4d 4d
0 0
JPR\COMP.251\D\Allen(IIT-JEE Wing)\2020–21\Nurture\Che\Unit-01\Atomic Structure

0.02 1.0
[(R) 4pr ]
2
Rnl(r)

0.01 0.5
4f
2

0 0 4f
0 5 10 15 20a0 5 10 15 20 24a0

Fig. Radial function Rnl(r) and radial probability distribution [(R)24pr2] for the hydrogen like atom in the 1s
state & thereby show that the most probable radial position, i.e., the radius with the highest radial probability
density, is a0/Z.

78
Atomic Structure
11.7 Angular Probability distrubtion function
The angular probability gives an idea of shape and orientation of the orbitals.
v Shapes of s-orbitals
The s-orbitals are spherically symmetrical about the nucleus, i.e., the probability of finding s electron is same in
all directions from the nuclesus. The size of the orbital depends on the value of principal quantum number, there
is one spherically symmetrical orbital. The 1s orbital is smaller than 2s-orbital and 2s-orbital is smaller than 3s,
but all are spherical in shape as shown in fig.

Node

1s 2s Node 3s
Although the s-orbitals belonging to different shells are spherically symmetrical, yet they differ in
certain respects as explained below :
(i) The probability of 1s electron is found to be maximum near the nucleus and decreases as the distance from
the nucleus increases. In case of 2s electrons, the probability is again maximum near the nucleus and then
decreases to zero and increases again and then decreases as the distance from the nucleus from the nucleus
increases. The intermediate region (a spherical shell) where the probability is zero is called a nodal surface
or simply node. Thus, 2s-orbital differs from 1s-orbital in having one node within it. Similarly, 3s has two
nodes. in general, any ns orbital has (n -1) nodes.

(ii) The size and energy of the s-orbital increases as the principal quantum number inreases, i.e., the size and
energy of s-orbital increases in the order 1s < 2s < 3s ....

The s orbital of higher energy levels are also symmetrically spherical and can be represented as
follows :
y
Nodal
suface

1s
z 2s
3s
v Shape of p-orbitals
There are three p-orbitals, commonly referred to as px, py and pz. These three p-orbitals, possess equivalent
energy and therefore, have same relation with the nucleus. They, however, differ in their direction & distribution
of the charge. These three p-orbitals are situated at right angle to another and are directed along x, y and z-axes
(figure)
JPR\COMP.251\D\Allen(IIT-JEE Wing)\2020–21\Nurture\Che\Unit-01\Atomic Structure

z z z
+


y y
+

y –
+
x x 2py x
– 2pz

2px
79
JEE-Chemistr y
l Each p orbital has dumb bell shape (2 lobes which are separated from each other by a point of zero probability
called nodal point or nucleus).
l The two lobes of each orbital are separated by a plane of zero electron density called nodal plane.
l Each p orbital of higher energy level are also dumb bell shape but they have nodal surface.
Nodal y y
surface
xz plane
x (Nodal plane)
2Px x
z 3Px
4Px z
Pxy
Nodal point

(Nodal plane)
y yz plane y

xy plane
(Nodal plane)

x x
z Px z
Pxz

v Shape of d-orbitals
When l = 2, 'm' has five values – 2, –1, 0, +1, +2. It implies that d subshell of any energy shell has five orbitals.

All the five orbitals are not identical in shape. Four of the d orbitals dxy, dyz, dxz, d x2 - y2 contain four lobes while

fifth orbital d z2 consists of only two lobes. The lobes dxy orbital lie between x and y axis. Similar is the case for

dyz and dxz. Four lobes of d x2 - y2 orbital are lying along x and y axis while the two lobes of d z2 orbital are lying
along z axis and contain a ring of negative charge surrounding the nucleus in xy plane.
m = –2, –1, 0, +1, +2

( )
Geometry of these d xy ,d yz ,d zx ,d z 2 ,d x 2 - y 2 orbits ® double dumb bell
z z

+
+


– y y

+

x x
+
3dxy
JPR\COMP.251\D\Allen(IIT-JEE Wing)\2020–21\Nurture\Che\Unit-01\Atomic Structure

3dz2 z
z z


+ –
+
+
y – y y

+
+
x x – +
x

3dyz
3dzx 3dx2 – y2

80
Atomic Structure
v In d orbital
(i) Nodal Point ® 1
(ii) Nodal surfaces (Radial nodes) :
3 dxy ® 0 Nodal surface 4 dxy ® 1 Nodal surface
5 dxy ® 2 Nodal surfaces n dxy ® ( n – 3 ) Nodal surfaces
(iii) Nodal plane (Angular nodes) :
dxy ® xz & yz nodal plane dxz ® xy & zy nodal plane
dzy ® zx & yx nodal plane d x 2 -y2 ® 2, nodal planes at 45° angle to X-axis.

d z2 ® 2 conical shape nodes

F Note :
Orbitals of d subshell are equivalent in energy d z 2 does not have any nodal plane.

v f-subshell
When l = 3 (f subshell)
Then m = –3, –2, –1 , 0 , +1 , +2 , +3
l The structure of f-orbital is very complex
l f-subshell show that f subshell has 7 orbitals which are equivalent in Energy.

v Representation of the orbitals


s subshell ® s orbital p subshell ® px py pz orbital

d subshell® d xy d yz d z2 d xz d x 2 - y2 orbital

f subshell ® fx3 fy3 fz3 fxyz fx( y2 -z2 ) fy(z2 - x2 ) fz( x2 -y2 ) orbital

Illustration 46. The radial wave equation for a certain e– wave in hydrogen atom is:
3/ 2
1 æ 1 ö
y= ç ÷ [(x - 1)(x 2 - 8x + 12)]e - r / 2
16 4 è a 0 ø
where x = 2r/a0 ; a0 = radius of first Bohr orbit. At node value of x is
(A) 1, 3 (B) 1, 2, 6 (C) 1, 3, 6 (D) 1, 5
Ans. (B)
Solution For node (x – 1) (x2 – 8x + 12) = 0
\ x = 1, 2, 6

Illustration 47. The wave functions of 3s and 3pz orbitals are given by
1/ 2 3/ 2
1 æ 1 ö æ zö æ 4zr 4 z 2 r 2 ö - zr/3a0
ψ 3s = ç ÷ çè a ÷ø çè 6 - + e
a 0 9 a 02 ÷ø
JPR\COMP.251\D\Allen(IIT-JEE Wing)\2020–21\Nurture\Che\Unit-01\Atomic Structure

9 3 è 4π ø 0

1/ 2 3/2
1 æ 3 ö æ zö æ 2zr ö æ 2zr ö - zr/3a0
y 3pZ = ç ÷
9 6 è 4π ø çè a ÷ø çè 4 - 3a ÷ø çè 3a ÷ø e cos q
0 0 0

from these choose the incorrect statement.


(A) Total number of nodes for 3pz & 3s orbitals are equal
(B) The angular nodal surface of 3pz orbital has the equation θ = π /2
(C) The radial nodal surfaces of 3s orbital and 3pz orbitals are at equal distance from the nucleus
(D) 3s electron have greater penetrating power into the nucleus in comparison to 3p, electrons
Ans. (C)

81
JEE-Chemistr y

æ 4Zr 4 Z 2 r 2 ö
Solution. 3s orbital has two radial node at the values of radius given by solutions of ç 6 - + . =0
è a 0 9 a 02 ÷ø

æ 4Zr ö
3pz orbital has one radial nodal surface at ç 4 - = 0 & one angular node at θ = π /2
è 3a 0 ÷ø
for 3 pz , at r = 0 ψ = 0 while for
3s at r = 0 ψ = maximum so, 3s has greater penetrating power than 3pz orbital

Illustration 48. The distance of spherical nodes from nucleus for the given orbital are
3/ 2
1 æZö
y radial = ç ÷ [(s 2 - 4s + 3)] exp( -s / 2)
9 2 è a0 ø

2Zr
where a0 & Z are the constants and s =
a0

a0 1 a0 3 a0 1 a0 a0 3 a0
(A) Zero, inifinty (B) , (C) , (D) ,
Z 2 Z 2 Z 2 Z Z 2 Z
Ans. (C)
Solution. Spherical node = y2 = 0
s 2 - 4s + 3 = 0 ; s 2 - 3s - s + 3 = 0
s(s - 3) - 1(s - 3) = 0 ; (s - 1)(s - 3) = 0
2Zr
s = 1,3 ; s = a = 1,3
0

3 a0 1 a0
r= ,
2 Z 2 Z

Illustration 49. For any orbital Y(r) = K .e–K’r (r4 + K1r3 + K2r2). Select true statement
(A) Orbital should be “5s”
(B) Orbital should be “5d”
(C) Angular wave function should be independent on (q, f)
(D) Curve of Y2(r) vs ‘r’ has maximum value at origin.
Ans. (B)
Solution. Y(r) = K .e–K’r[r2(r2 + K1r + K2)] Y(r) = K .e–K’r r2(r2 + K1r + K2)
Radial nodes = 2 n – l – 1 = 2
For l = 2
JPR\COMP.251\D\Allen(IIT-JEE Wing)\2020–21\Nurture\Che\Unit-01\Atomic Structure

Wave mechanical model


1. Consider y (wave function) of 2s atomic orbital of H-atom is -
r
1 é r ù - 2a0
y2s =
4 2p a 03 / 2 ê2 - a ú e
ë 0û

Find distance of radial node from nucleus in terms of a0.


(A) r = a0 (B) r = 2 a0 (C) r = a0• (D) data insufficient

82
Atomic Structure
+4
2. For an orbital in B radial function is :
3
1 æ z ö4 -s / 2
R(r) = ç ÷ (4 - s) s e
9 6 è a0 ø

Zr
where s = a and a0 = 0.529 Å ; Z = atomic number, r = radial distance from nucleus.
0
The radial node of orbital is at distance from nucleus.
(A) 0.529 Å (B) 2.12 Å (C) 1.06 Å (D) 0.423 Å

3. The wave function for an orbital is H-atom is given as


3 r
æ 2 öæ 1 ö2 æ r öæ r ö - 3a
ç ÷ ç ÷ ç 6 - ÷ç ÷ e 0 .sin q sin f
è 81 p ø è a 0 ø è a 0 øè a 0 ø
The orbital is :
(A) 2s (B) 3p (C) 2p (D) 3d

r
-
1
4. y1s = 3/2
e a0
pa 0
Find the distance 'r' from nucleus at which e– finding probability is max. [For H-atom] a0 ® I Bohr radius.

Comprehension for Q. 5 to Q.7


According to quantum mechanical model of H-like species, an electron can be represented by a wave function
(y) which contains all dynamic information about the electron. The nature of wave function depends on the type
of the orbital to which the electron belongs.
For an orbital
3/ 2 s
é 2 ùæ 1 ö 2
-
y= ê ú ça ÷ (27 - 18 s + 2s )e 3

ë 81 3 p ûè 0 ø

æ Zr ö
Where, s = ç a ÷ , r = radial distance from nucleus, a0 = 52.9 pm
è 0ø

5. The number of radial and angular nodes possible for the orbital given above are respectively :
(A) zero, zero (B) 0, 2 (C) 2, 0 (D) 2, 1

6. The orbital could possibly be :


(A) 4s (B) 4p (C) 3s (D) 3p

7. Which of the following represents the position of one of the radial nodes for Z = 1 ?
1 1 3 3
(A) r = (3 + 3)a 0 (B) r = (3 - 3)a 0 (C) r = (3 - 6)a 0 (D) r = (3 + 3)a 0
2 2 2 2
JPR\COMP.251\D\Allen(IIT-JEE Wing)\2020–21\Nurture\Che\Unit-01\Atomic Structure

12.0 ENERGY LEVEL DIAGRAMS


SL AL

A diagram representing relative energies of various orbitals in an atom is called level diagram.

12.1 Energy level diagram of Hydrogen atom


SL AL

In hydrogen atom there is only one electron which is present in 1s orbital in ground state. However, in excited
state the electron may jump to any of the orbitals belonging to higher energy levels. Energies of these various
orbitals can be calculated. The relative energies of these orbitals have been represented in fig.

83
JEE-Chemistr y

5
4s 4d 4d 4f
4
3s 3d 3d
3
Energy

2s 2p
2

1s
1
Energy level diagram of hydrogen atom (or mono electronic atoms)

It may be noted that all the orbitals of a particular energy level have same energy in hydrogen and hydrogen - like
atoms. For example 2s and 2p orbitals have same energy. The orbitals having equal energy are called degenerate.
Thus , for an atom having a single electron the principal quantum number n is most important in determining
the energy of the orbital. The value of l (the angular momentum quantum number) merely determines the shape
of the orbital.
Increasing energy : 1s < (2s = 2p) < (3s = 3p = 3d) < (4s = 4p = 4d = 4f) < ................

12.2 Energy level diagram of multi-electron atoms


SL AL

In case of multi-electron atoms the energies of various orbitals depend not only upon the nuclear charge but also
upon the other electrons present in the atom. In such atoms we have to consider not only the attraction between
the electron and the nucleus but also the repulsive interactions between the electron and the other electrons in
the atom.
The stability of an electron in a multielectron atom is due to the reason that the total attractive interactions are
greater than the total repulsive interactions. The attractive interactions of an electron increase with increase in
positive charge (Z) on the nucleus. However, due to the presence of electrons in the inner shells the electron does
not experience the full positive charge on the nucleus.
Thus an electron is shielded or screened from the attractive interactions of the nucleus by the electrons in the
inner shells. The net positive charge experienced by the electron from the nucleus is known as effective nuclear
charge (Zeff)
Zeff = Z – s
where s is the screening constant.
JPR\COMP.251\D\Allen(IIT-JEE Wing)\2020–21\Nurture\Che\Unit-01\Atomic Structure

It is impossible to calculate the exact energies of various orbitals in a multi-electron atom. However, approximate
values of their energies can be obtained from spectral data. Relative order of energies of various orbitals in all
multi-electron atoms is same and is illustrated in fig.

Main characteristic of this diagram are :


(i) The different sub-shells of a particular energy level may have different energies. For example , energy of 2s
sub-shell is different from the energy of 2p sub-shell.
(ii) In a particular energy level, the sub-shell having higher value of l has higher energy. For example , energy of
2p sub-shell (l = 1) is higher than energy of 2s sub-shell (l = 0). In general , increasing order of energies of
different types of sub-shells in a particular energy level is
s<p<d<f
84
Atomic Structure
6p 5d 4f

6s

5p 4d

5s
4p
3d

4s
Increasing energy

3p

3s

2p

2s

1s

Energy level diagram of multi-electron atoms

(iii) As the value of n increases, some sub-shells of lower energy level may have energy higher than the energy of
some sub-shells of higher energy. For example , energy of 3d is higher than the energy of 4s although the
latter belongs higher main energy level.
(iv) The increasing order of energies of various sub-shells in multi electron atom (case of empty sub-shells) is
decided by (n + l) rule.

12.3 Bohr Bury (n + l) rule (for multi-electronic atoms)


SL AL

The subshell having lowest (n + l) value has lower energy , when two or more subshell have same (n + l) value
JPR\COMP.251\D\Allen(IIT-JEE Wing)\2020–21\Nurture\Che\Unit-01\Atomic Structure

then the subshell with lowest value of n has lower energy.


Sub Shell n l n+l
1s 1 0 1
2s 2 0 2ù
2p 2 1 3úú (1)
3s 3 0 3úû (2)

3p 3 1 4 ù (1)
4s 4 0 4 úû (2)

85
JEE-Chemistr y
3d 3 2 5 ù (1)
4p 4 1 5 úú (2)
5s 5 0 5 úû (3)

4d 4 2 6 ù (1)
5p 5 1 6 úú (2)
6s 6 0 6 úû (3)

Increasing order of energy of sub-shells :


1s , 2s , 2p , 3s , 3p , 4s , 3d , 4p , 5s , 4d , 5p , 6s , 4f , 5d , 6p , 7s , 5f , 6d

13.0 RULES FOR FILLING OF ORBITALS IN AN ATOM


SL AL

(I) Aufbau Principle


(II) Pauli's exclusion principle
(III) Hund's maximum multiplicity principle

13.1 Aufbau Principle


SL AL

Aufbau is a German word and its meaning is ' Building up'.


l According to this rule, the electrons are added progressively to the various orbtials in the order of increasing
energies starting with the orbital of the lowest energy.

1s 2s 3s 4s 5s 6s 7s
Starting
point 2p 3p 4p 5p 6p 7p

3d 4d 5d 6d

4f 5f

Increasing order of energies of various orbitals is :


1s , 2s , 2p , 3s , 3p , 4s , 3d , 4p , 5s , 4d , 5p , 6s , 4f , 5d , 6p , 7s , 5f , 6d

13.2 Pauli's Exclusion principle


SL AL

In 1925 Pauli stated that no two electrons in an atom can have same values of all four quantum numbers, i.e.,
an orbital cannot have more than 2 electrons because three quantum numbers (principal , azimuthal and
magnetic) at the most may be same but the fourth must be different, i.e., spins must be in opposite directions.
JPR\COMP.251\D\Allen(IIT-JEE Wing)\2020–21\Nurture\Che\Unit-01\Atomic Structure

Ex. 6C
12
® 1s2 2s2 2p2
­¯ ­¯ ­­
px pz py
n 1 2 2
l 0 0 1
m 0 0 + 1, – 1, 0
1 1 1 1 1 1
s + ,– + ,– + ,+
2 2 2 2 2 2

86
Atomic Structure
Ex. 8O
16
® 1s2 2s2 2p4
­¯ ­¯ ­¯ ­ ­
n = 1 2 px pz py
l = 0 0 1
m = 0 0 +1 0 –1
1 1 1 1 1 1 1 1
s = + ,– + ,– + ,– + +
2 2 2 2 2 2 2 2

13.3 Hund's Maximum Multiplicity Rule


SL AL

(Multiplicity : Many of the same kind)


l According to Hund's rule electrons are distribuled among the degenerate orbitals of subshell in such a way
as to give maximum number of unpaired electron with parallel spin.

l Thus the orbital available in the subshell is first filled singly with parallel spin electron before they begin to
pair this means that pairing of electrons occurs with the introduction of second electron in 's' subshell, fourth
electron in 'p' subshell, 6th electron in 'd' subshell & 8th e– in 'f' subshell.

Ex. 26Fe ® 1s2 2s2 2p6 3s2 3p6 4s2 3d6


­¯ ­¯ ­¯ ­¯ ­¯ ­¯ ­¯ ­¯ ­¯ ­¯ ­¯ ­ ­ ­ ­

7N ® 1s2 2s2 2p3


­¯ ­¯ ­ ­ ­

17Cl ® 1s2, 2s2, 2p6, 3s2, 3p5


­¯ ­¯ ­¯ ­¯ ­¯ ­¯ ­¯ ­¯ ­

8O ® 1s2, 2s2, 2p4


­¯ ­¯ ­¯ ­ ­

19K ® 1s2, 2s2, 2p6 , 3s2 3p6, 4s1


­¯ ­¯ ­¯ ­¯ ­¯ ­¯ ­¯ ­¯ ­¯ ­

27Co ® 1s2, 2s2, 2p6 , 3s2 3p6, 4s2 3d7


­¯ ­¯ ­¯ ­¯ ­¯ ­¯ ­¯ ­¯ ­¯ ­¯ ­¯ ­¯ ­ ­ ­

13.4 Electronic configuration of atoms of various elements


SL AL

1H ® 1s1

2He ® 1s2
JPR\COMP.251\D\Allen(IIT-JEE Wing)\2020–21\Nurture\Che\Unit-01\Atomic Structure

3Li ® 1s2 , 2s1

4Be ® 1s2 , 2s2

5B ® 1s2 , 2s2 , 2p1

6C ® 1s2 , 2s 2 , 2p2

7N ® 1s2 , 2s2 , 2p3

8O ® 1s2 , 2s2 , 2p4

9F ® 1s2 , 2s2 , 2p5

10Ne ® 1s2 , 2s2 , 2p6

87
JEE-Chemistr y

11Na ® 1s2 , 2s2 , 2p6 , 3s1

12Mg ® 1s2 , 2s2 , 2p6 , 3s2

13Al ® 1s2 , 2s2 , 2p6 , 3s2 , 3p1

14Si ® 1s2 , 2s2 , 2p6 , 3s2 , 3p2

15p ® 1s2 , 2s2 , 2p6 , 3s2 , 3p3

16S ® 1s2 , 2s2 , 2p6 , 3s2 , 3p4

17Cl ® 1s2 , 2s2 , 2p6 , 3s2 , 3p5

18Ar ® 1s2 , 2s2 , 2p6 , 3s2 , 3p6

19K ® 1s2 , 2s2 , 2p6 , 3s2 , 3p6, 4s1

20Ca ® 1s2 , 2s2 , 2p6 , 3s2 , 3p6, 4s2

21Sc ® 1s2 , 2s2 , 2p6 , 3s2 , 3p6, 4s2, 3d1

22Ti ® 1s2 , 2s2 , 2p6 , 3s2 , 3p6, 4s2, 3d2

23V ® 1s2 , 2s2 , 2p6 , 3s2 , 3p6, 4s2, 3d3

24Cr ® 1s2 , 2s2 , 2p6 , 3s2 , 3p6, 4s1, 3d5 [Exception]

25Mn ® 2 2 6 2 6
1s , 2s , 2p , 3s , 3p , 4s , 3d 2 5

26Fe ® 1s2 , 2s2 , 2p6 , 3s2 , 3p6, 4s2, 3d6

27Co ® 1s2 , 2s2 , 2p6 , 3s2 , 3p6, 4s2, 3d7

28Ni ® 1s2 , 2s2 , 2p6 , 3s2 , 3p6, 4s2, 3d8

29Cu ® 1s2 , 2s2 , 2p6 , 3s2 , 3p6, 4s1, 3d10 [Exception]

30Zn ® 1s2 , 2s2 , 2p6 , 3s2 , 3p6, 4s2, 3d10

v Electronic configuration can be written by following different methods


l 26Fe ® (1) 1s2 , 2s2 , 2p6 , 3s2 , 3p6, 4s2, 3d6
(2) 1s2 , 2s2 , 2p6 , 3s2 , 3p6, 3d6 , 4s2
(3) 1s2 , 2s2p6 , 3s2p6d6 , 4s2
(4) [Ar] 4s2 3d6
2
1s2 2s2 2p 3s2 3p6 3d6 4s
6

l 26Fe ®
(n–2) (n–1) n
n ® Outer most Shell or Ultimate Shell or Valence Shell
In this Shell e– are called as valence electron or this is called core charge
(n – 1) ® Penultimate Shell of core or pre-valence Shell
(n – 2) ® Anti / Pre-penultimate Shell
l If we remove the last n Shell (ultimate Shell) then the remaining shell collectivly be called as Kernel.
JPR\COMP.251\D\Allen(IIT-JEE Wing)\2020–21\Nurture\Che\Unit-01\Atomic Structure

26 Fe ® 1s2 2s 2 2p6 3s2 3p6 3d6 4s2


Ex.
Kernel

13.5 Exception of Aufbau principle


SL AL

In some cases it is seen that the electronic configuration is slightly different from the arrangement given by
Aufbau principle. A simple region behind this is that half filled & full filled subshell have got extra stability.

Ex. 24Cr ® 1s2 2s2 2p6 3s2 3p6 4s2 3d4 (wrong configuration)
­¯ ­ ­­ ­

88
Atomic Structure

24Cr ® 1s2 2s2 2p6 3s2 3p6 4s1 3d5 (right configuration)
­ ­­­­­

29Cu ® 1s2 2s2 2p6 3s2 3p6 4s2 3d9 (wrong configuration)
­¯ ­¯ ­¯ ­¯ ­¯­

29Cu ® 1s2 2s2 2p6 3s2 3p6 4s1 3d10 (right configuration)
­ ­¯ ­¯ ­¯ ­¯ ­¯

v Explanation of Greater Stability of Half-filled and Fully-filled Electronic Configurations


The extra stability of half-filled and fully-filled electronic configuration can be explained in terms of symmetry
and exchange energy. The half-filled and fully-filled electronic configurations have symmetrical distribution of
electrons and this symmetry leads to stability. Moreover , in such configurations electrons can exchange their
positions among themselves to maximum extent. This exchange leads to stabilization which is expressed in terms
of exchange energy.

The half-filled and completely filled degenerate orbitals provide extra stability to the system due to
symmetrical arrangement and exchange energy as described below :
(i) Symmetrical arrangement : The electronic configurations in which all the orbitals of same sub-shell are
either completely - filled or exactly half - filled have relatively more symmetrical distribution of electrons and
therefore, lend more stability to the system. For example , expected configuration of chromium is 3d4 4s2.
But shifting of one electron from 4s to 3d-orbitals makes the configuration more symmetrical and hence
relatively more stable.

3d 4s 3d 4s
(Less symmetrical) (More symmetrical)
In the similar was shifting of one electron from 4s to 3d in copper also makes the configuration relatively
more stable.

3d 4s 3d 4s
(Less symmetrical) (More symmetrical)

(ii) Stability due to exchange energy : Degenerate electrons are known to exchange positions to decrease
energy. The half-filled or fully-filled degenerate orbitals have more number of exchanges and consequently,
have large exchange energy of stabilisation. The exchange means shifting of electrons from one orbital to
another within same sub-shell. Let us compare the number of exchanges in 3d4 4s2 and 3d5 4s1 configurations
of chromium.

3d
3 exchanges for
electron 1
JPR\COMP.251\D\Allen(IIT-JEE Wing)\2020–21\Nurture\Che\Unit-01\Atomic Structure

1 2 3 4 4s 1
4
(3d configuration)

2 exchanges for
electron 2
2

Total exchanges = 3 + 2 + 1 = 6 1 exchanges for


electron 3
3
Thus, in 3d4 arrangement electron exchanges are six which implies that there are six possible arrangements with
parallel spin in 3d4 configuration.
89
JEE-Chemistr y

3d 4s
4 exchanges for
electron 1
1 2 3 4 5 1
5
(3d configuration)

3 exchanges for
electron 2
2

2 exchanges for
electron 3
2

1 exchanges for
electron 4
4

Total exchanges = 4 + 3 + 2 + 1 = 10
It is quite evident from the above description that total number of electron exchanges in 3d5 arrangement is larger
which lends it relatively greater stability. In the similar way it can be established that number of exchanges in
3d10 configuration is larger than in 3d9 configuration which makes 3d10 configuration relatively more stable.
13.6 Electronic configuration of ions
SL AL

Ions are formed by loss or gain electrons by neutral atoms. A positive ion (or cation) is formed by loss of electron
whereas negative ion (or anion) is formed by gain of electrons. The electronic configurations of ions are written
just like the electronic configurations of atoms. However, it must be noted that while forming cations the
electrons are lost from the shell with highest value of principal quantum number. For example,
Fe : 1s2, 2s2, 2p6, 3s2, 3p6, 3d6, 4s2
Fe2+ : 1s2, 2s2, 2p6, 3s2, 3p6, 3d6
Fe3+ : 1s2, 2s2, 2p6, 3s2, 3p6, 3d5
Zn : 1s2, 2s2, 2p6, 3s2, 3p6, 3d10, 4s2
Zn2+ : 1s2, 2s2, 2p6, 3s2, 3p6, 3d10
The anions are formed by adding electrons to the vacant orbital of lowest energy.
F : 1s2, 2s2, 2p5
F– : 1s2, 2s2, 2p6
O : 1s2, 2s2, 2p4
O2–: 1s2, 2s2, 2p6

14.0 MAGNETIC PROPERTIES


JPR\COMP.251\D\Allen(IIT-JEE Wing)\2020–21\Nurture\Che\Unit-01\Atomic Structure

SL AL

v Paramagnetism
(i) The substances which are weakly attracted by magnetic field are paramagnetic and this phenomenon is
known as paramagnetism.
(ii) Their magnetic character is retained till they are in magnetic field and lose their magnetism when removed
from magnetic field.
(iii) They contain unpaired electrons.

v Diamagnetism
(i) The substances which are weakly repelled by magnetic field are diamagnetic and this phenomenon is
known as diamagnetism.
(ii) Diamagnetic substances lack unpaired electrons and their magnetic moment is zero e.g., NaCl, N2O4 etc.
90
Atomic Structure
v Ferromagnetism
(i) Solids which are strongly attracted by magnetic field and retain their magnetic property even if the field is
removed are known as ferromagnetic e.g., iron.
(ii) It cannot be observed in solution or liquid state.

v Total spin and spin magnetic moment


(i) The total spin(s) of an atom is given by s = 1/2 × n ; where n is number of unpaired electron in species.
(ii) The spin magnetic moment of electron (excluding orbit magnetic moment) is given by :

µeffective = Ö[4s(s+1)] or µeffective = Ö[n(n+2)] B.M. (Q s = 1/2 n)

The magnetic moment is expressed in Bohr magneton (B.M.)

Illustration 50. Select the wrong statement (s) from the following ?
(A) If the value of l = 0, the electron distribution is spherical
(B) The shape of the orbital is given by magnetic quantum number
(C) Angular momentum of 1s, 2s, 3s electrons are equal
(D) In an atom, all electrons travel with the same velocity
Solution. (B) is wrong because shape is given by azimuthal quantum number and magnetic quantum number
tells the orientation. (D) is wrong because electrons in different shells travel with different velocities.
So, (A) and (C) are the correct answer.

Illustration 51. Which of the following statement (s) is (are) correct ?


(A) The electronic configuration of Cr is [Ar]3d5, 4s1 (Atomic No. of Cr = 24)
(B) The magnetic quantum number may have a negative value
(C) In silver atom 23 electrons have spin of one type and 24 of the opposite type (Ag = 47)
(D) None of the above
Solution. (A), (B) and (C) are the correct answer since M can be positive/negative both and in Ag '23' electrons
have spin one direction and remaining '24' in another.

Illustration 52. What is the orbital angular momentum of an electron in 3p orbital.


h h
Solution Angular momentum (L) = l(l + 1) = 2 = .........
2p 2p

Illustration 53. What is spin angular momentum of an electron.


h
Solution Spin Angular momentum = s(s + 1)
2p

æ1ö3 h 3 h
ç ÷ = ´
è 2 ø 2 2p 4 2p
JPR\COMP.251\D\Allen(IIT-JEE Wing)\2020–21\Nurture\Che\Unit-01\Atomic Structure

Illustration 54. What is orbital angular momentum of an electron in 3d orbital.


h h 3h h
(A) 3 (B) 5 (C) (D) 6
2p 2p 2p 2p
Ans. (D)
® h
Solution Orbital angular momentum (L) = ( l )( l + 1)
2p

91
JEE-Chemistr y
Illustration 55. What are the possible set of quantum number for highest energy electron (in ground state) for
chlorine atom.
n l m
(A) 2 0 0
(B) 2 1 –2
(C) 3 1 2
(D) 3 1 0
Ans. (D)
Solution (n + l) is maximum for [D]

Illustration 56. The electrons identified by n & l ;


(i) n = 4, l = 1 (ii) n = 4, l = 0 (iii) n = 3, l = 2 (iv) n = 3, l = 1
can be placed in order of increasing energy from lowest to highest as
(A) (iv) < (ii) < (iii) < (i) (B) (ii) < (iv) < (i)
(C) (i) < (iii) < (ii) < (iv) (D) (iii) < (i) < (iv) < (ii)
Ans. (A)
Solution Apply Aufbau principle :
(i) The orbital with lowest value of (n + l) have lowest energy.
(ii) If n + l value for two orbitals are same, the orbital with lower value of n is having lower
energy.
for (i) n + l = 5
(ii) n + l = 4
(iii) n + l = 5
(iv) n + l = 4
for (iii) and (iv) n is lower hence correct order is (iv) < (ii) < (iii) < (i).

Illustration 57. A compound of vanadium has magnetic moment of 1.73 BM. Work out the electronic configuration
of vanadium ion in the compound.
Solution Vanadium belongs to 3d series with Z = 23. The magnetic moment of 3d series metal is given
by spin only formula.
µ= n(n + 2) BM (BM = Bohr's magneton)
Q 1.73 = 3
Þ n(n+2) = 3 Þ n = 1
Þ Magnetic moment correspond to one unpaired electron.
Þ Electronic configuration of vanadium atom 1s2 2s2 2p6 3s2 3p6 4s2 3d3.
For one unpaired electron 4 electron must be removed in which first 2 electron are lost from 4s
orbital (outermost).
Electronic configuration of V+4
1s2 2s2 2p6 3s2 3p6 4s0 3d1
JPR\COMP.251\D\Allen(IIT-JEE Wing)\2020–21\Nurture\Che\Unit-01\Atomic Structure

Electronic Configuration and Miscellaneous


1. A d-block element has total spin value of +3 or –3, then the magnetic moment of the element is
approximately:
(A) 2.83 B.M. (B) 3.87 B.M. (C) 5.9 B.M. (D) 6.93 B.M.

x+
2. Magnetic moment of 25 Mn is 15 B.M then the value of x is :
(A) 1 (B) 2 (C) 3 (D) 4

3. Magnetic moment of 26Fe2+ ion is same as


(A) 26Fe (B) 24Cr2+ (C) 28Ni4+ (D) All of these

92
Atomic Structure

h
4. Orbital angular momentum of an electron is 3 , then the number of orientations of this orbital in space
p
are
(A) 3 (B) 5 (C) 7 (D) 9

5. The correct order of the magnetic moment of [ 25Mn, 24Cr, 26Fe] is :


(A) Fe3+ > Cr+3 = Mn+4 (B) Fe3+ > Cr3+ > Mn4+
(C) Cr3+ = Mn4+ > Fe 3+ (D) Fe3+ > Mn+4 > Cr3+

6. Magnetic moment of Xn+ (Z = 26) is 24 B.M. Hence number of unpaired electrons and value of n respectively
are -
(A) 4, 2 (B) 2, 4 (C) 3, 1 (D) 0, 2

7. Magnetic moments of V (Z = 23), Cr (Z = 24), Mn (Z = 25) are x, y, z respectively. Hence :


(A) x = y = z (B) x < y < z (C) x < z < y (D) z < y < x

8. Predict total spin for each configuration.


(a) 1s2 (b) 1s2, 2s2 2p6 (c)1s2, 2s2 2p5 (d) 1s2, 2s2 2p3
(e) 1s2 2s2 2p6, 3s2 3p6 3d5, 4s2.

9. The electronic configuration of a carbon atom is 1s2, 2s2, 2p2 and consider the following four arrangements
of the 2p electrons. Which arrangement have lowest energy ?
(A) (B) (C) (D)
JPR\COMP.251\D\Allen(IIT-JEE Wing)\2020–21\Nurture\Che\Unit-01\Atomic Structure

93
JEE-Chemistr y

l Revolutions per sec = v / 2p

0.657 ´ Z 2 ´ 1016
r=
n3

1.52 ´ 10 -16 ´ n 3
l Time for one revolution = 2pr / v =
Z2
l No two elements will have identical line spectrum since no two elements have identical energy level therefore the
line spectrum of the elements are described as finger prints differing from each other like the finger prints of the
human beings.
l Since line spectrum is obtained by the emission of energy through the atoms of the element therefore line
spectrum is also called as atomic spectrum.
c
l Frequency, n=
l
hc
l Energy/photon, E = hn =
l
12375
Also, E = eV , if l is in Å
l

l Electronic energy change during transition, DE = E n2 - E n1 n2 > n1, emission spectra if electron jumps from
n2 to n1 shell and absorption spectra if electron excites from n1 to n2 shell.

n 2h 2
l Radius of nth Bohr orbit of H atom, rn = (where K = 9 × 109)
4 p2 me 2K

n2
r1 for H = 0.529 Å ; rn for H like atom rn = 0.529 × Å
Z

2pKZe 2
l Velocity of electron in nth Bohr orbit of H atom, v =
nh

Z
v = 2.18 × 108 cm / sec
n

2p2 mZ 2e4 K 2
l Energy of electron in nth Bohr orbit of H atom, E =
n 2h 2
JPR\COMP.251\D\Allen(IIT-JEE Wing)\2020–21\Nurture\Che\Unit-01\Atomic Structure

where n = 1, 2, 3..........

Z2
[E = –13.6 × kcal/mole (1 cal = 4.18 J)]
n2
E1 for H = – 21.72 × 10–12 erg = – 13.6 eV, E1 for H like atom = E1 for H × Z2

l Wavelength emitted during transition in H atom,

1 é1 1 ù 2p2 me4 é1 1ù
= RH ê 2 - 2 ú = ê 2 - 2 ú (in C.G.S.)
l ë n1 n 2 û ch 3 ë n1 n 2 û

94
Atomic Structure

1
l Photoelectric effect hv = w + mu 2 or hv = I.E. + K.E.
2

l Possible transitions for a jump from n2 to n1 = å (n 2 - n1 )

h h2
l de Broglie equation : l = =
mu 2 ´ K.E. ´ m

where l is wavelength, m is mass and u is velocity of particle.

l Heisenberg uncertainty principle :

h
Dp.Dx ³
4p

h
Du.Dx ³
4 pm
where Dp, Du and Dx are uncertainties in momentum, velocity and position respectively. Planck's constant
is h and m is mass of subatomic particle.

l Total no. of e– in main energy shell = 2n2


Total no. of e– in a subshell = 2 (2l + 1)
Maximum no. of e– in an orbital = 2
Total no. of orbitals in a subshell = (2l + 1)
No. of subshells in main energy shell = n
No. of orbitals in a main energy shell = n2
h
l Orbital angular momentum L = l(l +1)
2p
h 1
l Spin angular momentum S = s(s + 1) ; S =
2p 2
eh
l Spin only magnetic moment ms = 2p mc s( s + 1) or m= n (n + 2) B.M. (n = no. of unpaired electrons)

eh
l Orbital magnetic moment ms = 4p mc s( s + 1)

l Nodal planes : Radial nodes = n – l – 1, Angular nodes = l, Total nodes = (n – 1)

1
l A term multiplicity is also defined as M = 2|s| + 1 where |s| represents total spin = ´ nunpaired electrons .
2
JPR\COMP.251\D\Allen(IIT-JEE Wing)\2020–21\Nurture\Che\Unit-01\Atomic Structure

Hence Hund’s rule says that configuration should be such that multiplicity is maximum.

95
JEE-Chemistr y

SOME WORKED OUT EXAMPLES


Illustration 1.
Find the wavelengths of the first line of He+ ion spectral series whose interval with extreme lines is
1 1
- 4 –1
l1 l 2 = 2.7451 × 10 cm
Solution
Extreme lines means first and last
1 1 é1 1 ù é1 1 ù
- =RZ 2
ê 2
- 2 ú – RZ2 ê 2 - 2ú
l1 l 2 ë n1 ¥ û ë n1 (n1 + 1) û

1 1 RZ 2
or - =
l1 l 2 (n1 + 1)2

109677.76 ´ 22
4
2.7451 × 10 =
(n1 + 1)2
(n1 + 1) = 4
n1 = 3
Wavelength of first line,

1 é1 1 ù
= 109677.76 × 22 × ê 3 2 - 4 2 ú
l ë û
l = 4689 × 10–8 cm = 4689 Å

Illustration 2.
Find the wavelength emitted during the transition of electron in between two levels of Li2+ ion whose sum is
5 and difference is 3.
Solution.
Let the transition occurs between the level n1 and n2 and n2 > n1
Given that n1 + n2 = 5
n2 – n1 = 3
\ n1 = 1 and n2 = 4

é 1 1 ù
1 2ê - ú 2
é 15 ù
Therefore, = Rh × Z 2 = 109678 × (3) ê
êë (1) ë 16 úû
2
l ( 4 ) úû
\ l = 1.08 × 10–6 cm

Illustration 3.
The Lyman series of the hydrogen spectrum can be represented by the equation.

é 1 1 ù
JPR\COMP.251\D\Allen(IIT-JEE Wing)\2020–21\Nurture\Che\Unit-01\Atomic Structure

v = 3.2881 × 1015 s–1 ê 2 - ú


êë (1) ( n )2 úû
(where n = 2, 3,.....)
Calculate the maximum and minimum wavelength of lines in this series.
Solution.

é 1 1 ù
1 n 3.2881 ´ 1015 –1 ê - 2ú
n= = = m
êë (1)
2
l c 3 ´ 108 n úû

96
Atomic Structure

1
Wavelength is maximum ( nmin ) when n is minimum so that is maximum
n2

é ù
1 3.2881 ´ 1015 ê 1 - 1 ú
\ n min = =
êë (1)
2
l max 3 ´ 108 ( 2)2 úû

3 ´ 108 4
\ lmax = 15
´
3.2881 ´ 10 3
= 1.2165 × 10–7 m = 121.67 nm
Wavelength is minimum ( nmax ) when n is ¥
i.e. series converge

1 3.2881 ´ 1015
\ nmax = =
l min 3 ´ 108
\ lmin = 0.9124 × 10–7m 91.24 nm

Illustration 4.
When certain metal was irradiated with light frequency 0.4 × 1013 Hz the photo electrons emitted had twice
the kinetic energy as did photo electrons emitted when the same metal was irradiated with light frequency
1.0 × 1013 Hz. Calculate threshold frequency (n0) for the metal.
Solution.
hv = hn0 + KE
KE1 = h(n1 – n0)
KE1
KE2 = h(n2 – n0) =
2

v 2 - n0 1 1.0 ´ 1013 - n 0 1
\ v -n = Þ = Þ n0 = 1.6 × 1013 Hz
1 0 2 0.4 ´ 1013 - n 0 2

Illustration 5.
Iodine molecule dissociates into atoms after absorbing light of 3000 Å. If one quantum of radiation is absorbed
by each molecule, calculate the kinetic energy of iodine atoms. (Bond energy of I2 = 240 kJ (mol).
Solution.
Energy given to iodine molecule

hc 6.62 ´ 10 -34 ´ 3 ´ 108


= = 6.62 × 10–19 J
l 3000 ´ 10 -10
Also energy used for breaking up

240 ´ 10 3
I2 molecule = = 3.984 ´ 10 -19 J
6.023 ´ 10 23
JPR\COMP.251\D\Allen(IIT-JEE Wing)\2020–21\Nurture\Che\Unit-01\Atomic Structure

\ Energy used in imparting kinetic to two atoms = (6.62 – 3.984) × 10–19 J

( 6.62 - 3.984 ) ´ 10 -19


\ KE of iodine atom = = 1.318 × 10–19 J
2

Illustration 6.
Two hydrogen atoms collide head on and end up with zero kinetic energy. Each atom then emits a photon of
wavelength 121.6 nm. Which transition leads to this wavelength ? How fast were the hydrogen atoms travelling
before collision ?
Solution.
Wavelength is emitted in UV region and thus n1 = 1
97
JEE-Chemistr y

1 é1 1ù
For H atom = = RH ê 2 - 2 ú
l ë1 n û

1 é1 1ù
\ 7
-9 =1.097 × 10 ê 2
- 2ú
121.6 ´ 10 ë1 n û
\n=2
Also the energy released is due to collision and all the kinetic energy is released in form of photon.
1 hc
\ mv 2 =
2 l

1 6.625 ´ 10 -34 ´ 3 ´ 108


\ × 1.67 × 10–27 × v2 =
2 121.6 ´ 10 -9
\ v = 4.43 × 104 m/sec

Illustration 7.
Find the energy in kJ per mole of electronic charge accelerated by a potential of 2 volt.
Solution.
Energy in joules = charge in coulombs × potential difference in volt
= 1.6 × 10–19 × 6.02 × 1023 × 2 = 19.264 × 104 J or 192.264 kJ

Illustration 8.
Which hydrogen like ionic species has wavelength difference between the first line of Balmer and first line of
Lyman series equal to 59.3 × 10–9 m ? Neglect the reduced mass effect.
Solution.
Wave number of first Balmer line of an species with atomic number Z is given by

é1 1ù 2
v ' = RZ 2 ê 2 - 2 ú = 5RZ
ë2 3 û 36
Similarly wave number of v of first Lyman line is given by

2 é
1 1ù 3 2 1 1
v = RZ ê12 - 22 ú = 4 RZ ; v = l and v ' = l '
ë û

36 4 1 é 36 4 ù 88
\ l' – l = - = 2 ê
- ú= 2
5RZ 2
3RZ 2
RZ ë 5 3 û 15RZ

88
\ Z2 = -9
= 9 or Z = 3
59.3 ´ 10 ´ 15 ´ 1.097 ´ 107
\ Ionic species is Li2+

Illustration 9.
(i) What is highest frequency photon that can be emitted from hydrogen atom ? What is wavelength of this
JPR\COMP.251\D\Allen(IIT-JEE Wing)\2020–21\Nurture\Che\Unit-01\Atomic Structure

photon ?
(ii) Find the longest wavelength transition in the Paschen series of Be3+.
(iii) Find the ratio of the wavelength of first and the ultimate line of Balmer series of He+ ?
Solution.
(i) Highest frequency photon is emitted when electron comes from infinity to 1st energy level.

13.6Z2
E= - = -13.6 eV
12
or, 13.6 × 1.6 × 10–19 Joule = 2.176 × 10–18 Joule
E = hn

98
Atomic Structure

E 2.176 ´ 10-18 J
\ n= = = 0.328 × 1016 Hz
h 6.626 ´ 10-34 Js

c 3 ´ 108
n= \l= = 9.146 × 10–8 m
l 0.328 ´ 1016

é1 1ù
(ii) n = R H ´ Z2 ê 2 - 2 ú
n
ë 1 n 2û

For He ; Z = 4 ; For Paschen series n1 = 3


For longest wavelength n2 = 4

1 é1 1ù é1 1 ù 7
= 109678 × (4)2 × ê 2 - 2 ú = 109678 × 16 × ê - ú = 109678 × 16 ×
l ë3 4 û ë 9 16 û 144
l = 1172.20 Å

(iii) Wave number of first line of Balmer,

é1 1ù 5 ´ 4R 5R
n1 = RZ2 ê 2 - 2 ú = =
ë 2 3 û 36 9

9
\ Wavelength of first line of Balmer =
5R

2 é 1 1 ù 4R
Wave number of ultimate line of Balmer, n2 = RZ ê 2 - ú = =R
ë2 ¥û 4

1
\ Wavelength of ultimate line of Balmer =
R

9
\ Ratio =
5

Illustration 10.
An electron beam can undergo difraction by crystals. Through what potential should a beam of electrons be
accelerated so that its wavelength becomes equal to 1.0 Å.
Solution.
For an electron
1
mv 2 = eV where V is accelerating potential
2

h
l=
mv
2
JPR\COMP.251\D\Allen(IIT-JEE Wing)\2020–21\Nurture\Che\Unit-01\Atomic Structure

1 æ h ö
\ m = eV
2 çè ml ÷ø

( )
2
1 h2 1 ´ 6.625 ´ 10 -34
\ V= ´ = = 150.40 volt
2 ml 2 e 2 ´ 9.108 ´ 10 -31 ´ (1.0 ´ 10 -10 )2 ´ 1.602 ´ 10 -19

99
JEE-Chemistr y
Illustration 11.
The angular momentum of an electron in a Bohr's orbit of H-atom is 4.2178 × 10–34 kgm2/sec. Calculate the
wavelength of the spectral line emitted when electrons falls from this level to next lower level.
Solution.
nh
mvr =
2p

nh
= 4.2178 × 10–34
2p

4.2178 ´ 10-34 ´ 2 ´ 3.14


n= =4
6.625 ´ 10 -34

1 é1 1ù
= Rê 2 - 2ú
l ë n1 n2 û
The wavelength for transition from n = 4 to n = 3

1 é1 1ù
= 109678 ê 2 - 2 ú
l ë 3 4 û
l = 1.8 × 10–4 cm.

Illustration 12.
The kinetic energy of an electron in H like atom is 6.04 eV. Find the area of the third Bohr orbit to which this
electron belongs. Also report the atom.
Solution.
K.E. = 6.04 in 3rd orbit
Etotal = K.E. + P.E. = K.E. – 2 × K.E.
Þ –K.E. = – 6.04 eV
E1 for H = –13.6 eV and not for any orbit E = – 6.04 eV for H atom. Thus, atom for which K.E. is given
is other than H.
En H like atom = EnH × Z2

E1 13.6 2
´ Z2 Þ 6.04 = 2 ´ Z
n 2
3
Z2 = 3.99 » 4 Þ Z = 2

n2 32
\ The atom is He+ Þ rn = 0.529 × = 0.529 × = 2.3805 Å
Z 2

22
( )
2
Area, pr2 = ´ 2.3805 ´ 10-8 = 17.8 × 10–16 cm2
7

Illustration 13.
O2 undergoes photochemical dissociation into one normal oxygen atom and one oxygen atom 1.967 eV more
JPR\COMP.251\D\Allen(IIT-JEE Wing)\2020–21\Nurture\Che\Unit-01\Atomic Structure

energetic than normal. The dissociation of O2 into two normal atom of oxygen requires 498 kJmol–1. What
is the maximum wavelength effective for photo chemical dissociation of O2 ?
Solution.
We know
P2 ¾¾hn
® ONormal + OExcited
¾
O2 ¾® ONormal + ONormal
Energy required for simple dissociation of O2 into two normal atoms = 498 × 103Jmol–1

498 ´ 108
= Jmol -1
6.023 ´ 1023
If one atom in excited state has more energy, i.e.. 1.967 eV
100
Atomic Structure
–19
= 1.967 × 1.602 × 10 J
The energy required for photochemical dissociation of O2

498 ´ 103
= + 1.967 ´ 1.602 ´ 10-19
6.023 ´ 1023
= 82.68 × 10–20 + 31.51 × 10–20 = 114.19 × 10–20 Joule
hc
E=
l

6.625 ´ 10-34 ´ 3 ´ 108


114.19 × 10–20 =
l
l = 1740.2 × 10–10 m = 1740.2 Å.

Illustration 14.
The wave-mechanical model of atom is based upon :-
(A) de Broglie concept of dual character of matter
(B) Heisenberg's uncertainty principle
(C) Schrodinger wave equation
(D) All the above three
Ans. (D)

Illustration 15.
An orbital is correctly described by :-
(A) Y2 (B) Y (C) | Y2|Y (D) none
Ans. (A)

Illustration 16.
The orbital angular momentum of a d-electron is :-
(A) 6h (B) 2h (C) h (D) 2 h
Solution.

For d–electron, l = 2, orbital angular momentum = l ( l + 1) h = 2 ( 2 + 1) h = 6h


So, (A) is the correct answer

Illustration 17.
The following electron configuration of an atom in the ground state is not correct because :-
3s 3p 3d

(A) the energy of the atom is not minimum (B) Pauli's exclusion principle is violated
(C) Hund's rule is violated (D) Aufbau principle is not followed
Ans. (C)
JPR\COMP.251\D\Allen(IIT-JEE Wing)\2020–21\Nurture\Che\Unit-01\Atomic Structure

Illustration 18.
In the first bohr orbit of H atom the energy of an electron is –13.6 eV. The possible energy value (s) of excited
state (s) for electron in Bohr orbit of hydrogen is/are :-
(A) –3.4 eV (B) –4.2 eV (C) 6.8 eV (D) +6.8 eV
Solution.
-13.6
En = eV
n2

-13.6
For n = 2, E2 = = -3.4eV
4
So, (A) is the correct answer.
101
JEE-Chemistr y
Illustration 19.
The electronic configuration of an element is 1s2, 2s2, 2p6, 3s2, 3p6, 3d5, 4s1. This represents its :-
(A) excited state (B) ground state (C) cationic form (D) anionic form
Solution.
The given electronic configuration is ground state for chromium.
So, (B) is the correct answer

Illustration 20.
Which of the following sets of quantum number is/are incorrect ?
1 1
(A) n = 3, l = 3, m = 0, s = (B) n = 3, l = 2, m = 2, s = –
2 2

1 1
(C) n = 3, l = 1, m = 2, s = – (D) n = 3, l = 0, m = 0, s = +
2 2
Solution.
When n = 3, l cannot be 3] so (A) is incorrect when l = 1, m cannot be = +2.
So, (C) is incorrect
So, (A) and (C) is the correct answer.

Illustration 21.
Select the pairs of ions which have same electronic configuration ?
(A) Cr3+, Fe3+ (B) Fe3+, Mn2+ (C) Fe3+, Co3+ (D) Se3+, Cr3+
Solution.
Fe3+ and Mn2+ have same electronic configuration
So (B) is the correct answer.

Illustration 22.
If an electron in H atom has an energy of –78.4 kcal/mol. The orbit in which the electron is present
is :-
(A) 1st (B) 2nd (C) 3rd (D) 4th
Solution.
-313.6 -313.6
En = 2
kcal / mol Þ –78.4 = \n=2
n n2

Illustration 23.
What transition in the hydrogen spectrum would have the same wavelength as the Balmer transition,
n = 4 to n = 2 in the He+ spectrum ?
(A) n = 4 to n = 2 (B) n = 3 to n = 2 (C) n = 3 to n = 1 (D) n = 2 to n = 1
Solution.

1 æ1 1ö 3
n= = ç 2 - 2 ÷ RZ2 = R
l è 2 4 ø 4
JPR\COMP.251\D\Allen(IIT-JEE Wing)\2020–21\Nurture\Che\Unit-01\Atomic Structure

In H–spectrum for the same n or l as Z = 1, n = 1, n2 = 2


So, (D) is the correct answer.

Illustration 24.
Principal, azimuthal and magnetic quantum numbers are respectively related to :-
(A) size, orientation and shape (B) size, shape and orientation
(C) shape, size and orientation (D) none of these
Solution.
Principal gives size, i.e. azimuthal gives shape and magnetic quantum number gives the orientation.
So, (B) is the correct answer.

102
Atomic Structure
Illustration 25.
If the radius of 2nd Bohr orbit of hydrogen atom is r2. The radius of third Bohr orbit will be :-
4 9
(A) r2 (B) 4r2 (C) r2 (D) 9r2
9 4
Solution.

n2 h2
r=
4 p2 mZe 2

r2 22 9
\ r = 2 \ r3 = r2
3 3 4
So, (C) is the correct answer.

Illustration 26.
Difference between nth and (n + 1)th Bohr's radius of H–atom is equal to its (n – 1)th Bohr's radius. The value
of n is :-
(A) 1 (B) 2 (C) 3 (D) 4
Solution.
rn µ n2
But rn + 1 – rn = rn – 1
(n + 1)2 – n2 = (n – 1)2
n=4
So (D) is the correct answer

Illustration 27.
The dissociation energy of H2 is 430.53 kJ mol–1. If H2 is dissociated by illumination with radiation of wavelength
253.7 nm. The fraction of the radiant energy which will be converted into kinetic energy is given by :-
(A) 8.86% (B) 2.33% (C) 1.3% (D) 90%
Solution.

hc 430.53 ´ 103
= + K.E.
l 6.023 ´ 1023

6.626 ´ 10-34 ´ 3 ´ 108 430.53 ´ 103


K.E. = -9 – = 6.9 × 10–20
253.7 ´ 10 6.023 ´ 1023

6.9 ´ 10-20
\ Fraction = = 0.088 = 8.86%
7.83 ´ 10-19

Illustration 28.
No. of wave in third Bohr's orbit of hydrogen is :-
(A) 3 (B) 6 (C) 9 (D) 12
Solution.
JPR\COMP.251\D\Allen(IIT-JEE Wing)\2020–21\Nurture\Che\Unit-01\Atomic Structure

Circumference
Number of waves =
Wavelength

2pr 2pr 2p 2p nh
= = (mvr) = ´
l h / mv h h 2p
\n=3
So, (A) is the correct answer.

103
JEE-Chemistr y
Illustration 29.
In the hydrogen atoms, the electrons are excited to the 5th energy level. The number of the lines that may appear
in the spectrum will be :-
(A) 4 (B) 8 (C) 10 (D) 12
Solution.
No. of lines produced for a jump from fifth orbit to 1st orbit is given by

n ( n - 1) 5 ( 5 - 1)
= = = 10
2 2
So, (C) is the correct answer.

Illustration 30.
Light of wavelength l shines on a metal surface with intensity x and the metal emits Y electrons per second
of average energy, Z. What will happen to Y and Z if x is doubled ?
(A) Y will be double and Z will become half (B) Y will remain same and Z will be doubled
(C) Both Y and Z will be doubled (D) Y will be doubled but Z will remain same
Solution.
When intensity is doubled, number of electrons emitted per second is also doubled but average energy of
photoelectrons emitted remains the same.
So, (D) is the correct answer.

Illustration 31.
Which of the following is the ground state electronic configuration of nitrogen :-
(A) (B)

(C) (D)
Solution.
In (A) and (D), the unpaired electrons have spin in the same direction.
So, (A) and (D) are the correct answer.

Illustration 32.
For the energy levels in an atom, which one of the following statement/s is/are correct ?
(A) There are seven principal electron energy levels
(B) The second principal energy level can have four sub-energy levels and contain a maximum of eight
electrons
(C) The M energy level can have a maximum of 32 electrons.
(D) The 4s sub-energy level is at a lower energy than the 3d sub-energy level.
Solution.
(A) and (D) are true. (B) is wrong because for n = 2, l = 0, 1 (two sub-energy levels). (C) is wrong because
M shell means n = 3. Maximum electrons it can have = 2n2 = 2 × 32 = 18
So, (A) and (D) is the correct answer.

Illustration 33.
JPR\COMP.251\D\Allen(IIT-JEE Wing)\2020–21\Nurture\Che\Unit-01\Atomic Structure

Many elements have non-integral atomic masses because :-


(A) they have isotopes
(B) their isotopes have non-integral masses
(C) their isotopes have different masses
(D) the constituents, neutrons, protons and electrons combine to give rational masses
Solution.
Non-integral atomic masses are due to isotopes which have different masses.
So, (A) and (C) are the correct answer.

104
Atomic Structure
Illustration 34.
Give the sets of quantum numbers that describe an electron in a 3p orbital.
Solution
1 1
n = 3, l = 3, ml = +1, 0 or –1, ms = + or + .
2 2

Illustration 35.
Which of the following set of Quantum number not possible ?
(i) n = 2 (ii) n = 3 (iii) n = 4
l=0 l=2 l=3
m=–1 m=0 m=–2
1 1 1
s=– s= ± s= ±
2 2 2
Solution not possible possible possible

* * * * *
JPR\COMP.251\D\Allen(IIT-JEE Wing)\2020–21\Nurture\Che\Unit-01\Atomic Structure

105
JEE-Chemistr y

ANSWERS
BEGINNER'S BOX-1
1. 3.155 × 10–26 kg 2. 10–15 3. 6.3 × 106 m/s 4. 2.66 × 102sec
5. (D) 6. (AC) 7. 1000m

BEGINNER'S BOX-2
1. (A) 2. 0.526 3. 2.65 × 10–19 4. 1.125 × 1022
5. (a) 0.82 eV, (b) 544 nm 6. (a) 1.93 eV, (b) 1.93 volts
7. 3.313 × 10 –19
J 8. (A)
9. (B) 10. 1.988 × 10–19 J

BEGINNER'S BOX-3
1. (C) 2. (B) 3. (D) 4. (C) 5. (A)
6. (C) 7. (D) 8. (B) 9. (D) 10. (D)

BEGINNER'S BOX-4
1. (B) 2. (B) 3. 2.65 ´ 10-11 m 4. (B)
5. n2 = 5, Z = 3, Li . 2+
6. 937.3 Å 7. (A)

BEGINNER'S BOX-5
1. (B) 2. (C) 3. (A) 4. (D) 5. (C)
6. (B) 7. (D) 8. (A) 9. (C)

BEGINNER'S BOX-6
1. (D) 2. (D) 3. (B) 4. (C) 5. (D)
6. (C) 7. (B)

BEGINNER'S BOX-7
1. (B) 2. (D) 3. (B) 4. r = a0 = 0.529 Å
5. (C) 6. (C) 7. (D)

BEGINNER'S BOX-8
1. (D) 2. (D) 3. (D) 4. (C) 5. (A)
6. (A) 7. (C)
8. (a) 0 × (± 1/2) = 0
(b) 0 × (± 1/2) = 0
(c) 1 × (± 1/2) = ± 1/2
(d) 3 × (± 1/2) = ± 3/2
JPR\COMP.251\D\Allen(IIT-JEE Wing)\2020–21\Nurture\Che\Unit-01\Atomic Structure

(e) 5 × (± 1/2) = ± 5/2


9. (C)

106
Atomic Structure

SINGLE CHOICE CORRECT QUESTIONS


1. The maximum energy is present in any electron at :-
(A) Nucleus (B) Ground state
(C) First excited state (D) Infinite distance from the nucleus

2. The third line in Balmer series corresponds to an electronic transition between which Bohr's orbits in hydrogen :-
(A) 5 ® 3 (B) 5 ® 2 (C) 4 ® 3 (D) 4 ® 2

3. In compound FeCl2 the orbital angular momentum of last electron in its cation & magnetic moment (in Bohr
Magneton) of this compound are :-
(A) ( 6)h, 35 (B) ( 6)h, 24 (C) 0, 35 (D) none of these

4. If l0 is the threshold wavelength for photoelectric emission, l wavelength of light falling on the surface of metal,
and m mass of electron, then de Broglie wavelength of emitted electron is :-
1 1 1 1
é h( ll 0 ) ù 2 é h( l 0 - l ) ù 2 é h( l - l 0 ) ù 2 é hll0 ù 2
(A) ê ú (B) ê ú (C) ê ú (D) ê ú
ë 2mc(l 0 - l ) û ë 2mcll 0 ) û ë 2mcll 0 ) û ë 2mc û

5. The shortest wavelength of He+ in Balmer series is x, then longest wavelength in the Paschene series of Li+2 is :-
36x 16x 9x 5x
(A) (B) (C) (D)
5 7 5 9

6. An electron in a hydrogen atom in its ground state absorbs energy equal to ionisation energy of Li+2. The
wavelength of the emitted electron is :-
(A) 3.32 × 10–10 m (B) 1.17 Å (C) 2.32 × 10–9 nm (D) 3.33 pm

7. An electron, a proton and an alpha particle have kinetic energy of 16 E, 4E and E respectively. What is the
qualitative order of their de Broglie wavelengths :-
(A) le > lp = la (B) lp = la > le (C) lp > le > la (D) la < le >> lp
+3 – +
8. Given DH for the process Li (g) ¾® Li (g) + 3e is 19800 kJ/mole & IE1 for Li is 520 then IE2 & IE1 of Li
are respectively (approx value) :-
(A) 11775, 7505 (B) 19280, 520 (C) 11775, 19280 (D) Data insufficient
st nd
9. The ratio of difference in wavelengths of 1 and 2 lines of Lyman series in H-like atom to difference in wave-
nd rd
length for 2 and 3 lines of same series is :-
(A) 2.5 : 1 (B) 3.5 : 1 (C) 4.5 : 1 (D) 5.5 : 1

10. The quantum numbers of four electrons (e1 to e4) are given below :-
n l m s
JPR\COMP.251\D\Allen(IIT-JEE Wing)\2020–21\Nurture\Che\Unit-02\Atomic Structure

e1 3 0 0 +1/2
e2 4 0 0 1/2
e3 3 2 2 –1/2
e4 3 1 –1 1/2
The correct order of decreasing energy of these electrons is :
(A) e4 > e3 > e2 > e1 (B) e2 > e3 > e4 > e1 (C) e3 > e2 > e4 > e1 (D) none

11. If radius of second stationary orbit (in Bohr's atom) is R. Then radius of third orbit will be :-
(A) R/3 (B) 9R (C) R/9 (D) 2.25 R

12. The wavelength associated with a gold weighing 200 g and moving at a speed of 5 m/h is of the order :-
–10 –20 –30 –40
(A) 10 m (B) 10 m (C) 10 m (D) 10 m

107
JEE-Chemistr y
13. From the following observations predict the type of orbital :
Observation 1 : x y plane acts as nodal plane
Observation 2 : The angular function of the orbital intersect the three axis at origin only.
Observation 3 : R2(r) v/s r curve is obtained for the orbital is
2
R (r)

r
(A) 5pz (B) 6dxy (C) 6dx2 – y2 (D) 6 dyz

14. Electromagnetic radiations having l = 310 Å are subjected to a metal sheet having work function = 12.8 eV.
What will be the velocity of photoelectrons with maximum Kinetic Energy....
(A) 0, no emission will occur (B) 2.18 × 106 m/s
6
(C) 2.18 2 × 10 m/s (D) 8.72 × 106 m/s

15. If in Bohr's model, for unielectronic atom, time period of revolution is represented as Tn,z where n represents shell
no. and z represents atomic number then the value of T1,2 : T2,1 will be :-
(A) 8 : 1 (B) 1 : 8 (C) 1 : 1 (D) None of these

16. Which orbital is non-directional :-


(A) s (B) p (C) d (D) All

17. Uncertainty in position is twice the uncertainty in momentum Uncertainty in velocity is :-


h 1 h 1 h
(A) (B) (C) h (D)
p 2m p 2m 4p

18. The correct set of quantum numbers for the unpaired electron of chlorine atom is
n l m
(A) 2 1 0
(B) 2 1 1
(C) 3 1 1
(D) 3 0 0

19. Principal quantum number of an atom represents


(A) Size of the orbital (B) Spin angular momentum
(C) Orbital angular momentum (D) Space orientation of the orbital

20. The maximum number of electrons that can be accommodated in the Mth shell is
(A) 2 (B) 8 (C) 18 (D) 32

21. Which electronic level would allow the hydrogen atom to absorb a photon but not to emit a photon :-
(A) 3s (B) 2p (C) 2s (D) 1s

22. Correct set of four quantum numbers for valence electron of rubidium (Z = 37) is :-
JPR\COMP.251\D\Allen(IIT-JEE Wing)\2020–21\Nurture\Che\Unit-02\Atomic Structure

1 1 1 1
(A) 5, 0, 0, + (B) 5, 1, 0, + (C) 5, 1, 1, + (D) 6, 0, 0, +
2 2 2 2

23. The orbital diagram in which the Aufbau's principle is violated is :-


2s 2px 2p y 2p z 2s 2px 2py 2pz 2s 2px 2p y 2p z 2s 2 px 2py 2pz
(A) ­¯ (B) ­ (C) ­¯ ­ ­ (D) ­¯
­¯ ­ ­ ­¯ ­ ­ ­ ­¯ ­¯ ­

108
Atomic Structure
24. Which of the following sets of quantum numbers represent an impossible arrangement :-
n l m ms
1
(A) 3 2 –2
2
1
(B) 4 0 0
2
1
(C) 3 2 –3
2
1
(D) 5 3 0
2

25. The explanation for the presence of three unpaired electrons in the nitrogen atom can be given by :-
(A) Pauli's exclusions principle (B) Hund's rule
(C) Aufbau's principle (D) Uncertainty principle

26. The electronic configuration of an element is 1s2 2s2 2p6 3s2 3p6 3d5 4s1. This represents its :-
(A) Excited state (B) Ground state (C) Cationic form (D) None

27. Which of the following has maximum number of unpaired electron (atomic number of Fe 26) :-
(A) Fe (B) Fe (II) (C) Fe (III) (D) Fe (IV)

28. Which quantum number is not related with Schrodinger equation :-


(A) Principal (B) Azimuthal (C) Magnetic (D) Spin

29. Give the correct order of initials T (true) or F (false) for following statements :-
(I) If an ion has 2 electrons in K shell, 8 electrons in L shell and 6 electrons in M shell, then number of S
electrons present in that element is 6.
(II) The maximum number of electrons in a subshell is given by 2n2
(III) If electron has magnetic quantam number –1, then it cannot be present in s-orbital.
(IV) Only one radial node is present in 3p orbital.
(A) TTFF (B) FTTF (C) TFTT (D) FFTF
JPR\COMP.251\D\Allen(IIT-JEE Wing)\2020–21\Nurture\Che\Unit-02\Atomic Structure

109
JEE-Chemistr y

SECTION - 1 : MULTIPLE CHOICE CORRECT QUESTIONS


1. Choose the correct relation on the basis of Bohr's theory -

1 Z2
(A) velocity of electron µ (B) frequency of revolution µ
n n3

Z3
(C) radius of orbit µ n2Z (D) force on electron µ
n4

2. Hydrogen atoms in a particular excited state ‘n’, when all returned to ground state, 6 different photons are
emitted. Which of the following is/are incorrect.
(A) Out of 6 different photons only 2 photons have wavelength equal to that of visible light.
(B) If highest energy photon emitted from the above sample is incedent on the metal plate having work function
8 eV, AKE of liberated photo-electron may be equal to or less than 4.75 eV.
(C) Total number of radial nodes in all the orbitals of nth shell is 24.
(D) Total number of angular nodes in all the orbitals in (n-1)th shell is 23.

3. The change in orbital angular momentum corresponding to an electron transition inside a hydrogen atom can be
h h h h
(A) (B) (C) (D)
4p p 2p 8p

4. In which of these options do both constituents of the pair have the same magnetic moment -
(A) Zn2+ and Cu+ (B) Co2+ and Ni2+ (C) Mn4+ and Co2+ (D) Mg2+ and Sc+

5. If the wave number of 1st line of Balmer series of H-atom is ‘x’ then:

108 x
(A) wave number of 1st line of lyman series of the He+ ion will be
5
36 x
(B) wave number of 1st line of lyman series of the He+ ion will be
5
5
(C) the wave length of 2nd line of lyman series of H-atom is
32 x

32 x
(D) the wave length of 2nd line of lyman series of H-atom is
5

6. In a hydrogen like sample electron is in 2nd excited state, the Binding energy of 4th state of this sample is 13.6 eV,
then
JPR\COMP.251\D\Allen(IIT-JEE Wing)\2020–21\Nurture\Che\Unit-02\Atomic Structure

(A) A 25 eV photon can set free the electron from the second excited state of this sample.
(B) 3 different types of photon will be observed if electrons make transition up to ground state from the second
excited state
(C) If 23 eV photon is used then K. E. of the-ejected electron is 1 eV.
(D) 2nd line of Balmer series of this sample has same energy value as 1st excitation energy of H-atoms.

7. A hydrogen like atom in ground state absorbs ‘n’ photons having the same energy and it emits exactly ‘n’
photons when electronic transition takes place. Then the energy of the absorbed photon may be
(A) 91.8 eV (B) 40.8 eV (C) 48.4 eV (D) 54.4 eV

110
Atomic Structure

h
8. The orbital angular momentum of an electron is 3 . Which of the following may not be the permissible value
p
of angular momentum of this electron revolving in unknown Bohr orbit ?
h h h h
(A) (B) (C) 3. (D) 2.
2p p p p

9. For 24Cr which of the following statements is/are correct.


(A) Number of electrons with principle quantum number '3' are 13.
(B) Number of electrons with azimuthal quantum number '1' are 12.
(C) Number of electrons with magnetic quantum number '0' are 12.
(D) Number of minimum or maximum electrons with same spin quantum number are '9' or 15 respectively.

10. Choose the correct statement.


(A) dyz orbital lies in the xz plane. (B) pz orbital lies along x–axis
(C) Lobes of d x 2 - y 2 orbital are at 90º with z–axis (D) Lobes of dxy orbital are at 90º with z-axis.

SECTION - 2 : COMPREHENSION BASED QUESTIONS


(SINGLE CHOICE CORRECT QUESTION)
Comprehension-1
The general electronic configuration of outer most and penultimate shell is given as (n – 1)s2 (n – 1)p6
(n – 1)dxns2. Then for an element with n = 4 and x = 7.

11. The number of protons present in the divalent cation of the element of above configuration is :-
(A) 25 (B) 26 (C) 27 (D) 28

12. The element is isoelectronic with which of the following species :-


(A) Fe– (B) Ni+ (C) Cu2+ (D) All of these

13. The number of unpaired electron in the trivalent cation of the given element in isolated gaseous state is :-
(A) 0 (B) 3 (C) 4 (D) 1

Comprehension-2
Electronic configuration of elements give an idea about various properties of elements and hence it is one
of the basis for periodic classification of elements. An element prefers to stay in that configuration in which
its energy is least. A substance is said to show "Paramagnetic properties" if it has unpaired electrons.

14. Which of the following options correctly mention the configuration which is most stable among Q and R.

Q R
2s 2p 4s 3d
–I –I

2s 2p 4s 3d
JPR\COMP.251\D\Allen(IIT-JEE Wing)\2020–21\Nurture\Che\Unit-02\Atomic Structure

– II – II

(A) Q – I and R – I (B) Q – I and R – II


(C) Q – II and R – I (D) None of the above options

15. Which of the following is expected to be paramagnetic :


(A) Fe (B) Zn (C) Ne (D) Be

16. Specie having configuration same as ns2np4 where 'n' represents last shell is/are :
(A) 16O32 (B) 34Se79
19
(C) 9F (D) Both 16O32 and 34Se79

111
JEE-Chemistr y
Comprehension-3
Electron present in single electron species jumps from energy level 3 to 1. Emitted photons when passed through
a sample containing excited He+ ion causes further excitation to some higher energy level.
Z2
(Given En = 13.6 ):
n2

17. Determine atomic number of single electron species.


(A) 1 (B) 2 (C) 3 (D) 4

18. Determine Principal quantum number of initial excited level of He+.


(A) 1 (B) 2 (C) 4 (D) 6

19. Determine Principal quantum number of higher energy level of He+.


(A) 2 (B) 3 (C) 4 (D) 6

JPR\COMP.251\D\Allen(IIT-JEE Wing)\2020–21\Nurture\Che\Unit-02\Atomic Structure

112
Atomic Structure

SECTION - 1 : NUMERICAL ANSWER BASED QUESTIONS


1. A hydrogen like atom (atomic number Z) is in a higher excited state of quantum number n. This excited
atom can make a transition to the first excited state by successively emitting two photons of energies
10.20 eV and 17.00 eV respectively. Alternatively, the atom from the same excited state can make a
transition to the second excited state by successively emitting two photons of energy 4.25 eV and 5.95 eV
respectively. Determine the value of n + z. (ionisation energy of hydrogen atom = 13.6 eV).

2. If for any electron in an orbital another parameter ‘B’ is defined as


B = n + l + m, where n, l, m are the quantum numbers of that orbital then what will be the maximum value
of B for the last electron of 35Br.

3. In case of nitrogen, if M1 represents spin multiplicity if Hund’s rule is followed and M2 represents spin multiplicity
M1
if only Hund’s Rule is violated then the value of M will be:
2

4. A single electron orbits around a stationary nucleus of charge +Ze where Z is a constant from the nucleus and
e is the magnitude of the electric charge. The hydrogen like species required 47.2 eV to excite the electron from
the second Bohr orbit to the third Bohr orbit. Find the value of Z.

5. Find total no. of orbitals in nickel which have |m| £ 1 and at least one electron is present, where ‘m’ is magnetic
quantum number.
(Give your ans. as sum digits for example. If your ans is 57 then 5 + 7 = 12 and 1 + 2 =3)

6. Calculate total number of orbitals having (n + l) value = 8 and magnetic quantum number a non–zero
quantity.
(Given your ans. as sum digits for example. If your ans is 57 then 5 + 7 = 12 and 1 + 2 =3)

7. Maximum number of electrons in parallel spin in the group state of Chromium atom is:
(Given your ans. as sum digits for example. If your ans is 57 then 5 + 7 = 12 and 1 + 2 =3)

8. If the magnetic quantum number for an electron is –3, the minimum value for its principal quantum number
is:

h
9. The energy of an excited H-atom is –3.4 eV. If angular momentum of e– is p . Then calculate p.
p

10. Find the quantum number 'n' corresponding to the excited state of He+ ion if on transition to the ground state
that ion emits two photons in succession with wavelengths 108.5 and 30.4 nm.

SECTION - 2 : MATRIX - MATCH QUESTIONS


11. Frequency = f, Time period = T, Energy of nth orbit = En, radius of nth orbit = rn, Atomic number = Z, Orbit
JPR\COMP.251\D\Allen(IIT-JEE Wing)\2020–21\Nurture\Che\Unit-02\Atomic Structure

number = n :
Column-I Column-II
(A) f (p) n3
(B) T (q) Z2
1
(C) En (r)
n2
1
(D) rn (s) Z

113
JEE-Chemistr y
12. Column I & Column II contain data on Schrondinger Wave-Mechanical model, where symbols have their usual
meanings. Match the columns :-
Column I Column II (Type of orbital)

Yr
(A) (p) 4s

Yr 4pr
2 2

(B) (q) 5px

r
(C) Y (q, f) = K (independent of q & f) (r) 3s
(D) atleast one angular node is present (s) 6dxy

13. Match the following:


Column I Column II
(A) Same number of unpaired electrons are present in (p) Na+, Mg2+, F¯
(B) Same number of electron in s & p subshells. (q) F¯, Mg, O2–
(C) Same number of electrons with the l = 1 (r) Mg, Ne, O2–
(D) Same number of total electrons (s) Li, Na, K

14. Match the following:


Column I Column II
(A) Orbital with equal number of radial and angular nodes (p) 3d x 2 - y2
(B) Orbital with number of radial nodes less than the number of angular nodes (q) 2pz
(C) Orbital with zero radial nodes but two angular node. (r) 3px
(s) 5dxy

JPR\COMP.251\D\Allen(IIT-JEE Wing)\2020–21\Nurture\Che\Unit-02\Atomic Structure

114
Atomic Structure

1. Which one of the following constitutes a group of the isoelectronic species? [AIEEE-2008]
2- - + 2- - - 2- 2-
(1) C ,O ,CO,NO
2 2
(2) NO ,C ,CN ,N2
2
(3) CN ,N2 ,O ,C2 2
(4) N2 ,O ,NO + ,CO
-
2

2. The ionziation enthalpy of hydrogen atom is 1.312 x 106 J mol–1. The energy required to excite the electron in
the atom from n =1 to n =2 is [AIEEE-2008]
(1) 8.51 x 105 J mol–1 (2) 6.56 x 105 J mol–1 (3) 7.56 x 105 J mol–1 (4) 9.84 x 105 J mol–1

3. In an atom, an electron is moving with a speed of 600 m/s with an accuracy of 0.005%. Certainity with which
the position of the electron can be located is (h = 6.6 × 10 –34 kg m2 s–1, mass of electron, em = 9.1 × 10–31 kg):-
[AIEEE-2008]
(1) 1.92 × 10–3 m (2) 3.84 × 10–3 m (3) 1.52 × 10–4 m (4) 5.10 × 10–3 m

4. Calculate the wavelength (in nanometer) associated with a proton moving at 1.0 × 103 ms–1
(Mass of proton = 1.67 × 10–27 kg and h = 6.63 × 10–34 Js) :- [AIEEE-2009]
(1) 2.5 nm (2) 14.0 nm (3) 0.032 nm (4) 0.40 nm

5. Ionisation energy of He+ is 19.6 × 10–18 J atom–1. The energy of the first stationary state (n = 1) of Li2+ is:-
[AIEEE-2010]
(1) 8.82 × 10–17 J atom–1 (2) 4.41 × 10–16 J atom–1
(3) –4.41 × 10–17 J atom–1 (4) –2.2 × 10–15 J atom–1

6. A gas absorbs a photon of 355 nm and emits at two wavelengths. If one of the emissions is at 680 nm, the
other is at :- [AIEEE-2011]
(1) 743 nm (2) 518 nm (3) 1035 nm (4) 325 nm

7. The frequency of light emitted for the transition n = 4 to n = 2 of He+ is equal to the transition in H atom
corresponding to which of the following :- [AIEEE-2011]
(1) n = 3 to n = 1 (2) n = 2 to n = 1 (3) n = 3 to n = 2 (4) n = 4 to n = 3

8. The electrons identified by quantum numbers n and l :- [AIEEE ONLINE - 2012]


(a) n = 4 , l = 1 (b) n = 4, l = 0 (c) n = 3, l = 2 (d) n = 3, l = 1
Can be placed in order of increasing energy as
(1) (a) < (c) < (b) < (d) (2) (c) < (d) < (b) < (a)
(3) (d) < (b) < (c) < (a) (4) (b) < (d) < (a) < (c)

9. If the kinetic energy of an electron is increased four times, the wavelength of the de-Broglie wave associated
with it would become :- [AIEEE ONLINE - 2012]
(1) Two times (2) Half (3) One fourth (4) Four times

10. Which of the following paramagnetic ions would exhibit a magnetic moment (spin only) of the order of
5 BM ? [AIEEE ONLINE - 2012]
(At. Nos : Mn = 25, Cr = 24, V = 23, Ti = 22)
(1) V2+ (2) Ti2+ (3) Mn2+ (4) Cr2+
JPR\COMP.251\D\Allen(IIT-JEE Wing)\2020–21\Nurture\Che\Unit-02\Atomic Structure

11. a, v and u represent most probable velocity, average velocity and root mean square velocity respectively of a
gas at a particular temperature. The correct order among the following is :- [AIEEE ONLINE - 2012]
(1) a > u > v (2) v > u > a (3) u > v > a (4) u > a > v

12. The wave number of the first emission line in the Balmer series of H-Spectrum is :
(R = Rydberg constant) : [JEE MAINS - ONLINE - 2013]
3 9 5 7
(1) R (2) R (3) R (4) R
4 400 36 6

115
JEE-Chemistr y
13. Given [JEE MAINS - ONLINE - 2013]
(a) n=5, ml= +1 (b) n = 2, l = 1, ml = – l, ms = – l/2
The maximum number of electron(s) in an atom that can have the quantum numbers as given in (a) and (b)
are respectively :
(1) 8 and 1 (2) 25 and 1 (3) 2 and 4 (4) 4 and 1

14. The energy of an electron in first Bohr orbit of H-atom is – 13.6 eV. The energy value of electron in the excited
state of Li2+ is : [JEE MAINS - ONLINE - 2014]
(1) –30.6 eV (2) –27.2 eV (3) 27.2 eV (4) 30.6 eV

15. If l0 and l be the threshold wavelength and wavelength of incident light, the velocity of photoelectron ejected
from the metal surface is [JEE MAINS - ONLINE - 2014]

2hc æ l 0 - l ö 2h æ 1 1 ö 2h 2hc
(1) ç
m è ll 0 ø
÷ (2) ç - ÷
m è l0 l ø
(3) ( l0 - l ) (4) ( l0 - l )
m m

16. Ionization energy of gaseous Na atoms is 495.5 kjmol–1. The lowest possible frequency of light that ionizes a
sodium atom is [JEE MAINS - ONLINE - 2014]
(h = 6.626 × 10–34 Js, NA = 6.022 × 1023 mol–1)
(1) 3.15 × 1015 s–1 (2) 4.76 × 1014 s–1 (3) 1.24 × 1015 s–1 (4) 7.50 × 104 s–1

17. Which of the following is the energy of a possible excited state of hydrogen ? [JEE MAINS-2015]
(1) + 13.6 eV (2) – 6.8 eV (3) – 3.4 eV (4) + 6.8 eV

18. The total number of orbitals associated with the principal quantum number 5 is :
[JEE MAINS - ONLINE - 2016]
(1) 25 (2) 5 (3) 20 (4) 10

19. If the shortest wavelength in Lyman series of hydrogen atom is A, then the longest wavelength in Paschen series
of He+ is : [JEE MAINS - ONLINE - 2017]
36A 9A 36A 5A
(1) (2) (3) (4)
5 5 7 9

20. The radius of the second Bohr orbit for hydrogen atom is : [JEE MAINS-2017]
(Planck's Const. h = 6.6262 × 10–34 Js; mass of electron = 9.1091 × 10–31 kg; charge of electron
e = 1.60210 × 10–19 C; permittivity of vacuum Î0 = 8.854185 × 10–12 kg–1 m–3A2)
(1) 2.12 Å (2) 1.65Å (3) 4.76 Å (4) 0.529 Å

æ 1 ö
21. For emission line of atomic hydrogen from ni = 8 to nf = n the plot of wave number (v) against ç 2 ÷
èn ø
will be (The Rydberg constant, R H is in wave number unit). [JEE MAINS - ONLINE - 2019]
(1) Linear with slope - RH (2) Linear with intercept - RH
(3) Non linear (4) Linear with slope RH
JPR\COMP.251\D\Allen(IIT-JEE Wing)\2020–21\Nurture\Che\Unit-02\Atomic Structure

22. The 71st electron of an element X with an atomic number of 71 enters into the orbital :
[JEE MAINS - ONLINE - 2019]
(1) 4f (2) 6p (3) 6s (4) 5d

116
Atomic Structure
23. Which of the graphs shown below does not represent the relationship between incident light and the electron
ejected form metal surface ? [JEE MAINS - ONLINE - 2019]

number K.E. of
– –
(1) of e s (2) e s

0 0
Frequency of Intensity of
light light

K.E. of K.E. of
e– s e– s
(3) (4)

0 0
Frequency of energy of
light light

24. Heat treatment of muscular pain involves radiation of wavelength of about 900 nm. Which spectral line of
H-atom is suitable for this purpose ? [JEE MAINS - ONLINE - 2019]
[RH = 1 × 105 cm–1, h = 6.6 × 10–34 Js, c = 3 × 108 ms–1]
(1) Paschen, 5 ® 3 (2) Paschen, ¥ ® 3 (3) Lyman, ¥ ® 1 (4) Balmer, ¥ ® 2

25. The quantum number of four electrons are given below - [JEE MAINS - ONLINE - 2019]
I. n = 4, l = 2, ml = –2, ms = – ½ II. n = 3, l = 2, ml = 1, ms = + ½
III. n = 4, l = 1, ml = 0, ms = + ½ IV. n = 3, l = 1, ml = 1, ms = – ½
The correct order of their increasing energies will be -
(1) IV < III < II < I (2) IV < II < III < I (3) I < II < III < IV (4) I < III < II < IV

26. For any given series of spectral lines of atomic hydrogen, let Dn = n max – n min be the difference in maximum

and minimum frequencies in cm–1. The ratio Dn Lyman / Dn Balmer is : [JEE MAINS - ONLINE - 2019]
(1) 27 : 5 (2) 4 : 1 (3) 5 : 4 (4) 9 : 4

27. The graph betweeen |y|2 and r(radial distance) is shown below. This represents :-

2
|y|
[JEE MAINS - ONLINE - 2019]

r
(1) 3s orbital (2) 1s orbital (3) 2p orbital (4) 2s orbital
JPR\COMP.251\D\Allen(IIT-JEE Wing)\2020–21\Nurture\Che\Unit-02\Atomic Structure

28. The ratio of the shortest wavelength of two spectral series of hydrogen spectrum is found to be about 9. The
spectral series are: [JEE MAINS - ONLINE - 2019]
(1) Paschen and Pfund (2) Lyman and Paschen
(3) Brackett and Pfund (4) Balmer and Brackett

1
29. The number of orbitals associated with quantum numbers n = 5, ms = + is :
2
[JEE MAINS - ONLINE - 2020]
(1) 11 (2) 25 (3) 15 (4) 50

117
JEE-Chemistr y

1. The energy required to break one mole of Cl–Cl bonds in Cl2 is 242 kJ mol–1. The longest wavelength of light
capable of breaking a single Cl–Cl bond is (C = 3 × 108 ms–1 and NA = 6.02 × 1023 mol–1) [AIEEE-2010]
(1) 494 nm (2) 594 nm (3) 640 nm (4) 700 nm

2. The limiting line in Balmer series will have a frequency of : [AIEEE ONLINE - 2012]
(Rydberg constant, RH = 3.29 × 1015 cycles/s)
(1) 3.65 × 1014 s–1 (2) 8.22 × 1014 s–1 (3) 3.29 × 1015 s–1 (4) 5.26 × 1013 s–1

3. If the radius of first orbit of H atom is a0, the de-Broglie wavelength of an electron in the third orbit is :-
[AIEEE ONLINE - 2012]
(1) 6 p a0 (2) 8 p a0 (3) 2 p a0 (4) 4 p a0

4. The de Broglie wavelength of a car of mass 1000 kg and velocity 36 km/hr is :


(h = 6.63 × l0–34 Js) [JEE MAINS - ONLINE - 2013]
–31 –34 –38
(1) 6.626 × l0 m (2) 6.626 × 10 m (3) 6.626 × 10 m (4) 6.626 × 10–30 m

5. Based on the equation

æ 1 1 ö
DE = -2.0 ´10-18 J ç 2 - 2 ÷ [JEE MAINS - ONLINE - 2014]
è n 2 n1 ø
the wavelength of the light that must be absorbed to excite hydrogen electron from level n = 1 to level
n = 2 will be (h = 6.625 × 10–34 Js, C = 3 × 108 ms–1)
(1) 2.650 × 10–7 m (2) 1.325 × 10–7 m (3) 1.325 × 10–10 m (4) 5.300 × 10–10 m

6. Excited hydrogen atom emits light in the ultraviolet region at 2.47 × l015 Hz. With this frequency, the energy
of a single photon is :- (h = 6.63 × 10– 34 Js) [JEE MAINS - ONLINE - 2014]

(1) 2.680 × 10–19 J (2) 6.111 × 10–17 J (3) 1.640 × 10–18 J (4) 8.041 × 10–40 J

7. The de-Brogile wavelenths of a particle of mass 6.63 g moving with a velocity 100 ms–1 is :-
[JEE MAINS - ONLINE - 2014]
(1) 10–35 m (2) 10–33 m (3) 10–25 m (4)10–31 m

8. The correct set of four quantum numbers for the valence electrons of rubidium atom (Z = 37) is:
[JEE MAINS-2014]
1 1 1 1
(1) 5, 0, 0 + (2) 5, 1, 0 + (3) 5, 1, 1 + (4) 5, 0, 1 +
2 2 2 2

9. The heat of atomization of methane and ethane are 360 kJ/mol and 620 kJ/mol, respectively. The longest wavelength
of light capable of breaking the C-C bond is :
JPR\COMP.251\D\Allen(IIT-JEE Wing)\2020–21\Nurture\Che\Unit-02\Atomic Structure

(Avogadro number = 6.02 × 1023) [JEE MAINS - ONLINE - 2015]


h = 6.62 × 10–34 J s):
(1) 1.49 × 103 nm (2) 2.48 × 104 nm (3) 2.48 × 103 nm (4) 1.49 × 104 nm

3
10. At temperature T, the average kinetic energy of any particle is kT- The de Broglie wavelength follows the
2
order : [JEE MAINS - ONLINE - 2015]
(1) Visible photon > Thermal electron > Thermal neutron
(2) Thermal proton > Thermal electron > Visible photon
(3) Visible photon > Thermal neutron > Thermal electron
(4) Thermal proton > Visible photon > Thermal electron

118
Atomic Structure
11. A stream of electrons from a heated filament was passed between two charged plates kept at a potential
difference V esu. If e and m are charge and mass of an electron, respectively, then the value of h/l (where l is
wavelength associated with electron wave) is given by: [JEE MAINS-2016]
(1) me V (2) 2me V (3) meV (4) 2meV

12. The electron in the hydrogen atom undergoes transition from higher orbitals to orbital of radius 211.6 pm. This
transition is associated with:- [JEE MAINS - ONLINE - 2017]
(1) Brackett series (2) Balmer series (3) Lyman series (4) Paschen series

13. Which of the following combination of statements is true regarding the interpretation of the atomic orbitals ?
[JEE MAINS - ONLINE - 2019]
(a) An electron in an orbital of high angular momentum stays away from the nucleus than an electron in the
orbital of lower angular momentum.
(b) For a given value of the principal quantum number, the size of the orbit is inversely proportional to the azimuthal
quantum number.
h
(c) According to wave mechanics, the ground state angular momentum is equal to .
2p
(d) The plot of y Vs r for various azimuthal quantum numbers, shows peak shifting towards higher r value.
(1) (b), (c) (2) (a), (d) (3) (a), (b) (4) (a), (c)

14. The de Broglie wavelength (l) associated with a photoelectron varies with the frequency (v) of the incident radiation
as, [v0 is thershold frequency] : [JEE MAINS - ONLINE - 2019]

1 1
1 1 1
1
(1) lµ (2) lµ (3) lµ (4) lµ
3
(v - v0 )2 (v - v0 )4 (v - v0 )
(v - v0 )2

15. What is the work function of the metal if the light of wavelength 4000 Å generates photoelectrons of velocity
6 × 105 ms–1 form it ? [JEE MAINS - ONLINE - 2019]
(Mass of electron = 9 × 10–31 kg
Velocity of light = 3 × 108 ms–1
Planck's constant = 6.626 × 10–34 Js
Charge of electron = 1.6 × 10–19 JeV–1)
(1) 0.9 eV (2) 4.0 eV (3) 2.1 eV (4) 3.1 eV

16. If p is the momentum of the fastest electron ejected from a metal surface after the irradiation of light having
wavelength l, then for 1.5 p momentum of the photoelectron, the wavelength of the light should be:
(Assume kinetic energy of ejected photoelectron to be very high in comparison to work function)
[JEE MAINS - ONLINE - 2019]
1 3 2 4
(1) l (2) l (3) l (4) l
2 4 3 9

17. Which one of the following about an electron occupying the 1s orbital in a hydrogen atom is incorrect ? (The
JPR\COMP.251\D\Allen(IIT-JEE Wing)\2020–21\Nurture\Che\Unit-02\Atomic Structure

Bohr radius is represented by a0) [JEE MAINS - ONLINE - 2019]


(1) The electron can be found at a distance 2a0 from the nucleus
(2) The probability density of finding the electron is maximum at the nucleus.
(3) The magnitude of potential energy is double that of its kinetic energy on an average.
(4) The total energy of the electron is maximum when it is at a distance a0 from the nucleus.

119
JEE-Chemistr y
18. The electrons are more likely to be found : [JEE MAINS - ONLINE - 2019]

(1) in the region a and b (2) in the region a and c


(3) only in the region c (4) only in the region a

19. Among the following, the energy of 2s orbital is lowest in : [JEE MAINS - ONLINE - 2019]
(1) K (2) Na (3) Li (4) H

20. The radius of the second Bohr orbit, in terms of the Bohr radius, a0, in Li2+ is :
[JEE MAINS - ONLINE - 2020]
4a 0 2a 0 2a 0 4a 0
(1) (2) (3) (4)
9 9 3 3

ì1 1ü
21. For the Balmer series in the spectrum of H atom, n = R H í 2 - 2 ý , the correct statements among (I) and
î n1 n 2 þ
(IV) are :
(I) As wavelength decreases, the lines in the series converge
(II) The integer n1 is equal to 2
(III) The lines of longest wavelength corresponds to n2 = 3
(IV) The ionization energy of hydrogen can be calculated from wave number of these lines
[JEE MAINS - ONLINE - 2020]
(1) (II), (III), (IV) (2) (I), (II), (III) (3) (I), (III), (IV) (4) (I), (II), (IV)

22. The de Broglie wavelength of an electron in the 4th Bohr orbit is : [JEE MAINS - ONLINE - 2019]
(1) 8pa0 (2) 2pa0 (3) 4pa0 (4) 6pa0

JPR\COMP.251\D\Allen(IIT-JEE Wing)\2020–21\Nurture\Che\Unit-02\Atomic Structure

120
Atomic Structure

1. Match the entries in Column I with the correctly related quantum number(s) in Column II. Indicate your
answer by darkening the appropriate bubbles of the 4 × 4 matrix given in the ORS. [JEE 2008]
Column I Column II
(A) Orbital angular momentum of the (p) Principal quantum number
electron in a hydrogen-like atomic orbital
(B) A hydrogen-like one-electron wave (q) Azimuthal quantum number
function obeying Pauli principle
(C) Shape, size and orientation of hydrogen (r) Magnetic quantum number
like atomic orbitals
(D) Probability density of electron at the nucleus (s) Electron spin quantum number
in hydrogen-like atom

Paragraph for questions 2 to 4 [JEE 2010]


The hydrogen-like species Li2+ is in a spherically symmetric state S1 with one radial node. Upon absorbing
light the ion undergoes transition to a state S2. The state S2 has one radial node and its energy is equal
to the ground state energy of the hydrogen atom.

*2. The state S1 is :-


(A) 1s (B) 2s (C) 2p (D) 3s

*3. Energy of the state S1 in units of the hydrogen atom ground state energy is :-
(A) 0.75 (B) 1.50 (C) 2.25 (D) 4.50

*4. The orbital angular momentum quantum number of the state S2 is :-


(A) 0 (B) 1 (C) 2 (D) 3

5. The maximum number of electrons that can have principal quantum number, n=3, and spin quantum number,
ms = – 1/2, is [JEE 2011]

*6. The work function (f) of some metals is listed below. The number of metals which will show photoelectric
effect when light of 300 nm wavelength falls on the metal is :- [JEE 2011]

Metal Li Na K Mg Cu Ag Fe Pt W
f(eV) 2.4 2.3 2.2 3.7 4.8 4.3 4.7 6.3 4.75

*7. The kinetic energy of an electron in the second Bohr orbit of a hydrogen atom is [a0 is Bohr radius]
[JEE 2012]
h2 h2 h2 h2
(A) (B) (C) (D)
4 p 2 ma 20 16 p 2 ma 20 32 p 2 ma 20 32 p 2 ma 20
JPR\COMP.251\D\Allen(IIT-JEE Wing)\2020–21\Nurture\Che\Unit-02\Atomic Structure

1
8. In an atom, the total number of electrons having quantum numbers n = 4 |ml| = 1 and ms = - is
2
[JEE 2014]

9. Not considering the electronic spin the degeneracy of the second excited state (n = 3) of H atom is 9, while the
degeneracy of the second excited state of H– is

121
JEE-Chemistr y
*10. P is the probability of find the 1s electron of hydrogen atom in a spherical shell of infinitesimal thickness, dr, at
a distance r from the nucleus. The volume of this shell is 4pr2dr. The qualitative sketch of the dependence of
P on r is [JEE 2016]

P P P P

(A) (B) (C) (D)

O r O r O r O r

Answer Q.11, Q.12 and Q.13 by appropriately matching the information given in the three columns of
the following table.
The wave function, Y n, l, m is a mathematical function whose value depends upon spherical polar coordinates
1

(r, q, f) of the electron and characterized by the quantum numbers n, l and m1. Here r is distance from nucleus,
q is colatitude and f is azimuth. In the mathematical functions given in the Table, Z is atomic number and a0 is
Bohr radius.

Column I Column 2 Column 3


(I) 1s orbital (i) Y n, l, m1 (P)
3/2 æ Zr ö

æ Z ö a ÷
µç ÷ e è 0ø
è a0 ø

0
r / a0
(II) 2s orbital (ii) One radial node (Q) Probability density at
1
nucleus µ 3
a0
(III) 2pz orbital (iii) Y n, l, m1 (R) Probability density is
æ Zr ö
maximum at nucleus
5/2

æ Z ö 2a ÷
µç ÷ re è 0 ø cos q
è a0 ø
(IV) 3d 2z orbital (iv) xy-plane is a nodal plane (S) Energy needed to excite
electron from n = 2 state to
27
n = 4 state is times the
32
energy needed to excite
electron from n = 2 state to
n = 6 state

*11. For the given oribtal in Coumn 1, the only CORRECT combination for any hydrogen-like species is
[JEE 2017]
(A) (IV) (iv) (R) (B) (II) (ii) (P) (C) (III) (iii) (P) (D) (I) (ii) (S)
JPR\COMP.251\D\Allen(IIT-JEE Wing)\2020–21\Nurture\Che\Unit-02\Atomic Structure

*12. For He+ ion, the only INCORRECT combination is [JEE 2017]
(A) (II) (ii) (Q) (B) (I) (i) (S) (C) (I) (i) (R) (D) (I) (iii) (R)

*13. For hydrogen atom, the only CORRECT combination is [JEE 2017]
(A) (I) (iv) (R) (B) (I) (i) (P) (C) (II) (i) (Q) (D) (I) (i) (S)

122
Atomic Structure
14. The ground state energy of hydrogen atom is –13.6 eV. Consider an electronic state Y of He+ whose energy,
azimuthal quantum number and magnetic quantum number are –3.4 eV, 2 and 0 respectively. Which of the
following statement(s) is(are) true for the state Y? [JEE 2019]

(A) It has 2 angular nodes


(B) It has 3 radial nodes
(C) It is a 4d state
(D) The nuclear charge experienced by the electron in this state is less than 2e, where e is the magnitude of the
electronic charge.

Questions 15 to 16 [JEE 2020]


Answer the following by appropriately matching the lists based on the information given in the
paragraph
Consider the Bohr's model of a one-electron atom where the electron moves around the nucleus. In the
following List-I contains some quantities for the nth orbit of the atom and List-II contains options showing how
they depend on n.
List-I
List-II
(I) Radius of the nth orbit (P) µ n–2
(II) Angular momentum of the electron in the nth orbit (Q) µ n–1
th
(III) Kinetic energy of the electron in the n orbit (R) µ n0
th
(IV) Potential energy of the electron in the n orbit (S) µ n1
(T) µ n2
(U) µ n1/2
15. Which of the following options has the correct combination considering List-I and List-II?
(A) (II), (R) (B) (I), (P) (C) (I), (T) (D) (II), (Q)

16. Which of the following options has the correct combination considering List-I and List-II?
(A) (III), (S) (B) (IV), (Q) (C) (IV), (U) (D) (III), (P)
JPR\COMP.251\D\Allen(IIT-JEE Wing)\2020–21\Nurture\Che\Unit-02\Atomic Structure

123
JEE-Chemistr y

ANSWERS
EXERCISE-1
Que. 1 2 3 4 5 6 7 8 9 10 11 12 13 14 15
Ans. D B B A B B A A B C D C D C D
Que. 16 17 18 19 20 21 22 23 24 25 26 27 28 29
Ans. A C C A C D A B C B C C D C

EXERCISE-2
l MULTIPLE CHOICE CORRECT QUESTIONS
1. (ABD) 2. (CD) 3. (BC) 4. (AC) 5. (AC)
6. (AB) 7. (AB) 8. (AB) 9. (ABCD) 10. (CD)

l COMPREHENSION BASED QUESTIONS


11. (C) 12. (D) 13. (C) 14. (B) 15. (A)
16. (D) 17. (A) 18. (B) 19. (D)

EXERCISE-3
l NUMERICAL ANSWER BASED QUESTIONS
1. (9) 2. (6) 3. (2) 4. (5) 5. (4)
6. (3) 7. (6) 8. (4) 9. (1) 10. (5)

l MATRIX MATCH QUESTIONS


11. (A) - (q), (B) - (p), (C) - (qr), (D) - (rs) 12. (A) - (p), (B) - (pqs), (C) - (pr), (D) - (qs)
13. (A) - (pqrs), (B) - (p), (C) - (pqr), (D) - (p) 14. (A) - (rs), (B) - (qp), (C) - (p)

EXERCISE-4(A)
Que. 1 2 3 4 5 6 7 8 9 10 11 12 13 14 15
Ans. 2 4 1 4 3 1 2 3 2 3 3 3 1 1 1
Que. 16 17 18 19 20 21 22 23 24 25 26 27 28 29
Ans. 3 3 1 3 1 4 1 3 2 2 2 4 2 2

EXERCISE-4(B)
Que. 1 2 3 4 5 6 7 8 9 10 11 12 13 14 15
Ans. 1 2 1 3 2 3 2 1 1 1 4 2 2 2 3
Que. 16 17 18 19 20 21 22
Ans. 4 4 2 1 4 2 1

EXERCISE-5
1. (A)- q,r (B)- p, q, r, s (C)- p, q, r (D)- p, q 2. (B) 3. (C)
JPR\COMP.251\D\Allen(IIT-JEE Wing)\2020–21\Nurture\Che\Unit-02\Atomic Structure

4. (B) 5. (9) 6. (4) 7. (C) 8. (6)


9. (3) 10. (B) 11. (B) 12. (D) 13. (D)
14. (AC) 15. (C) 16. (D)

124
Atomic Structure

Not To Be Discussed in Class

SECTION - 1 : SINGLE CHOICE CORRECT QUESTIONS


1. Which of the following quantum number will be for 5d sub-shell ?
(A) n = 5, l = 2 (B) n = 6, l = 3 (C) n = 4, l = 0 (D) n = 5, l = 4

2. What is the maximum number of electrons in the possible sub-shells for n + l = 4 ?


(A) 8 (B) 6 (C) 12 (D) 16

3. Which of the following should be the possible sub-shells for n + l = 7 ?


(A) 7s 6p 5d 4f (B) 4f 5p 6s 4d (C) 7s 6p 5d 6d (D) 4s 4d 6p 7s

4. An atom has a mass of 0.02 kg and uncertainty in its velocity is 9.218 × 10–6 m/s then uncertainty in position
is (h = 6.626 × 10–34 Js)
(A) 2.86× 10–28 m (B) 2.86 × 10–32 cm (C) 1.5 × 10–27 m (D) 3.9 × 10–10 m

5. Energy of H- atom in the ground state is –13.6 eV, Hence energy in the second excited state is-
(A) –6.8 eV (B) –3.4 eV (C) –1.51 eV (D) –4.3 eV

6. Uncertainty in position of a particle of 25 g in space is 10–5 m. Hence uncertainty in velocity (ms–1) is (Planck's
constant h = 6.6 × 10–34 Js)
(A) 2.1 × 10–28 (B) 2.1 × 10–34 (C) 0.5 × 10–34 (D) 5.0 × 20–24

h
7. The orbital angular momentum for an electron revolving in an orbit is given by l(l + 1) . . This momentum
2p
for an s-electron will be given by
h 1 h h
(A) 2. (B) + . (C) zero (D)
2p 2 2p 2p

8. The number of d-electrons retained in Fe2+ (At. no. of Fe = 26) ion is :


(A) 6 (B) 3 (C) 4 (D) 5

9. The de Broglie wavelength of a tennis ball of mass 60 g moving with a velocity of 10 metres per second is
approximately :
(A) 10–25 metres (B) 10–33 metres (C) 10–31 metres (D) 10–16 metres

10. In Balmer series of lines of hydrogen spectrum, the third line from the red end corresponds to which one of the
following inter-orbit jumps of the electron for Bohr orbits in an atom of hydrogen ?
(A) 2 ® 5 (B) 3 ® 2 (C) 5 ® 2 (D) 4 ® 1

11. Which of the following sets of quantum number is correct for an electron in 4f orbital ?
1 1
(A) n = 3, l = 2, m = – 2, s = + (B) n = 4, l = 4, m = – 4, s = –
2 2
JPR\COMP.251\D\Allen(IIT-JEE Wing)\2020–21\Nurture\Che\Unit-01\Atomic Structure

1 1
(C) n = 4, l = 3, m = + 1, s = + (D) n = 4, l = 3, m = + 4, s = +
2 2

12. Consider the ground state of Cr atom (Z = 24). The numbers of electrons with the azimuthal quantum numbers,
l = 1 and 2 are, respectively
(A) 16 and 5 (B) 12 and 5 (C) 16 and 4 (D) 12 and 4

13. The wavelength of the radiation emitted, when in a hydrogen atom electron falls from infinity to stationary
state 1, would be (Rydberg constant = 1.097× 107 m–1) :
(A) 9.1 × 10–8 nm (B) 192 nm (C) 406 nm (D) 91 nm

177
JEE-Chemistr y

14. Which one of the following sets of ions represents the collection of isoelectronic species ?
(A) Na+, Mg2+, Al3+, Cl– (B) Na+, Ca2+, Sc3+, F– (C) K+, Cl–, Mg2+, Sc3+ (D) K+, Ca2+, Sc3+, Cl–

15. In a multi-electron atom, which of the following orbitals described by the three quantum members will have the
same energy in the absence of magnetic and electric fields ?
(a) n = 1, l = 0, m = 0 (b) n = 2, l = 0, m = 0
(c) n = 2, l = 1, m = 1 (d) n = 3, l = 2, m = 1
(e) n = 3, l = 2, m = 0
(A) (d) and (e) (B) (c) and (d) (C) (b) and (c) (D) (a) and (b)

16. Of the following sets which one does not contain isoelectronic species ?
(A) BO33- , CO32- , NO-3 (B) SO32- , CO32- , NO-3 (C) CN - , N 2 , C 22- (D) PO34- , SO42- , ClO-4

17. Which of the following statements in relation to the hydrogen atom is correct ?
(A) 3s, 3p and 3d orbitals all have the same energy
(B) 3s and 3p orbitals are of lower energy than 3d orbitals
(C) 3p orbital is lower in energy than 3d orbital
(D) 3s orbitals is lower in energy than 3p orbital

18. According to Bohr's theory angular momentum of electron in 5th shell is :-


(A) 1.0 h/p (B) 10 h/p (C) 2.5 h/p (D) 25 h/p

19. Uncertainty in the position of an electron (mass = 9.1 × 10–31 Kg) moving with a velocity 300 ms–1, accurate
upto 0.001%, will be :- (h = 6.63 × 10–34 Js)
(A) 5.76 × 10–2 m (B) 1.92 × 10–2 m (C) 3.84 × 10–2 m (D) 19.2 × 10–2 m

20. Which of the following sets of quantum numbers represents the highest energy of an atom ?
(A) n = 3, l = 1, m =1, s = +½ (B) n = 3, l = 2, m =1, s = +½
(C) n = 4, l = 0, m =0, s = +½ (D) n = 3, l = 0, m =0, s = +½

21. For which orbital angular probability distribution is maximum at an angle of 45° to the axial direction :-
(A) dx2 – y2 (B) dz2 (C) dxy (D) Px

22. Which orbit would be the first to have 'g' subshell :-


(A) 3rd (B) 4th (C) 5th (D) 6th

23. The decreasing order of energy of the 3d, 4s, 3p, 3s orbitals is :-
(A) 3d > 3s > 4s > 3p (B) 3s > 4s > 3p > 3d (C) 3d > 4s > 3p > 3s (D) 4s > 3d > 3s > 3p

24. If n and l are respectively the principle and azimuthal quantum numbers, then the expression for calculating the
total number of electrons in any orbit is :-
l =n l = n -1 l = n +1 l = n -1
(A) å 2(2l + 1) (B) å 2(2l + 1) (C) å 2(2l + 1) (D) å 2(2l + 1)
l =1 l =1 l =0 l =0
JPR\COMP.251\D\Allen(IIT-JEE Wing)\2020–21\Nurture\Che\Unit-01\Atomic Structure

25. If wavelength is equal to the distance travelled by the electron in one second, then :-
h h h h
(A) l = (B) l = (C) l = (D) l =
pv m p m

26. According to Schrodinger model nature of electron in an atom is as :-


(A) Particles only (B) Wave only
(C) Both simultaneously (D) Sometimes waves and sometimes particle

27. Which describes orbital :-


(A) y (B) y2 (C) |y2|y (D) All

178
Atomic Structure
28. In order to have the same wavelength for the electron (mass me) and the neutron (mass mn) their velocities
should be in the ratio (electron velocity/neutron velocity) :-
(A) mn/me (B) mn × me (C) me/mn (D) one

29. The quantum numbers +1/2 and –1/2 for the electron spin represent :-
(A) Rotation of the electron in clockwise and anticlockwise direction respectively.
(B) Rotation of the electron in anticlockwise and clockwise direction respectively.
(C) Magnetic moment of the electron pointing up and down respectively.
(D) Two quantum mechanical spin states which have no classifical analogue.

30. Which is true about y :-


(A) y represents the probability of finding an electron around the nucleus
(B) y represent the amplitude of the electron wave
(C) Both A and B
(D) None of these

31. Consider an electron in the nth orbit of a hydrogen atom in the Bohr model. The circumference of the orbit can
be expressed in terms of the de Broglie wavelength l of the electron as :-
(A) (0.529) nl (B) nl (C) (13.6) l (D) nl

32. A particle X moving with a certain velocity has a debroglie wave length of 1A°. If particle Y has a mass of 25%
that of X and velocity 75% that of X, debroglies wave length of Y will be :-
(A) 3 A° (B) 5.33 A° (C) 6.88 A° (D) 48 A°

33. What are the values of the orbital angular momentum of an electron in the orbitals 1s, 3s, 3d and 2p :-
(A) 0, 0, 6h, 2h (B) 1, 1, 4h, 2h (C) 0, 1, 6h, 3h (D) 0, 0, 20h, 6

34. If m = magnetic quantum number and l = azimuthal quantum number then :-


m -1
(A) m = l + 2 (B) m = 2l2 + 1 (C) l = (D) l = 2m + 1
2

35. The number of unpaired electrons in Mn4+ (Z = 25) is :-


(A) Four (B) Two (C) Five (D) Three

36. After np orbitals are filled, the next orbital filled will be :-
(A) (n + 1) s (B) (n + 2) p (C) (n + 1) d (D) (n + 2) s

37. The value of the magnetic moment of a particular ion is 2.83 Bohr magneton. The ion is :-
2+ 2+ 2+ 3+
(A) Fe (B) Ni (C) Mn (D) Co

38. In Bohr's model of the hydrogen atom the ratio between the period of revolution of an electron in the orbit of
n = 1 to the period of the revolution of the electron in the orbit n = 2 is :-
(A) 1 : 2 (B) 2 : 1 (C) 1 : 4 (D) 1 : 8

39. Let u1 be the frequency of the series limit of the Lyman series, u2 be the frequency of the first line of the lyman
JPR\COMP.251\D\Allen(IIT-JEE Wing)\2020–21\Nurture\Che\Unit-01\Atomic Structure

series, and u3 be the frequency of the series limit of the Balmer series :-
(A) u1 – u2 = u3 (B) u2 – u1 = u3 (C) u3 = 1/2 (u1 – u3) (D) u1 + u2 = u3

40. The energies of energy levels A, B and C for a given atom are in the sequence EA < EB < EC. If the radiations
of wavelengths l1, l2 and l3 are emitted due to the atomic transitions C to B, B to A and C to A respectively then
which of the following relations is correct :-
l l
(A) l1 + l2 + l3 = 0 (B) l3 = l1 + l22 (C) l3 = l1 + l2 (D) l3= 1 2
l1 + l2

+
41. The wavelengths of photons emitted by electron transition between two similar levels in H and He are l1 and
l2 respectively. Then :-
(A) l2= l1 (B) l2= 2l1 (C) l2= l1/2 (D) l2 = l1/4

179
JEE-Chemistr y
42. If first ionization potential of an atom is 16 V, then the first excitation potential will be :-
(A) 10.2 V (B) 12 V (C) 14 V (D) 16 V

43. In which transition minimum energy is emitted :-


(A) ¥ ® 1 (B) 2 ® 1 (C) 3 ® 2 (D) n ® (n – 1) (n ³ 4)

44. No. of visible lines when an electron returns from 5th orbit to ground state in H spectrum :-
(A) 5 (B) 4 (C) 3 (D) 10

45. Which of the following has maximum number of unpaired electron (atomic number of Fe 26)
(A) Fe (B) Fe (II) (C) Fe (III) (D) Fe (IV)

46. Suppose a particle has four quantum numbers such that the permitted values are:
n = 1, 2, 3........
l = (n – 1), (n – 3), (n – 5) ...........but no negative value
æ 1ö æ 1ö
j = çè l + ÷ø or çè l - ÷ø , the later is not negative
2 2
m = –j in integer step to +j what are the other permitted values for n = 2?
3 3 1 1
(A) l = 1, j = ,m = - (B) l = 0, j = ;m = -
2 2 2 2
1 3
(C) l = 1, j = ;m = - (D) All of these
2 2

47. Which of the following is / are false.


(A) Multiplicity in Fe3+ is greater than Co3+.
(B) Ti3+, Cr+, Sc2+ ions are diamagnetic.
(C) Value of (n + l + m) for last electron in Na is 3.
(D) Number of neutron in 1.07 gm Fe(OH)3 is 3.25 × 1023

48. The orbital diagram in which Hund’s rule is violated is

(I) (II)

(III) (IV)
(A) I & II (B) II & III (C) III & IV (D) II & IV

49. Match the column:


(‘l’ and ‘m’ are respectively the azimuthal and magnetic quantum numbers)
Column I Column II
(I) Total number of values of (l) for a shell (P) 0, 1, 2, .................(n–1)
(II) Values of (l) for a shell (Q) +l, .....+2, +1, 0, –1, –2, .......–l
(III) Total number of values of (m) for a subshell (R) (2l + 1)
(IV) Values of (m) for a subshell (S) n
(A) (I) S (II) P (III) R (IV) Q (B) (I) S (II) R (III) P (IV) Q
(C) (I) P (II) S (III) R (IV) Q (D) (I) P (II) R (III) Q (IV) S
JPR\COMP.251\D\Allen(IIT-JEE Wing)\2020–21\Nurture\Che\Unit-01\Atomic Structure

50. Which of the following statements regarding subshell filling order for a neutral atom is/are correct ?
(I) Electrons are assigned to the 4s subshell before they are assigned to the 3d subshell
(II) Electrons are assigned to the 4f subshell before they are assigned to the 6s subshell
(III) Electrons are assigned to the 4d subshell before they are assigned to the 5p subshell
(A) I only (B) II only (C) I and III (D) I, II and III

51. If the nitrogen atom had electronic configuration 1 s7, it would have energy lower that of normal ground state
configuration 1s2 2s2 2p3, because the electrons would be closer to the nucleus. Yet 1 s7 is not observed because
it violates :-
(A) Heisenberg uncertainty principle (B) Hunds rule
(C) Pauli's exclusion principle (D) Bohr postulate of stationary orbits

180
Atomic Structure
52. The explanation for the presence of three unpaired electrons in the nitrogen atom can be given by
(A) Pauli’s exclusion principle (B) Hund’s rule
(C) Aufbau’s principle (D) Uncertainity principle

53. Which of the following has the maximum number of unpaired electrons ?
(A) Mg2+ (B) Ti3+ (C) V3+ (D) Fe2+

54. The electrons, identified by n & l ;


(i) n = 4 , l = 1 (ii) n = 4 , l = 0 (iii) n = 3 , l = 2
(iv) n = 3 , l = 1 can be placed in order of increasing energy, from the lowest to highest as :
(A) (iv) < (ii) < (iii) < (i) (B) (iii) < (ii) < (iv) < (i)
(C) (i) < (iii) < (ii) < (iv) (D) (iii) < (i) < (iv) < (ii)

56. The magnetic moment of cobalt of the compund Hg[Co(SCN)4] is [Given : Co+2]
(A) 3 (B) 8 (C) 15 (D) 24

57. The radius of which of the following orbit is same as that of the first Bohr’s orbit of hydrogen atom?
(A) He+ (n = 2) (B) Li2+ (n = 2) (C) Li2+ (n = 3) (D) Be3+ (n = 2)

58. The magnetic moment of cobalt of the compund Hg[Co(SCN)4] is [Given : Co+2]
(A) 3 (B) 8 (C) 15 (D) 24

SECTION - 2 : MULTIPLE CHOICE CORRECT QUESTIONS


59. Which of the following statements is/are incorrect ?
(A) Lanthanum is the first element of lanthanides (B) Actinium violates the Aufbau's principle.
(C) Chromium violates the Pauli's exclusion principle. (D) Total 10 exchanges are possible for d electrons in Zn.

60. Bohr's theory is not applicable to -


(A) He (B) Li2+ (C) He2+ (D) the H-atom

61. In which transition, one quantum of energy is emitted -


(A) n = 4 ® n = 2 (B) n = 3 ® n = 1 (C) n = 4 ® n = 1 (D) n = 2 ® n = 1

62. The magnitude of the spin angular momentum of an electron is given by -

h h 3 h 1 h
(A) S = s(s + 1) (B) S = s (C) S = ´ (D) S = ± ´
2p 2p 2 2p 2 2p

63. A hydrogen - like atom has ground state binding energy 122.4 eV. Then :
(A) its atomic number is 3
(B) an photon of 90 eV can excite it to a higher state
JPR\COMP.251\D\Allen(IIT-JEE Wing)\2020–21\Nurture\Che\Unit-01\Atomic Structure

(C) an 80 eV photon cannot excite it to a higher state


(D) an electron of 8.2 eV and a photon of 91.8 eV are emitted when a 100 eV electron interacts with it

64. In a hydrogen like sample two different types of photons A and B are produced by electronic transition. Photon
B has it’s wavelength in infrared region if photon A has more energy then B, then the photon A may belong to
the region.
(A) ultraviolet (B) visible (C) infrared (D) None

65. In a H-like sample electrons make transition from 4th excited state to 2nd state then
(A)10 different spectral lines are observed
(B) 6 different spectral lines are observed
(C) number of lines belonging to the balmier series is 3
(D) Number of lines belonging to paschen series is 2.

181
JEE-Chemistr y
66. Gaseous state electronic configuration of nitrogen atom can be represented as :
(A) ­¯ ­¯ ­ ­ ­ (B) ­¯ ­¯ ­ ¯ ­ (C) ­¯ ­¯ ­ ¯ ¯ (D) ­¯ ­¯ ¯ ¯ ¯

67. The electronic configuration of an element is 1s2 2s2 2p6 3s2 3p6 3d5 4s1. This represents its :
(A) excited state (B) ground state (C) cationic form (D) none

SECTION - 3 : COMPREHENSION BASED QUESTIONS


Paragraph for Q.68 to Q.73
Read the following rules and answer the questions at the end of it.
Electrons in various suborbits of an orbit are filled in increasing order to their energies.
Pairing of electrons in various orbitals of a suborbit takes place only after each orbital is half-filled.
No two electrons in an atom can have the same set of quantum number.

68. Cr (Z = 24), Mn+ (Z = 25), Fe2+ (Z = 26) and Co3+ (Z = 27) are isoelectronic each having 24 electrons.
Thus,
(A) all have configurations as [Ar] 4s1 3d5
(B) Cr and Mn+ have configurations as [Ar] 4s1 3d5 while Fe2+ and Co3+ have configurations as [Ar]3d6.
(C) all have configurations as [Ar] 3d6
(D) all have configurations as [Ar] 4s2 3d6

69. A compound of vanadium has a magnetic moment of 1.73 BM. Electronic configuration of the vanadium
ion in the compound is :
(A) [Ar] 4s0 3d1 (B) [Ar] 4s2 3d3 (C) [Ar] 4s1 3d0 (D) [Ar] 4s0 3d5

70. Which of these ions are expected to be paramagnetic and coloured in aqueous solution ?
(A) Fe3+, Ti3+, Co3+ (B) Cu+, Ti4+, Sc3+ (C) Fe3+, Ni2+, V5+ (D) Cu+, Cu2+, Fe2+

71. While writing the following electronic configuration of Fe some rules have been violated :
I : Aufbau rule, II : Hund's rule III : Pauli's exclusion principle
Ar ­­ ­ ­ ­ ­ ­¯
3d 4s
(A) I, II (B) II, III (C) I, III (D) III

72. How many elements would be in the second period of the periodic table if the spin quantum number (ms)
1 1
could have the value of - ,0, + ?
2 2
(A) 8 (B) 10 (C) 12 (D) 18

73. The sub-shell that arises after f sub-shell is called g sub-shell.


(A) it contains 18 electrons and 9 orbitals
(B) it corresponds to l = 4 and first occurs in 5th energy level
(C) a g-orbital can have maximum of two electrons
(D) all the above statements are true.

SECTION - 4 : MATRIX - MATCH QUESTIONS


JPR\COMP.251\D\Allen(IIT-JEE Wing)\2020–21\Nurture\Che\Unit-01\Atomic Structure

74. Column-I Column-II


(A) Aufbau principle (p) Line spectrum in visible region
(B) de broglie (q) Orientation of an electron in an orbital
(C) Angular momentum (r) Photon
(D) Hund's rule (s) l = h/mv
(E) Balmer series (t) Electronic configuration
(F) Planck's law (u) mvr

182
Atomic Structure

75. Given in hydrogenic atom rn, Vn, E, Kn stand for radius, potential energy, total energy and kinetic energy in nth
orbit. Find the value of U,v,x,y.
Column - I Column - II
Vn
(A) U = (p) 1
Kn
1
x
(B)
rn µ E (q) –2

(C) rn µ Zy (r) –1
(Z = Atomic number)
(D) v = (Orbital angular momentum of electron (s) 0
in its lowest energy )

SECTION - 5 : NUMERICAL ANSWER BASED QUESTIONS


76. Calculate the number of exchange pairs of electrons present in configuration of Cu according to Aufbau principle
considering 3d orbitals.

77. How many electrons are present in P–shell.

78. According to Aufbau’s Principle, the maximum of electron that can be accomodated in the outermost orbit
(ab) and the penultimate orbit (cd) is :

79. Identify number of correct statements.


(A) Weight of 0.002 gm–atom of fluorine is 38 mg.
(B) Average atomic weigth of an element is the weight of most of the atoms.
(C) The value of ‘l’ for 5th electron of any element (Z > 5) will be same.
(D) In case of CO weight of carbon atoms is equal to weight of oxygen atoms.
(E) The value of ‘m’ must be zero for last electron of Ne.
(F) To obtain 0.1 mole of K2SO4.Al2(SO4)3.24H2O weight of oxygen required is 64 g.
(G) N–3 and P–3 are isoelectronic.
(H) Hydrogen(H) and Deuterium (D) are isobars.
80. Find the number of electrons having the value of azimuthal quantum number ‘l’ = 1 for Cd2+.
81. Calculate the binding energy per mole when threshold wavelength of photon is 240 nm.
82. The wavelength of a certain line in the Paschen series is 1093.6 nm. What is the value of nhigh for this line
[RH = 1.0973 × 10+7 m–1].
SECTION - 6 : SUBJECTIVE QUESTIONS
83. A gas of identical H-like atom has some atoms in the lowest (ground) energy level A and some atoms in a particular
upper (excited) energy level B and there are no atoms in any other energy level. The atoms of the gas make
transition to a higher energy level by absorbing monochromatic light of photon energy 2.7 eV. Subsequently,
JPR\COMP.251\D\Allen(IIT-JEE Wing)\2020–21\Nurture\Che\Unit-01\Atomic Structure

the atoms emit radiation of only six different photons energies. Some of the emitted photons have energy
2.7 eV. Some have more and some have less than 2.7 eV.
(a) Find the principal quantum number of initially excited level B.
(b) Find the ionisation energy for the gas atoms.
(c) Find the maximum and the minimum energies of the emitted photons.

84. An energy of 68 eV is required to excite a hydrogen like atom from its second Bohr orbit to the third. The nuclear
charge is Ze. Find the value of Z, the kinetic energy of the electron in the first Bohr orbit and the wavelength
of the radiation required to eject the electrons from the first Bohr orbit to infinity.

183
JEE-Chemistr y
85. The ionisation energy of a H-like Bohr atom is 4 Rydbergs.
(i) What is the wavelength of radiation emitted when the e– jumps from the first excited state to the ground
state?
(ii)What is the radius of first Bohr orbit for this atom? [1 Rydberg = 2.18 × 10–18 J]

86. Photon having wavelength 12.4 nm was allowed to strike a metal plate having work function 25 eV. Calculate
the -
(a) Maximum kinetic energy of photoelectrons emitted in eV.
(b) Wavelength of electron with maximum kinetic energy in A°.
(c) Calculate the uncertainity in wavelength of emitted electron, if the uncertainity in the momentum is
6.62 × 10–28 kg m/sec.

87. The angular momentum of an electron in a Bohr's orbit of H-atom is 3.1652 × 10–34 kg-m2/sec. Calculate the
wave number in terms of Rydberg constant (R) of the spectral line emitted when an electron falls from this level
to the ground state. (Use h = 6.626 × 10–34 Js).

88. Wavelength of the Balmer Ha line (first line) is 6565 Å. Calculate the wavelength of Hb (second line).

89. Calculate the energy emitted when electrons of 1.0 g atom of hydrogen undergo transition giving the spectral
line of lowest energy in the visible region of its atomic spectrum.

90. A photon having l = 854 Å causes the ionization of a nitrogen atom. Give the I.E. per mole of nitrogen in
KJ.

91. Calculate energy of electron which is moving in the orbit that has its radius, Sixteen times the radius of first
Bohr orbit for H-atom.

-21.7 ´ 10-12
92. The electron energy in hydrogen atom is given by En = ergs. Calculate the energy required to
n2
remove an e– completely from n = 2 orbit. What is the largest wavelength in cm of light that can be used to
cause this transition.

93. The velocity of e– in a certain Bohr orbit of the hydrogen atom bears the ratio 1 : 275 to the velocity of light.
What is the quantum no. "n" of the orbit and the wave no. of the radiation emitted for the transition form the
quantum state (n + 1) to the ground state.

94. A doubly ionised lithium atom is hydrogen like with atomic number Z = 3. Find the wavelength of the radiation
required to excite the electron in Li2+ from the first to the third Bohr orbit.

95. Estimate the difference in energy between I and II Bohr Orbit for a hydrogen atom. At what minimum At. no.
a transition from n = 2 to n = 1 energy level would result in the emission of X-rays with l = 3.0 × 10–8 m?
Which hydrogen like species does this At. no. correspond to:

96. 1.8 g atoms of hydrogen are excited to radiations. The study of spectra indicates that 27% of the atoms are
in 3rd energy level and 15% of atoms in 2n d energy level and the rest in ground state. If I.P. of H is 21.7 ×
10–12 erg. Calculate.
(i) No. of atoms present in III & II energy level.
JPR\COMP.251\D\Allen(IIT-JEE Wing)\2020–21\Nurture\Che\Unit-01\Atomic Structure

(ii) Total energy evolved when all the atoms return to ground state.

97. The vapours of Hg absorb some electrons accelerated by a potential difference of 4.5 volt as a result of it light
is emitted. If the full energy of single incident e– is supposed to be converted into light emitted by single Hg
atom, find the wave no. of the light.

98. A stationary He+ ion emitted a photon corresponding to a first line of the Lyman series. The photon liberated
a photoelectron from a stationary H atom in ground state. What is the velocity of photoelectron ?

99. The hydrogen atom in the ground state is excited by means of monochromatic radiation of wavelength x A0.
The resulting spectrum consists of 15 different lines. Calculate the value of x.

184
Atomic Structure
100. If the average life time of an excited state of H atom is of order 10 sec, estimate how many orbits an e– makes
–8

when it is in the state n = 2 and before it suffers a transition to n = 1 state.

101. The energy levels of hypothetical one electron atom are shown below.
0 eV —— n = ¥
–0.50 eV —— n = 5
–1.45 eV —— n = 4
–3.08 eV —— n = 3
–5.3 eV —— n = 2
–15.6 eV —— n = 1
(a) Find the ionisation potential of atom?
(b) Find the short wavelength limit of the series terminating at n = 2 ?
(c) Find the wave no. of photon emitted for the transition made by the electron from third orbit to first
orbit ?
(d) Find the minimum energy that an electron will have after interacting with this atom in the ground
state, if the initial kinetic energy of the electron is (i) 6eV (ii) 11 eV ?

102. Suppose 10–17 J of light energy is needed by the interior of the human eye to see an object. How many photons
of green light (l = 550 nm) are needed to generate this minimum amount of energy?

103. Find the number of photons of radiation of frequency 5 × 1013 s–1 that must be absorbed in order to melt one
g ice when the latent heat of fusion of ice is 330 J/g.

104. The eyes of certain member of the reptile family pass a single visual signal to the brain when the visual receptors
are struck by photons of wavelength 850 nm. If a total energy of 3.15 × 10–14 J is required to trip the signal,
what is the minimum number of photons that must strike the receptor?

105. To what series does the spectral lines of atomic hydrogen belong if its wave number is equal to the
difference between the wave number of the following two lines of the Balmer series 486.1 and 410.2 nm. What
is the wavelength of this ?

106. Calculate the threshold frequency of metal if the binding energy is 180.69 kJ mol–1 of electron.

107. A metal was irradiated by light of frequency 3.2 × 1015 s–1. The photoelectron produced had its KE,
2 times the KE of the photoelectron which was produced when the same metal was irradiated with a light of
frequency 2.0 × 1015 s–1. What is work function ?

108. U.V. light of wavelength 800 A° & 700 A° falls on hydrogen atoms in their ground state & liberates electrons
with kinetic energy 1.8 eV and 4 eV respectively. Calculate planck's constant.

109. A potential difference of 20 kV is applied across an X-ray tube. Find the minimum wavelength of X-ray generated.

110. The K.E. of an electron emitted from tungsten surface is 3.06 eV. What voltage would be required to bring the
electron to rest.

æ 8kT ö
111. What is de-Broglie wavelength of a He-atom in a container at room temperature. ç Use U avg. = ÷
JPR\COMP.251\D\Allen(IIT-JEE Wing)\2020–21\Nurture\Che\Unit-01\Atomic Structure

è pm ø

112. Through what potential difference must an electron pass to have a wavelength of 500 A°.

113. A proton is accelerated to one tenth of the velocity of light. If its velocity can be measured with a precision
± 1%. What must be its uncertainty in position ?

114. To what effective potential a proton beam be subjected to give its protons a wavelength of 1 × 10–10 m.

115. He atom can be excited to 1s1 2p1 by l = 58.44 nm. If lowest excited state for He lies 4857 cm–1 below the
above. Calculate the energy for the lower excitation state.

185
JEE-Chemistr y
116. A certain dye absorbs 4530 A° and fluoresence at 5080 A° these being wavelengths of maximum absorption
that under given conditions 47% of the absorbed energy is emitted. Calculate the ratio of the no. of quanta
emitted to the number absorbed.

117. The dissociation energy of H2 is 430.53 kJ/mol. If H2 is exposed to radiant energy of wavelength 253.7 nm,
what % of radiant energy will be converted into K.E ?

118. Iodine molecule dissociates into atoms after absorbing light of 4500 A0 If one quantum of radiation is absorbed
by each molecule, calculate the K.E. of iodine atoms.
(Bond energy of I2 = 240 kJ/mol)

119. What is de-Broglie wavelength associated with an e– accelerated through potential difference of 100 kV ?

120. Calculate the de-broglie wavelength associated with motion of earth (mass 6 × 1024 kg) orbiting around the
sun at a speed of 3 × 106 m/s.

121. A base ball of mass 200 g is moving with velocity 30 × 102 cm/s. If we can locate the base ball with error
equal in magnitude to the l of the light used (5000 Å), how will the uncertainty in momentum compared with
the total momentum of base ball ?

122. An electron has a speed of 40 m/s, accurate up 99.99 %. What is the uncertainty in locating position?

123. Instead of principal quantum number (n), azimuthal quantum number (l) & magnetic quantum number m, a
set of new quantum numbers s, t and u was introduced with similar logic but different values as defined below.
s = 1, 2, 3, ........¥ all positive integral values.
t = (s2 – 12), (s2 – 22), (s2 – 32) ........ No negative value
(t + 1) (t + 1)
u= - to + (including zero, if any) in integral steps.
2 2
Each orbital can have maximum four electrons.
(s + t) rule is defined, similar to (n + l) rule.
(i) Number of electrons that can be accommodated in s = 2 and s = 3 shell.
(ii) Number of electrons for which s = 2, t = 3 for an element with atomic number 24.
(iii) The number of subshells in which the third shell is subdivided equal to

124. Calculate the Rydberg constant R if He+ ions are known to have the wavelength difference between the first
(of the longest wavelength) lines of Balmer and Lyman series equal to 133.7 nm.

125. Calculate the wavelength in angstrom of photon that is emitted when an e– in Bohr orbit n = 2 returns to the
orbit n = 1. The ionization potential of the ground state of hydrogen atom is 2.17 × 10–11 erg/atom.

126. One mole He+ ions are excited. Spectral analysis showed existence of 50% ions in 3rd orbit, 25% in
2nd and rest in ground state. Calculate total energy evolved when all the ions return to the ground
state.

127. Calculate the frequency of e– in the first Bohr orbit in a H-atom.


JPR\COMP.251\D\Allen(IIT-JEE Wing)\2020–21\Nurture\Che\Unit-01\Atomic Structure

128. To what series does the spectral lines of atomic hydrogen belong if its wave number is equal number is equal
to the difference between the wave numbers of the following two lines of the Balmer series 486.1 and 410.2
nm? What is the wavelength of this line ?

129. Hydrogen atom in its ground state is excited by means of monochromatic radiation of wavelength
975 A°. How many different lines are possible in the resulting spectrum? Calculate the longest wavelength amongst
them.

186
Atomic Structure
130. (a) The Schrodinger wave equation for hydrogen atom is
3/2
1 æ 1 ö æ r0 ö - r / a
y 2s = 1/ 2 ç ÷ ç2- ÷e
4(2p) è a 0 ø è a0 ø

Where a0 is Bohr's radius. Let the rdial node in 2s be at r0. Then find r0 in terms of a0.
(b) A base ball having mass 100 g moves with velocity 100 m/s. Find out the value of wavelength of base ball.

131. (a) Calculate velocity of electron in first Bohr orbit of hydrogen atom (Given r = a0)
(b) Find de-Broglie wavelength of the electron in first Bohr orbit.
(c) Find the orbital angular momentum of 2p-orbital in terms of h/2p units.

SECTION - 7 : ASSERTION–REASON QUESTIONS


These questions contains, Statement I (assertion) and Statement II (reason).
(A) Statement-I is true, Statement-II is true ; Statement-II is correct explanation for Statement-I.
(B) Statement-I is true, Statement-II is true ; Statement-II is NOT a correct explanation for statement-I
(C) Statement-I is true, Statement-II is false
(D) Statement-I is false, Statement-II is true
132. Statement-I : Nodal plane of px atomic orbital is yz plane.
Because
Statement-II : In px atomic orbital electron density is zero in the yz plane.

133. Statement-I : No two electrons in an atom can have the same values of four quantum numbers.
Because
Statement-II : No two electrons in an atom can be simultaneously in the same shell, same subshell, same
orbitals and have same spin.

134. Statement-I : p-orbital has dumb-bell shape.


Because
Statement-II : Electrons present in p-orbital can have one of three values for 'm', i.e. 0, +1, –1.

135. Statement-I : The ground state configuration of Cr is 3d5 4s1.


Because
Statement-II : A set of exactly half filled orbitals containing parallel spin arrangement provide extra
stability.

136. Statement-I : Limiting line in the balmer series has a wavelength of 36.4 mm.
Because
Statement-II : Limiting lines is obtained for a jump of electron from n = ¥ to n = 2 for Balmer series.

137. Statement-I : The electronic configuration of nitrogen atom is represented as :

not as
JPR\COMP.251\D\Allen(IIT-JEE Wing)\2020–21\Nurture\Che\Unit-01\Atomic Structure

Because
Statement-II : The configuration of ground state of an atom is the one which has the greatest multiplicity.

138. Statement-I : The configuration of B atom cannot be 1s2 2s3.


Because
Statement-II : Hund's rule demands that the configuration should display maximum multiplicity.
139. Statement-1 : The groundstate configuration of Cr is [Ar] 3d54s1
Statement-2 : The energy of atom is lesser in 3d5 4s1 configuration compared to 3d4 4s2 configuration.

140. Statement-1 :Minimum principal quantum number of an orbital belonging to 'g' sub-shell is 5.
Statement-2 : For a given value of principal quantum number (n), l may have values 0 to (n–1) only.

187
JEE-Chemistr y

ANSWERS
l SINGLE CHOICE CORRECT QUESTIONS
1. (A) 2. (A) 3. (A) 4. (A) 5. (C)
6. (A) 7. (C) 8. (A) 9. (B) 10. (C)
11. (C) 12. (B) 13. (D) 14. (D) 15. (A)
16. (B) 17. (A) 18. (C) 19. (B) 20. (B)
21. (C) 22. (C) 23. (C) 24. (D) 25. (D)
26. (B) 27. (B) 28. (A) 29. (D) 30. (B)
31. (D) 32. (B) 33. (A) 34. (C) 35. (D)
36. (A) 37. (B) 38. (D) 39. (A) 40. (D)
41. (D) 42. (B) 43. (D) 44. (C) 45. (C)
46. (A) 47. (B) 48. (C) 49. (A) 50. (C)
51. (C) 52. (B) 53. (D) 54. (A) 56. (C)
57. (D) 58. (C)

l MULTIPLE CHOICE CORRECT QUESTIONS


59. (ACD) 60. (AC) 61. (ABCD) 62. (AC) 63. (ACD)
64. (ABC) 65. (BCD) 66. (AD) 67. (BC)

l COMPREHENSION BASED QUESTIONS


68. (B) 69. (A) 70. (A) 71. (D) 72. (C)
73. (D)

l MATRIX MATCH QUESTIONS


74. (A) - (t), (B) - (s), (C) - (u), (D) - (q), (E) - (p), (F) - (r)
75. (A)- (q), (B)- (p), (C) - (r), (D) - (s)

l NUMERICAL ANSWER BASED QUESTIONS


76. (16) 77. (72) 78. (818) 79. (3) 80. (18)
81. (497) 82. (6)

l SUBJECTIVE QUESTIONS
83. (a) n = 2, (b) 14.4 eV, (c) 13.5eV, 0.7eV 84. 6 ; 489.6 eV, 25.28 A°
85. 300.89 A°, 2.645 × 10–9 cm

æ8ö
86. (a) 75 eV ; (b) 1.414 A° ; (c) 2 × 10–14 m 87. Rç ÷
è9ø
88. 4863 Å 89. 1.827 × 105 J/mol
90. 1403 kJ/mol 91. –1.36 × 10–19 Joules
92. 5.425 × 10–12 ergs, 3.7 × 10–5 cm 93. 2 ; 9.75 × 104 cm–1
94. 113.74 Å 95. 10.2 eV, Z=2
96. 292.68 × 1021 atoms, 162.60 × 1021 atoms, 832.50 kJ
97. 3.63×106m–1 98. 3.09 × 108 cm/sec
99. 938 Å 100. 8 × 106
JPR\COMP.251\D\Allen(IIT-JEE Wing)\2020–21\Nurture\Che\Unit-01\Atomic Structure

101. (a) 15.6 eV


(b) 233.9 nm,
(c) 1.008 × 107 m–1
(d) (i) electron will not interact (ii) 0.7 eV
102. 28 photons 103. 1022
104. 1.35 × 105 105. Brackett ; 2.63 × 10–4 m
106. 4.5 × 1014 s–1 107. 319.2 kJ/mol
108. 6.57 × 10–34 Js 109. 0.62 Å
110. 3.06 V 111. 0.79 Å
112. 6.03×10–4volt 113. 1.05 × 10–13m
114. 0.0826 volts 115. 3.3 × 10–18 J
116. 0.527 117. 8.68 %
188
Atomic Structure

118. 2.186 × 10–20 Joules 119. 3.88 pm


120. 3.68 × 10–65 m 121. 1.75 × 10–29
122. 0.0144 m 123. (i) 28, 76 ; (ii) (0) ; (iii) (3)
124. 1.096 × 107 m–1 125. 1220 A°
126. 331.13×104J 127. 6530 × 1012 Hz
128. n1 = 4, n2 = 6, 2.63 × 10–4 cm 129. six, 18800 Å
130. (a) r0 = 2a0 ; (b) 6.626 × 10–35 m
h
131. (a) 2.197 × 106 m/s; (b) 3.31 Å; (c) 2.
2p

l ASSERTION–REASON QUESTIONS
132. (A) 133. (A) 134. (B) 135. (A) 136. (A)
137. (A) 138. (B) 139. (A) 140. (A)
JPR\COMP.251\D\Allen(IIT-JEE Wing)\2020–21\Nurture\Che\Unit-01\Atomic Structure

189

You might also like